USMLE Paso 3

  • 0 0 0
  • Like this paper and download? You can publish your own PDF file online for free in a few minutes! Sign Up
File loading please wait...
Citation preview

http://bookmedico.blogspot.com

LAN G E Q &A



USM LE PASO 3 CUARTA EDICIÓN

Donald A. Briscoe, MD Associate Program Director Family Medicine Residency Program CHRISTUS St. Joseph Hospital Houston, Texas

Traducción: Dr. Germán Arias Rebatet

ERRNPHGLFREORJVSRWFRP MÉXICO • BOGOTÁ • BUENOS AIRES • CARACAS • GUATEMALA LISBOA • MADRID • NUEVA YORK • SAN JUAN • SANTIAGO • SAO PAULO AUCKLAND • LONDRES • MILÁN • MONTREAL • NUEVA DELHI SAN FRANCISCO • SINGAPUR • ST. LOUIS • SIDNEY • TORONTO

http://bookmedico.blogspot.com

http://bookmedico.blogspot.com Director editorial: Marco Antonio Tovar Sosa Editor sponsor: Javier De León Fraga Corrección de estilo: Norma Angélica García Carbajal Supervisora de edición: Leonora Véliz Salazar Supervisora de producción: Olga Adriana Sánchez Navarrete

NOTA La medicina es una ciencia en constante desarrollo. Conforme surjan nuevos conocimientos, se requerirán cambios de la terapéutica. El(los) autor(es) y los editores se han esforzado para que los cuadros de dosificación medicamentosa sean precisos y acordes con lo establecido en la fecha de publicación. Sin embargo, ante los posibles errores humanos y cambios en la medicina, ni los editores ni cualquier otra persona que haya participado en la preparación de la obra garantizan que la información contenida en ella sea precisa o completa, tampoco son responsables de errores u omisiones, ni de los resultados que con dicha información se obtengan. Convendría recurrir a otras fuentes de datos, por ejemplo, y de manera particular, habrá que consultar la hoja informativa que se adjunta con cada medicamento, para tener certeza de que la información de esta obra es precisa y no se han introducido cambios en la dosis recomendada o en las contraindicaciones para su administración. Esto es de particular importancia con respecto a fármacos nuevos o de uso no frecuente. También deberá consultarse a los laboratorios para recabar información sobre los valores normales.

LANGE Q & A PARA EL USMLE PASO 3 Prohibida la reproducción total o parcial de esta obra, Por cualquier medio, sin autorización escrita del editor.

DERECHOS RESERVADOS © 2007, respecto a la primera edición en español por, McGRAW-HILL INTERAMERICANA EDITORES, S. A. de C. V. A subsidiary of the McGraw-Hill Companies, Inc. Prolongación Paseo de la Reforma 1015, Torre A, Piso 17, Col. Desarrollo Santa Fe, Delegación Alvaro Obregón C. P. 01376, México, D. F. Miembro de la Cámara Nacional de la Industria Editorial Mexicana Reg. No. 736

ISBN 970-10-6127-6

Translated from the fifth English edition of: Lange Q & A. USMLE, step 3 By: Donald A. Briscoe Copyright © 2005 by McGraw-Hill Companies, Inc. All Rights Reserved ISBN: 0-07-144579-X

1234567890 Impreso en México

09865432107 Printed in Mexico

ERRNPHGLFREORJVSRWFRP http://bookmedico.blogspot.com

http://bookmedico.blogspot.com

Contenido

ERRNPHGLFREORJVSRWFRP Colaboradores .........................................................................................................................................................v Introducción ......................................................................................................................................................... vii Agradecimientos ..................................................................................................................................................xv Cuadro de cifras normales de exámenes de laboratorio ........................................................................... xvii 1. Medicina interna............................................................................................................................................. 1 Preguntas .......................................................................................................................................................... 1 Respuestas y explicaciones........................................................................................................................... 35 Bibliografía ..................................................................................................................................................... 67 2. Cirugía ............................................................................................................................................................ 69 Preguntas ........................................................................................................................................................ 69 Respuestas y explicaciones........................................................................................................................... 89 Bibliografía.................................................................................................................................................... 110 3. Pediatría ........................................................................................................................................................ 111 Preguntas ...................................................................................................................................................... 111 Respuestas y explicaciones......................................................................................................................... 126 Bibliografía.................................................................................................................................................... 138 4. Ginecología y obstetricia .......................................................................................................................... 139 Preguntas ...................................................................................................................................................... 139 Respuestas y explicaciones......................................................................................................................... 157 Bibliografía.................................................................................................................................................... 178 5. Psiquiatría .................................................................................................................................................... 179 Preguntas ...................................................................................................................................................... 179 Respuestas y explicaciones......................................................................................................................... 196 Bibliografía.................................................................................................................................................... 212 6. Medicina preventiva .................................................................................................................................. 213 Preguntas ...................................................................................................................................................... 213 Respuestas y explicaciones......................................................................................................................... 228 Bibliografía.................................................................................................................................................... 248

http://bookmedico.blogspot.com

3

http://bookmedico.blogspot.com 7. Patología ....................................................................................................................................................... 249 Preguntas ......................................................................................................................................................249 Respuestas y explicaciones.........................................................................................................................259 Bibliografía....................................................................................................................................................267 8. Examen de práctica 1 .................................................................................................................................. 269 Preguntas ...................................................................................................................................................... 269 Respuestas y explicaciones......................................................................................................................... 280 Bibliografía.................................................................................................................................................... 291 9. Examen de práctica 2 .................................................................................................................................. 293 Preguntas ...................................................................................................................................................... 293 Respuestas y explicaciones......................................................................................................................... 304 Bibliografía.................................................................................................................................................... 315 10. Examen de práctica 3 .................................................................................................................................. 317 Preguntas ...................................................................................................................................................... 317 Respuestas y explicaciones......................................................................................................................... 327 Bibliografía.................................................................................................................................................... 338 11. Examen de práctica 4 .................................................................................................................................. 339 Preguntas ...................................................................................................................................................... 339 Respuestas y explicaciones......................................................................................................................... 349 Bibliografía.................................................................................................................................................... 359 Índice .................................................................................................................................................................... 361 Las imágenes a color se encuentran entre las páginas 110 y 111.

ERRNPHGLFREORJVSRWFRP

http://bookmedico.blogspot.com

http://bookmedico.blogspot.com iv

Contenido

Colaboradores

Douglas Adler, MD

Edison Catalano, MD

Assistant Profesor University of Texas Medical School at Houston Houston, Texas Medicina Interna-Gastroenterología

Professor and Chief Robert Wood Johnson medical School at Camden Camden, New Jersey Pathología

S. Sohail Ahmed, MD

Abigail Caudle, MD

Assistant Professor Boston University Scholl of Medicine Boston, Massachusetts Medicina Interna Reumatología

Chief Resident University of North Carolina Scholl of Medicine Chapel Hill, North Carolina Cirugía

Cesar Arias, MD

Evelyn Chan, MD

Residente University of Texas Medical School at Houston Houston, Texas Medicina Interna

Assistant Professor University of Texas Medical School at Houston Houston, Texas Medicina Interna- Ética Médica

Sean Blitzstein, MD

Christian Chisholm, MD

Clinical Assistant Professor of Psychiatry University of Illinois at Chicago Chicago, Illinois Psiquiatría

Assistant professor University of Virginia School of Medicine Charlottesville, Virginia Obstetricia y ginecología

Kristina Bogar, DO

Mohamed Elfar, MD

Chief resident, Family Practice CHRISTUS St. Joseph Hospital Houston, Texas Medicina Interna, Pediatría

Resident, Surgery CHRISTUS St. Joseph Hospital Houston, Texas Cirugía

Donald A. Briscoe, MD

Miguel Escobar, MD

Associate Program Director Family Medicine Residency Program CHRISTUS St. Joseph Hospital Houston, Texas Medicina Preventiva, Medicina Interna

Assistant Professor University of Texas Medical School at Houston Houston, Texas Medicina Interna-Hematología

Elise Everett, MD Joan Bull, MD Professor University of Texas Medical School at Houston Houston, Texas Medicina Interna-Oncología

Clinical Instructor University of Virginia School of Medicine Charlottesville, Virginia Obstetricia y ginecología

http://bookmedico.blogspot.com

5

http://bookmedico.blogspot.com Christopher Greeley, MD

Uday Khosla, MD

Assistant Professor Vanderbilt University School of Medicine Nashville, Tennessee Pediatría

Práctica privada Houston, Texas Medicina Interna-Nefrología

Hong Jin Kim, MD Vanessa Gregg, MD Resident University of Virginia School of Medicine Charlottesville, Virginia Obstetricia y ginecología

Assistant Professor University of North Carolina School of Medicine Chapel Hill, North Carolina Cirugía

Steven Mays, MD Eric Haas, MD

Assistant Professor University of Texas Medical School at Houston Houston, Texas Medicina Interna-Dermatología

Colorectal Surgical Associates Department of Surgery CHRISTUS St. Joseph Hospital Houston, Texas Cirugía

Amal Melhem, MD Residente University of Texas medical School at Houston Houston, Texas Medicina Interna

Mark Hughes, MD Assistant Professor The Johns Hopkins School of Medicine Baltimore, Maryland Medicina Interna- Ética médica

Philip Orlander, MD

Kathie Hullfish, MD Associate Professor University of Virginia School of Medicine Charlottesville, Virginia Obstetricia y ginecología

William Irvin, MD Associate Professor University of Virginia School of Medicine Charlottesville, Virginia Obstetricia y ginecología

David Johnson, PhD Associate Professor Duquesne University Mylan School of Pharmacy

Professor University of Texas Medical School at Houston Houston, Texas Medicina Interna-Endocrinología

Alberto Puig, MD, pHD Assistant Professor University of Texas Medical School at Houston Houston, Texas Medicina Interna- Medicina General

Husam Saad Eddin, MD Residente, Family Practice CHRISTUS St. Joseph Hospital Houston, Texas Medicina Preventiva, Medicina Interna

http://bookmedico.blogspot.com

6

http://bookmedico.blogspot.com vi

Colaboradores

Introducción

Este libro se diseñó para las personas que preparan el United States Medical Licensing Examination (USMLE) paso 3. Proporciona una revisión amplia de más de 850 preguntas de opción múltiple “similares al examen” que revisan los aspectos de las ciencias clínicas. Se proporcionan explicaciones detalladas para cada pregunta que tienen como objeto explicar por qué la respuesta es correcta y, cuando sea apropiado, por qué las respuestas son incorrectas. Además, la última sección de este libro tiene exámenes de práctica integrados, que abarcan múltiples especialidades y que proporcionan un mecanismo para autoevaluación y para simular las preguntas de opción múltiple del examen USMLE paso 3.

United States Medical Licensing Examination, paso 3

Objetivo del examen El objetivo del examen USMLE paso 3 es valorar si el médico posee y puede aplicar los conocimientos médicos y la comprensión de las ciencias clínicas consideradas esenciales para la práctica de la medicina sin supervisión, con énfasis en el tratamiento del paciente Pittsburgh, Philadelphia en la medicina ambulatoria. La inclusión del paso 3 en Medicina Interna- Farmacología la secuencia de exámenes USMLE asegura que se valore en forma apropiada el conocimiento y capacidad de los Philip C. Johnson, MD médicos que asumen la responsabilidad independiente Professor de proporcionar atención médica a los pacientes. University of Texas Medical School at Houston Houston, Texas Formato del examen Medicina Interna-Enfermedades infecciosas El examen paso 3 consiste de preguntas de opción

Judianne MDcomputadorizadas, distribuimúltiple Kellaway, y simulaciones

Associate Professor das de acuerdo al contenido del programa. El material University of Texas Medical School Houstonque tienen de examen es preparado por losatcomités Houston, Texas representación amplia en la profesión médica. Los Oftalmología

comités están formados por expertos reconocidos en sus campos, académicos y no académicos, practicantes de la medicina y por miembros del comité estatal de licencias médicas. El paso 3 es un examen de dos días de duración, 8 h por cada día. El examen tiene aproximadamente 480 preguntas de opción múltiple divididas en bloques de 35 o 50 preguntas. Se permiten 45 a 60 min para completar cada bloque, dependiendo del número de preguntas. También hay aproximadamente nueve simulaciones por computadora, con un caso en cada bloque, que debe completarse en el segundo día de examen. Se cuenta con 25 min para completar cada bloque. Se dispone de 45 min de descanso por día. Cada tiempo de descanso puede dividirse en la forma que el alumno elija. Se puede tomar un corto descanso entre bloques o un descanso más largo para el almuerzo. Si se completa el bloque antes, se puede utilizar el tiempo restante para descansos. No se puede utilizar el tiempo sobrante para completar otros bloques de la prueba. Si el alumno toma mucho tiempo de descanso, se reduce el tiempo necesario para terminar el último bloque. Si se agota el tiempo mientras responde un bloque, el alumno no podrá desplazarse a nuevas pantallas. Algunas versiones del software utilizado cierran la sesión mientras que otras le permiten responder la pregunta que está viendo en pantalla. En este último caso, el tiempo general de la sesión esta corriendo y el Margaret Walkup, para MD descansos o para completar la tiempo remanente Residente sesión se reduce. University of Northpaso Carolina School of en Medicine El examen 3 se organiza dos dimensioChapel Hill, North Carolina nes principales: casos clínicos y conocimientos médicos. Cirugía Los casos clínicos son las circunstancias que encuentra el médico en la práctica clínica con los pacientes. Christopher Williams, MD de pacientes por primera Estos incluyen valoraciones Assistant Professor vez en la sala de urgencias, pacientes conocidos para University of Virginia School of valorados Medicine en situaciones cuidado continuo y pacientes Charlottesville, Virginia que ponen en riesgo la vida. Estos casos clínicos pueden Obstetricia y ginecología presentarse en clínicas, consultorios, instituciones con enfermeras calificadas, hospitales, salas de urgencia o

http://bookmedico.blogspot.com

http://bookmedico.blogspot.com 8

Introducción

por vía telefónica. Cada pregunta también representa una de las seis principales ramas de la medicina. Estos son el interrogatorio y exploración física, valoración de exámenes de laboratorio y estudios diagnósticos, formulación del diagnóstico más probable, valoración de la gravedad de la enfermedad del paciente, aplicación de conocimientos científicos y mecanismos de la enfermedad y tratamiento del paciente. Cada tratamiento incluye aspectos de mantenimiento de la salud, intervenciones clínicas, tratamiento y aspectos legales y éticos. En el boletín de información del USMLE 2005 se especifica que los casos clínicos abarcan los siguientes aspectos: 20 a 30% de atención inicial; 50 a 60% de atención continua y 15 a 25% de atención de urgencia. Los conocimientos médicos se distribuyeron de la siguiente manera: 8 a 12% para obtener el interrogatorio y realizar la exploración física; 8 a 12% para la valoración de estudios de laboratorio y estudios diagnósticos; 8 a 12% para formular el diagnóstico más probable; 8 a 12% para valorar la gravedad de los problemas médicos del paciente; 8 a 12% para aplicar conceptos científicos y mecanismos de la enfermedad y 45 a 55% para el tratamiento del paciente. Estos porcentajes están sujetos a cambios. La información más actualizada se encuentra disponible en la página electrónica de USMLE (www. usmle.org). Durante el tiempo permitido para cada bloque individual, las preguntas pueden responderse en cualquier orden, revisarse y ser cambiadas. Después de salir de un bloque, no es posible realizar revisiones de las preguntas o cambiar las respuestas. Las políticas con respecto a la revisión de las preguntas pueden cambiarse sin notificación. Las políticas actuales con respecto a la revisión se encuentran publicadas en la página electrónica de USMLE en www.USMLE.org. La interfaz del programa de cómputo incluye, entre otras características, iconos para marcar las preguntas para revisión, revisión automatizada de preguntas marcadas o incompletas y un reloj que indica el tiempo restante. En la página electrónica de USMLE se encuentra un tutorial para la utilización del programa de cómputo. El día del examen también se dispone de un tutorial con duración de 15 min para su uso en ese día. El examen paso 3 se dirige a reflejar la diversidad de poblaciones que requieren atención médica con respecto a la edad, grupo cultural y ocupación. La distribución de la población de pacientes tiene por objeto ser representativa de los datos recopilados de varias bases de datos nacionales que estudian los servicios de salud en Estados Unidos.

Casos clínicos Las preguntas de opción múltiple se organizan en bloques que corresponden con los casos clínicos en los que se encontrará al paciente. Cada caso clínico se describe al inicio del bloque. Estas descripciones se muestran a continuación porque podrían aparecer durante el examen.

Consultorio/ centro de salud Se valora a los pacientes en dos ubicaciones: en el consultorio, que se encuentra adyacente a un hospital y en un centro de salud comunitario. En el consultorio se proporciona atención primaria por médicos generales. La mayoría de los pacientes que se valoran pertenecen a su propia consulta y acuden para visitas programadas regulares. En ocasiones se encontrarán pacientes que fueron tratados por un compañero. Las interconsultas estarán asentadas en los expedientes de los pacientes. Los pacientes conocidos pueden ser tratados por teléfono. Es necesario responder las preguntas con respecto a la información que aparece en la prensa y que requieren interpretación con base en las publicaciones médicas. Hay una amplia gama de servicios de estudios de laboratorio y radiología.

Pacientes hospitalizados El médico tiene autorización para hospitalizar pacientes, lo que incluye servicios de atención a niños y mujeres. En ocasiones se valorarán pacientes en las unidades de cuidados intensivos. Los pacientes posoperados por lo general son valorados en sus habitaciones, a menos que se especifique que la valoración se lleva a cabo en el área de recuperación posquirúrgica. También es posible que se llame al médico para valorar pacientes en unidades psiquiátricas. Es posible que se valoren pacientes en unidades de corta estancia después de cirugías ambulatorias o de pacientes que fueron hospitalizados para observación. También podría solicitarse valoración en asilos o instituciones de cuidados prolongados así como en unidades de destoxificación.

Sala de urgencias La mayoría de los pacientes presentados son nuevos para el médico, pero en ocasiones el médico acuerda valorar a un paciente conocido después de una comunicación telefónica. En general, en estos casos se valoran a pacientes que requieren atención urgente. También se cuenta con una amplia gama de servicios sociales, lo que incluye intervención para crisis después de violación, apoyo familiar y asistencia de seguridad por la policía local.

http://bookmedico.blogspot.com

http://bookmedico.blogspot.com Introducción

Formatos de preguntas del examen USMLE Paso 3

Preguntas de opción múltiple Se presentan en varias formas en cada bloque de preguntas. Cada uno de los formatos requiere elegir la mejor respuesta. Las instrucciones generales para responder las preguntas son las siguientes: lea cada pregunta cuidadosamente y en el orden en que se presentan. Después elija la mejor respuesta de las opciones ofrecidas. Más de una respuesta puede ser parcialmente correcta, pero debe elegirse la MEJOR respuesta al elegir con el ratón del ordenador la respuesta apropiada o presionar la letra en el teclado.

Preguntas simples Éste es el formato tradicional, utilizado con mayor frecuencia en las preguntas de opción múltiple. Por lo general consiste de un paciente en un caso clínico y la razón para la visita. Al caso clínico le continúan cuatro o cinco opciones de respuesta enumeradas con las letras A, B, C, D y E. Es necesario que elija la mejor respuesta a la pregunta. Las otras opciones pueden ser parcialmente correctas pero se elige sólo la MEJOR RESPUESTA. Proceso para contestar preguntas simples: • Lea cuidadosamente la descripción del caso clínico. • Intente formular una respuesta y después búsquela en la lista de respuestas. • Otro método consiste en leer cada respuesta cuidadosamente y eliminar las que considera incorrectas. • De las opciones restantes, elija una, la que considere que es más correcta. • Si no está seguro con respecto a la pregunta es mejor adivinar, porque una respuesta sin contestar se considera como incorrecta. A continuación se muestra un ejemplo de instrucciones de este tipo de pregunta:

Pregunta de ejemplo 1 1. Un varón de 45 años de edad de raza negra acude a consulta por primera vez porque comenta “oriné con sangre en la mañana”. El paciente niega otros síntomas. A la exploración física se encuentra aumento de la presión arterial y se palpan ambos riñones. La información en el interrogatorio que es más pertinente para el padecimiento actual es:

A) Uso crónico de analgésicos. B) Tabaquismo.

9

C) Antecedente familiar de nefropatía. D) Exposición ocupacional al tetracloruro de carbono. E) Infecciones faríngeas recientes. La respuesta correcta es C.

Grupos de varias preguntas El caso clínico de un mismo paciente puede acompañarse de dos o tres preguntas consecutivas con respecto a la información presentada. Cada pregunta está vinculada con el caso clínico, pero analiza puntos diferentes. Las preguntas se diseñaron para responderse de manera independiente una de otra. Se le pide que elija la mejor respuesta para cada pregunta. Otras opciones pueden ser parcialmente correctas pero sólo debe elegir la MEJOR RESPUESTA. El proceso para responder estas preguntas es el mismo que para el formato de preguntas simples.

Preguntas de ejemplo 2 a 4 Una mujer caucásica de 38 años de edad, profesora de medio tiempo y madre de tres niños, acude a consulta para valoración de hipertensión. Se le ha valorado desde el nacimiento de su primer hijo, hace ocho años. Una semana antes se detectó hipertensión arterial durante un examen programado para el ingreso a la escuela de graduados. Los signos vitales son:

Temperatura 37°C Pulso: 100 lpm Respiraciones: 22 por min TA: 164/100 mmHg (brazo derecho en decúbito dorsal) 2. El dato que con mayor probabilidad se debe encontrar en la exploración física es: A) B) C) D)

Soplo abdominal. Cardiomegalia. Disminución de los pulsos femorales. Aumento de tamaño de la glándula tiroides. E) Retinas normales. La respuesta correcta es E.

3. El paso más apropiado a continuación consiste en solicitar: A) Biometría hemática completa.

http://bookmedico.blogspot.com

http://bookmedico.blogspot.com 10

Introducción

B) C) D) E)

Electrólitos séricos y creatinina. Glucosa sérica. Tiroxina sérica. Urocultivo.

La respuesta correcta es B.

4. Para valorar los factores de riesgo de esta paciente para aterogénesis la prueba más apropiada es la cuantificación de: A) B) C) D) E)

Actividad de renina plasmática. Colesterol sérico. Triglicéridos séricos. Excreción de aldosterona en orina. Excreción de metanefrinas en orina.

La respuesta correcta es B.

Casos Con base en el caso clínico de un paciente o de una familia, se hacen hasta dos o tres preguntas relacionadas con el caso clínico. Se añaden datos conforme se conoce información del caso. Es sumamente importante responder las preguntas en el orden presentado. A menudo transcurre tiempo en el mismo caso y la orientación que al inicio se tenía puede alterarse con la información adicional presentada conforme avanza el caso. Si desea omitir preguntas, asegúrese de responder las preguntas previas con información presentada hasta ese punto. Cada pregunta está dirigida a responderse de manera independiente. Debe elegirse la MEJOR RESPUESTA para cada pregunta.

5. El dato en la exploración física que es más compatible con costocondritis como causa del dolor torácico es: A) Crepitación en la cara anterior del segundo y tercero arcos costales. B) Dolor a la palpación profunda del esternón con la mano. C) Área de dolor localizada en el área paraesternal. D) Dolor a la inspiración. E) Exploración física normal. La repuesta correcta es C.

6. Basado en que el paciente al inicio negaba el consumo de drogas, el paso más apropiado para confirmar el diagnóstico de uso de cocaína es: A) Preguntar al laboratorio si se dispone de un estudio toxicológico en una muestra sanguínea previa. B) Llamar a la familia para obtener información adicional. C) Medir la concentración de catecolaminas plasmáticas. D) Obtener una muestra de orina para análisis de orina programado pero también solicitar estudio toxicológico. E) Comentar los datos de la valoración con el paciente e interrogar con respecto al diagnóstico sospechado. La respuesta correcta es E.

Preguntas de ejemplo 5 a 7 Un varón de 24 años de edad acude a consulta por dolor torácico intermitente que inició hace unas cuantas semanas. Se ha valorado a este paciente durante los últimos dos años y ha estado por lo demás sano. Comenta que no tiene dolor al momento de la valoración. En la revisión de su expediente se encuentra que la concentración sérica de colesterol fue normal en un examen previo hace un año. No se le ha valorado desde dicha visita y comenta que no ha tenido otros síntomas o problemas en ese lapso de tiempo. El paciente fuma una cajetilla de cigarrillos al día. Con el interrogatorio dirigido, el sujeto comenta que no consume alcohol o drogas ilegales. Los detalles son vagos, pero describe que el dolor torácico es una sensación de opresión subesternal y que no se relaciona con el esfuerzo.

7. Se confirma el uso de cocaína. El paciente admite una posible relación temporal entre el uso de cocaína y el dolor torácico. Expresa su preocupación con respecto a los riesgos para la salud a largo plazo. El médico debe informarle que: A) La isquemia miocárdica inducida por cocaína puede tratarse con bloqueadores β. B) Puede ocurrir la muerte por infarto miocárdico o arritmia inducidos por cocaína. C) La presencia de secuelas neuropsiquiátricas por el consumo de drogas indica el riesgo de muerte súbita relacionado con el uso de cocaína.

http://bookmedico.blogspot.com

http://bookmedico.blogspot.com Introducción

D) El infarto miocárdico con onda Q ocurre sólo después de fumar “crack” o de la administración intravenosa de cocaína. E) La arteriopatía coronaria es el principal factor de riesgo para muerte cardiaca súbita relacionada con el consumo de cocaína. La respuesta correcta es B.

Simulación de casos por computadora (CCS) La simulación de casos por computadora le permite proporcionar atención médica a pacientes simulados. Se elige qué información diagnóstica obtener y cómo tratar y vigilar el progreso del paciente. En los casos clínicos simulados por computadora es posible solicitar información con respecto al interrogatorio y exploración física, solicitar exámenes de laboratorio, solicitar estudios diagnósticos, realizar procedimientos o interconsultas así como iniciar tratamiento médico u otras medidas terapéuticas. Conforme pasa el tiempo, el estado del enfermo cambia con base en el problema subyacente y las intervenciones solicitadas. Se deben vigilar los resultados de los exámenes que solicita y los resultados de las intervenciones que se realizan. Cuando se confirma que ya no hay nada más por hacer, se puede avanzar en el tiempo a fin de revalorar el estado del sujeto. No es posible regresar en el tiempo, pero puede cambiar las indicaciones médicas para reflejar la actualización del plan de tratamiento. El expediente del paciente contiene una hoja de órdenes y reportes con los resultados de los estudios solicitados. Se pueden revisar los signos vitales, las notas de progreso, las actualizaciones del paciente y los resultados de pruebas. Es posible desplazar a la persona al consultorio, el hogar, la sala de urgencias, la unidad de cuidados intensivos y el área de hospitalización. El proceso de calificación de los casos clínicos simulados por computadora compara la estrategia del tratamiento del paciente con las políticas señaladas por expertos. Las acciones similares dentro de un rango de estrategias óptimas producirán las calificaciones más altas. Las acciones peligrosas e innecesarias tendrán efectos negativos en la calificación. Es necesario balancear entre esmero, eficacia, evitación del riesgo y oportunidad para responder a las situaciones clínicas. El día del examen no se dispone de tiempo de práctica con el programa de casos clínicos simulados por computadora. Es necesario revisar el material de orientación sobre el CCS y practicar con los casos de ejemplo con anterioridad al día del examen a fin de comprender la forma en que trabaja el sistema. Los solicitantes pueden

11

tener acceso a un disco compacto con casos clínicos simples, los cuales también se encuentran disponibles en la página electrónica de USMLE.

Información específica con respecto al examen paso 3 El USMLE es patrocinado por la Federation of State Medical Boards (FSMB) y el National Board of Medical Examiners (NBME). Las reglas para el programa USMLE se establecen por un comité mixto que incluye representantes de FSMB, NBME, Educational Comission for Foreing Medical Graduates (ECFMG) y la American public. La información se publica en un boletín anual de información que se encuentra disponible en la página electrónica de USMLE (www.usmle.org). Es necesario familiarizarse con las políticas y procedimientos del boletín de información para el año en que se realizará el examen. Es posible que haya cambios en el programa USMLE después de la publicación del boletín. Si ocurren cambios la información se publica en la página electrónica de USMLE. La institución para el registro para el examen paso 3 es el FSMB. Debe establecerse contacto con esta federación para información con respecto a la forma en que se realiza la solicitud para el examen, materiales de información, información del estado de solicitud o programación o información para obtener una nueva cita (si no se presentó a la cita previa). Se puede establecer contacto con la FSMB en:

FSMB Department of Examination Services P.O. Box 619850 Dallas, TX 75261-9850 Página electrónica: www.fsmb.org Correo electrónico: [email protected] Para ser elegible para el examen USMLE paso 3 antes de enviar la solicitud el estudiante debe: • Satisfacer los requisitos para el examen paso 3 ante el departamento de licencia médicas en los que realizó la solicitud. • Tener el grado de médico (o su equivalente) o el grado DO (dentista). • Aprobar los exámenes paso 1, paso 2 CK y si es necesario con base en las reglas, el examen paso 2 CS. • Obtener la certificación por el ECFMG o completar con éxito el programa de “la quinta vía” si es graduado de una escuela de medicina fuera de Estados Unidos o Canadá.

http://bookmedico.blogspot.com

http://bookmedico.blogspot.com 12

Introducción

Los procedimientos de solicitud para el examen paso 3 varían entre las diferentes jurisdicciones. Es necesario realizar la solicitud al menos tres meses antes de la fecha en la que se desea tomar el examen. El examen paso 3 se realiza en los centros de prueba Prometric en Estados Unidos y sus territorios. Prometric es una división de Thomson Learning. Una vez que se ha aprobado la solicitud, se asigna un periodo de elegibilidad. La solicitud le permite hacer una cita en el centro de pruebas Prometric. El periodo de elegibilidad inicia de inmediato y dura casi 90 días calendario, de forma que debe establecerse contacto con el centro Prometric de inmediato una vez de que se recibió el permiso para programar las pruebas. Si no es posible tomar el examen durante el periodo de elegibilidad, debe establecerse contacto con la FSMB para solicitar una prolongación del periodo de elegibilidad por tres meses (hay un costo y aplican ciertas restricciones). Si no se toma el examen en el periodo de elegibilidad y desea tomarlo en el futuro, es necesario enviar una nueva solicitud y realizar nuevamente los pagos.

Condiciones físicas El día de la prueba debe llegar al centro Prometric 30 min antes de la hora programada para el examen. Si se llega tarde podrían no admitirlo. Si se arriba 30 min después del tiempo programado no se le permitirá realizar el examen. Cuando llegue al centro de pruebas es necesario presentar su solicitud y alguna identificación aceptada. Éstas incluyen pasaporte, licencia de conducir con fotografía, tarjeta de identidad nacional, otra identificación emitida por el gobierno y la tarjeta de identificación emitida por ECFMG. Si no tiene su solicitud e identificación no se le admitirá en la prueba. El personal del centro de pruebas vigila todas las sesiones de pruebas. Deben seguirse las instrucciones en forma meticulosa durante el examen. No están autorizados a responder preguntas con respecto al contenido del examen, al programa o a la calificación. El día del examen no se permite llevar objetos personales al área de pruebas. Si se llevan objetos personales al centro de pruebas se guardarán en un armario designado o un cubículo fuera del área de pruebas. Se proporcionan superficies laminadas para escritura, marcadores y un borrador, que deben regresarse al final de la prueba. No se permite realizar notas excepto sobre los materiales proporcionados. No es posible abandonar el área de pruebas para descansos a menos que en el monitor de la computadora se observe el letrero de descanso. Durante los descansos es posible utilizar el teléfono u otros dispositivos de comunicación, pero sólo con propósitos no relacionados con el contenido de

la prueba. No está permitido extraer materiales (escritos, impresos, grabados, etc.) del centro de pruebas. Las reglas completas de conductas se encuentran disponibles en el boletín de información.

Organización de este libro Este libro está organizado para revisar cada una de las áreas de ciencias clínicas que se encontrarán en el examen paso 3. En cada capítulo se presentan primero las preguntas, seguidas por las respuestas y explicaciones y bibliografía para estudio adicional. Los formatos de preguntas se presentan en un estilo similar al que se tendrá el examen. Esto tiene por objeto que el estudiante se familiarice antes de la realización del examen. Como sucede en los exámenes reales, los exámenes de práctica se organizaron en bloques de 50 preguntas, que deben responderse en 1 h y se organizan en uno de los tres escenarios clínicos antes descritos (consultorio/centro de salud, pacientes hospitalizados, sala de urgencias). La cantidad de tiempo permitido es proporcional a la cantidad de tiempo disponible para cada bloque de preguntas durante el examen real.

Respuestas explicaciones y bibliografía En cada una de las secciones del libro, las secciones de preguntas se continúan con una sección que contiene las respuestas, explicaciones y bibliografía de las preguntas. Esta sección indica la respuesta a cada pregunta, proporciona una explicación y revisión de por qué las respuestas son correctas o incorrectas e informa donde es posible encontrar más información sobre el tema. Los autores recomiendan utilizar esta sección como base para el estudio y comprensión adicionales. Si se elige la respuesta correcta a una pregunta, se puede leer la explicación con fines de reforzamiento y para añadir conocimiento con respecto al tema. Si se elige una respuesta errónea, se puede ver la explicación con fines de análisis del material en cuestión. Se puede revisar la referencia para una revisión más a fondo.

Cómo utilizar este libro Hay dos formas lógicas para obtener la mayor utilidad del libro. Los autores lo denominan plan A y plan B. En el plan A se realizan los exámenes de práctica de acuerdo a las instrucciones. Después de contestar los ex{menes y verificar las respuestas, el número de preguntas que se respondieron en forma incorrecta son un

http://bookmedico.blogspot.com

http://bookmedico.blogspot.com Introducción

buen indicador de su conocimiento inicial y de los tipos de preguntas que se respondieron de forma incorrecta lo que ayudará a trabajar en la dirección correcta para la preparación y revisión. En este punto se pueden utilizar los capítulos de especialidades individuales del libro para ayudar a mejorar las áreas de debilidad relativa. En el plan B, se avanza primero a través de los capítulos de ciencias clínicas. Una vez que se ha completado este proceso, entonces se realizan los exámenes de práctica y se verifican las respuestas para valorar qué tan bien preparado se encuentra en ese momento. Si aún existe un área de debilidad significativa est{ a tiempo para tomar acciones correctivas. En el plan A, al tomar primero los exámenes de práctica se obtendrá retroalimentación rápida con respecto a las áreas iniciales de debilidad y fortaleza. Tal vez se encuentre que se conoce el material muy bien, lo cual indica que quizá sólo se necesita un curso rápido de revisión. Es bueno saberlo en etapas tempranas de la preparación para el examen. Por otra parte, se pueden

13

encontrar otras áreas específicas de debilidad (como pediatría y psiquiatría), lo que permite enfocarse a la preparación de esas áreas, no sólo en este libro sino en libros de texto de pediatría y psiquiatría. No obstante, es poco probable que se carezca de preparación previa antes de tomar el examen USMLE paso 3 (en especial porque usted posee este libro). Por tanto, sería más realista tomar el examen de práctica después de haber revisado las secciones de especialidad, como en el plan B. Esto tal vez le proporcione una situación más realista, porque pocas personas realizan un examen sin estudiar primero. En este caso el estudiante debe hacer ciertas revisiones (superficiales o profundas) y realizar algunos exámenes de práctica que reflejen el tiempo de estudio. Si después de la revisión de las secciones de especialidad y de realizar los exámenes de práctica su calificación indica áreas de debilidad, debe regresar a la sección temática y complementar la preparación con los libros de texto apropiados. Ahora respire profundo, relájese y buena suerte.

http://bookmedico.blogspot.com

http://bookmedico.blogspot.com Introducción

http://bookmedico.blogspot.com

xiii

http://bookmedico.blogspot.com

Agradecimientos

Sólo un nombre aparece en la portada de este libro, pero está lejos de ser el trabajo de un solo hombre. Deseo agradecer a todos los colaboradores de este libro. El proyecto me dio la oportunidad de trabajar con viejos amigos y con algunos nuevos amigos. Aprecio las excelentes colaboraciones y dedicación de todos. También deseo agradecer el trabajo de los colaboradores de las ediciones previas del libro, publicado como Appleton & Lange’s Review for USMLE step 3. Se han hecho cambios significativos en esta edición, basados en las ediciones previas. Deseo reconocer la colaboración de Catherine Johnson y del personal profesional de McGraw-Hill que me permitió embarcarme en este esfuerzo. También aprecio la colaboración de todas las personas del CHRISTUS St. Joseph Hospital. Mis colaboradores en la residencia de medicina familiar (Dan Kalb, Neeta Gautam, Juan Garcia, Olufunke Odetunde, Kent Lee y Anush Pillai) quienes me brindaron la oportunidad de trabajar y proporcionar comentarios útiles y sugerencias. Le agradezco a los residentes con los cuales tuve el privilegio de trabajar y de los cuales aprendí

algo nuevo cada día. Agradezco a Shaylor Thomas, Tracy Salinas, el personal clínico y administrativo del FPC y CMG por todo lo que hicieron durante la residencia y también a los pacientes. Deseo agradecer especialmente a Eugene Toy, quien me dio por primera vez la oportunidad y me introdujo en la responsabilidad de este libro. Esto no sería posible sin el apoyo de mi familia. A Heather, Cal, Casey y a mis padres, a quienes agradezco y les brindo mi amor. A Lisa Peden, Eileen Lafleur y a toda nuestra familia en Maryland y Texas, gracias por su ayuda durante este periodo de tiempo de gran trabajo. Por último, a nombre de todas las personas que participaron en este proyecto, dedico este libro a aquellos para quienes va dirigido, los residentes a la mitad de su entrenamiento médico. El examen USMLE paso 3 es sólo un obstáculo más a vencer en el camino. Les deseamos la mejor de las suertes en el examen y en su carrera profesional.

http://bookmedico.blogspot.com

Donald A. Briscoe, MD

15

http://bookmedico.blogspot.com Introducción

http://bookmedico.blogspot.com

xiii

http://bookmedico.blogspot.com

USMLE PASO 3. CIFRAS NORMALES DE LOS EXÁMENES DE LABORATORIO VALORES DE REFERENCIA SANGRE, PLASMA, SUERO * Aminotransferasa de alanina (ALT) sérica * Fosfatasa alcalina sérica Amilasa sérica * Aminotransferasa de aspartato (AST) * Bilirrubina sérica (adultos), total//directa Calcio sérico (total) * Colesterol sérico total HDL LDL Cortisol sérico nmol/L

* Creatinina sérica Electrólitos séricos * Sodio (Na+) * Potasio (K+) * Cloruro (Cl-) * Bicarbonato (HCO 3-) Ferritina sérica Hormona foliculoestimulante, suero/plasma

Gases en sangre arterial (aire ambiental) PO2 PCO 2 PH * glucosa sérica Inmunoglobulinas séricas IgA IgE IgG IgM Hierro Deshidrogenasa de lactato en suero Hormona luteinizante, suero/plasma

Osmolalidad sérica Fósforo (inorgánico) sérico Proteínas en suero Totales (en decúbito) Albúmina Globulina Hormona estimulante de tiroides (TSH), sérica Tiroxina (T4), sérica Triglicéridos Triyodotironina (T3), captación con resinas *Nitrógeno ureico en suero (BUN) Ácido úrico en suero LÍQUIDO CEFALORRAQUÍDEO Recuento celular Cloruro Gammaglobulinas Glucosa

10-40 U/L Varones: 30-100 U/L Mujeres: 45-115 U/L 25-125 U/L 15-40 U/L 0.1-1.0 mg/100 ml // 0.0-0.3 mg/100 ml 8.4-10.2 mg/100 ml

10-40 U/L Varones: 30-100 U/L Mujeres: 45-115 U/L 25-125 U/L 15-40 U/L 2-17 µmol/L//0-5 µmol/L 2.1-2.8 mmol/L

150-240 mg/100 ml 30-70 mg/100 ml 8 mg/100 ml Calcio sérico 1 cm en la dimensión máxima. Mientras más pequeña sea la enfermedad residual más prolongada será la supervivencia de la paciente, y las pacientes que no tienen enfermedad residual visible tienen la supervivencia más prolongada. Con base en la gran cantidad de información retrospectiva que apoya la importancia de la cirugía de citorreducción óptima, deben realizarse intentos durante la cirugía inicial de citorreducción para obtener la extirpación óptima del tumor, de preferencia sin enfermedad residual visible al finalizar la intervención. (Surgical

management of ovarian cancer. Semin Oncol. 2002 Feb;29(1 Suppl 1):3-8). 88. B) El nervio femoral es el lesionado con mayor frecuencia al momento de procedimientos quirúrgicos ginecológicos. Este nervio puede lesionarse al momento de la laparotomía por la colocación inapropiada de separadores con valvas fijas. Cuando las hojas de los separadores se colocan muy profundas sobre el borde externo de la pelvis pueden comprimir directamente el músculo psoas y, por tanto, al nervio femoral que transcurre en el interior de este músculo. Mientras más prolongada sea la compresión nerviosa, más pronunciada y de mayor duración será la lesión posoperatoria. El nervio femoral también puede lesionarse al momento de la operación vaginal como consecuencia de una posición inadecuada de litotomía, con flexión extrema de la cadera y extensión máxima de la rodilla. El nervio femoral es una rama del plexo nervioso lumbosacro y tiene función motora y sensorial. La lesión al nervio femoral se manifiesta con disminución o ausencia de los reflejos osteotendinosos profundos, incapacidad para levantar la pierna, flexionar la cadera o extensión de la rodilla. También puede haber pérdida de la sensibilidad cutánea sobre la cara anterior del muslo y sobre el borde interno del muslo y pantorrilla. Las lesiones neurológicas después de procedimientos quirúrgicos ginecológicos son poco frecuentes, y se observan en 1 a 3% de todos los procedimientos ginecológicos. Una vez que ocurren dichas lesiones, suele necesitarse fisioterapia posoperatoria y a menudo se requieren ortesis hasta que se restablece la función neurológica. Los riesgos potenciales para lesión neurológica al momento de la intervención quirúrgica pueden reducirse con la colocación cuidadosa de las hojas de los separadores al momento de la laparotomía y con la posición apropiada de litotomía al momento de procedimientos quirúrgicos vaginales. La trombosis venosa profunda por lo general se manifiesta con edema asimétrico de las extremidades inferiores en el posoperatorio, pero sin déficit motor o sensorial asociados. Un accidente cerebrovascular no detectado durante la intervención quirúrgica por lo general se manifiesta con déficit central amplio y no con déficit focales en la extremidad inferior que se menciona en este caso. La lesión al nervio ciático se manifiesta con un grupo diferente de déficit

http://bookmedico.blogspot.com

http://bookmedico.blogspot.com Respuestas: 88–91

neurológicos, lo que incluye la incapacidad para extender la cadera, para flexionar la rodilla, y para la eversión y dorsiflexión del tobillo. La diabetes no diagnosticada puede manifestarse con diversas secuelas neurológicas, lo que incluye neuropatía periférica, nefropatía y retinopatía. Rara vez la neuropatía diabética no diagnosticada se manifiesta con el déficit focal, que se observa en este caso clínico. (Irving W, Andersen W, Taylor P. Rice L. Minimizing the risk of neurologic injury in gynecologic surgery. Obstet Gynecol. 2004 Feb;103(2):374-382). 89. B) 90. C) 91. E)

Explicación de las preguntas 89 a 91 El trazo de vigilancia fetal muestra la presencia de desaceleraciones tempranas. Éstas se caracterizan por disminución gradual en la frecuencia cardíaca fetal y regreso gradual a las cifras iniciales, que se asocian con la contracción. El inicio y la recuperación de la frecuencia cardíaca son simultáneos con el inicio y recuperación de la contracción. Se cree que esto se debe a estimulación vagal por compresión de la cabeza fetal. No se relaciona con hipoxia o acidosis fetales y

177

no se precisa intervención alguna excepto por la vigilancia cuidadosa del trabajo de parto. Las desaceleraciones variables son causadas por compresión del cordón umbilical. Se caracterizan por disminución súbita en la frecuencia cardíaca. El inicio de las desaceleraciones con frecuencia varía en contracciones sucesivas, y por lo general duran menos de 2 min. Las desaceleraciones tardías son disminuciones graduales en la frecuencia cardíaca que inician durante el momento de contracción máxima o después de ésta y regresan a las cifras basales una vez que ha concluido la contracción. A menudo es la primera anomalía en la frecuencia cardíaca fetal que se observa en la hipoxia uteroplacentaria inducida. Los trastornos que causan hipotensión materna, actividad uterina excesiva o disfunción placentaria pueden inducir desaceleraciones tardías. La taquicardia fetal se define como la frecuencia cardíaca fetal basal >160 lpm y se considera grave si la frecuencia cardíaca es >180 lpm. La causa más común es la fiebre materna, pero también puede ser consecuencia de sufrimiento fetal, arritmias o de ciertos medicamentos. La hiperestimulación es un patrón de frecuencia cardíaca normal causado por contracciones uterinas frecuentes. Esto ocurre con mayor frecuencia durante el trabajo de parto que se incrementa con la administración de oxitocina. El tratamiento inicial incluye la reducción de la dosis o la interrupción de la administración de oxitocina. (Cunningham et al., pp. 336-344).

http://bookmedico.blogspot.com

http://bookmedico.blogspot.com 178

4: Ginecología y obstetricia

BIBLIOGRAFÍA

Beckmann CRB, Ling FW, Laube DW, et al., Obstetrics and Gynecology, 4th ed. Baltimore, MD: Lippincot Williams & Wilkins, 2002. Cunningham FG, et al. Williams Obstetrics, 21st ed. New York, NY: McGraw-Hill, 2001. Gabbe SG, Niebyl JR, Simpson JL. Obstetrics: Normal and Problem Pregnancies, 4th ed., New York, NY: Churchill Livingstone, 2002.

178

Speroff L, Glass RH, Kase NG. Clinical Gynecologic Endocrinology and Infertility, 6th ed. Baltimore, MD: Lippincot Williams & Wilkins, 1999. Stenchever M, et al. Comprehensive Gynecology, 4th ed. St. Louis, MO: Mosby, 2001.

http://bookmedico.blogspot.com

http://bookmedico.blogspot.com CAPÍTULO 5

Psiquiatría Preguntas Preguntas 1 y 2

Preguntas 3 y 4

Una mujer de 29 años de edad acude a la clínica de atención de primer contacto por cefaleas frecuentes de varios meses de evolución. Durante el interrogatorio se le observa temerosa y retraída, con poco contacto ocular y renuencia a responder las preguntas. Con el apoyo adicional es capaz de describir un intenso sentimiento de tristeza, junto con insomnio significativo, mala concentración, fatiga, anhedonía e hiporexia con pérdida de peso de 10 kg.

Un paciente de 40 años regresa a consulta con resultados de exámenes de laboratorio. El médico le comenta que los exámenes de sangre son compatibles con leucemia y se le informa que debe someterse a biopsia de médula ósea para precisar el diagnóstico. Mientras se le informa al paciente, él permanece silencioso, y con la mirada fija sobre el médico. Cuando el médico concluye, el paciente comenta “los médicos piensan que son muy inteligentes”. El paciente comenta que ha sido tratado en forma inadecuada por médicos en el pasado y que de hecho tiene varias demandas pendientes por mala práctica médica. El paciente cree que la biopsia fue recomendada sólo “porque los médicos desean utilizarme para publicaciones y mejorar en su carrera”.

1. ¿Cuál es la pregunta más importante que debe hacerse antes de que la paciente abandone el consultorio? A) ¿Ha bebido alcohol o ha utilizado drogas ilegales en fechas recientes? B) ¿Ha tomado algún medicamento de venta libre? C) ¿Ha recibido tratamiento por alguna enfermedad? D) ¿Se ha sentido de esta forma con anterioridad? E) ¿Ha pensado en lastimarse usted misma? 2. Se decide iniciar el tratamiento para síntomas depresivos con farmacoterapia. Con respecto a la selección de la clase específica de fármaco, ¿qué antecedente heredofamiliar es de gran importancia? A) B) C) D) E)

Alergias. Síntomas de depresión. Síntomas maníacos. Enfermedades. Farmacodependencia.

3. Con base en la información mencionada, ¿cuál es el diagnóstico más probable? A) Trastorno antisocial de la personalidad. B) Trastorno narcisista de la personalidad. C) Trastorno paranoide de la personalidad. D) Trastorno esquizoide de la personalidad. E) Trastorno esquizotípico de la personalidad. 4. ¿Cuál de los siguientes es el método más eficaz al interactuar con este paciente? A) Defender sus recomendaciones mencionando sus credenciales profesionales. B) Alentar al paciente para que hable con un psiquiatra a fin de que le proporcione apoyo adicional. C) Interpretar su ira como un mecanismo de defensa contra el temor de tener leucemia. D) Proporcionar información detallada con respecto a su diagnóstico diferencial. E) Enviarlo con otro médico para evitar demandas.

http://bookmedico.blogspot.com

http://bookmedico.blogspot.com 180

5: Psiquiatría

Preguntas 5 y 6 Una mujer casada, de 38 años de edad es llevada a la clínica de medicina familiar por su esposo. Apenas responde a las preguntas, mantiene inclinada la cabeza y mira al piso. El esposo proporciona la mayor parte de sus datos y niega antecedentes personales patológicos o familiares de enfermedades mentales, pero informa que desde hace varios meses su esposa ha estado cada vez más deprimida y retraída. No participa en sus pasatiempos habituales, sino que permanece acostada en su hogar. “Da vueltas en la cama toda la noche”. Su esposo asegura que come pequeñas cantidades de alimento pero ha perdido una cantidad importante de peso. Tiene ideas recurrentes de sentimientos de culpa con respecto a numerosos problemas y en fechas recientes ha hablado de suicidio, aunque no lo ha planificado o intentado. Ha rechazado acudir al médico; el marido insistió en llevarla a consulta este día porque ella le dijo que está poseída por el demonio y le comentó que ella “se irá al infierno”. 5. ¿Cuál es el diagnóstico más probable? A) Trastorno bipolar, depresión con características psicóticas. B) Delirio de tipo somático. C) Depresión mayor con características psicóticas. D) Trastorno esquizoafectivo de tipo depresivo. E) Esquizofrenia de tipo paranoide. 6. ¿Cuál es el tratamiento farmacológico más eficaz para esta paciente? A) B) C) D)

Antidepresivos solos. Antidepresivos y antipsicóticos. Antipsicóticos solos. Únicamente estabilizadores del estado de ánimo. E) Estabilizadores del estado de ánimo y antipsicóticos.

Preguntas 7 y 8 Una mujer de 16 años de edad es llevada a la clínica de medicina familiar para su examen anual. Su talla se encuentra en el promedio y su peso se encuentra por arriba del mismo. Cuando se le comenta esto, ella se sonroja y rápidamente comenta que está tratando de perder peso. Cuando se le interroga respecto a sus

hábitos dietéticos, admite que consume grandes cantidades de alimentos en una comida, por ejemplo dos pizzas, un emparedado grande y un galón (3.8 L) de helado. También informa que con frecuencia se induce vómito a fin de compensar la comida excesiva, pero niega el uso de laxantes. La paciente está consciente que su conducta no es saludable pero siente que “esta fuera de control”. 7. Los estudios de laboratorio con mayor probabilidad demostrarán uno de los siguientes trastornos. A) B) C) D) E)

Acidosis. Hipercloremia. Hipernatremia. Hipopotasiemia. Leucopenia.

8. Después de comentar el caso con los padres, ellos deciden que se inicie tratamiento psicotrópico y el envío a un programa para tratamiento de trastornos de la alimentación. ¿Qué clase de farmacoterapia será más eficaz en esta paciente? A) B) C) D) E)

Anticonvulsivantes. Antipsicóticos. Benzodiazepinas. Estabilizadores del estado de ánimo. Inhibidores selectivos de la recaptación de serotonina.

Preguntas 9 y 10 Un niño de cuatro años de edad es llevado a la sala de urgencias por su madre para valoración. Cuando se interroga al niño con respecto a síntomas específicos, él parece ansioso y comenta “me duele cuando voy a orinar”. Su madre en secreto añade que “él tiene otra infección urinaria”. La madre enumera los antibióticos que se le han indicado en el pasado y exige que sea hospitalizado para estudios. En la exploración los signos vitales son normales, excepto por temperatura de 38.8ºC. En la exploración física llama la atención el dolor a la palpación en la región suprapúbica y evidencia de traumatismo uretral reciente. El análisis de orina es compatible con infección de vías urinarias. La revisión del expediente clínico muestra múltiples consultas a la sala de urgencias por diversos síntomas físicos, lo que incluye cuadros clínicos similares de infección de vías urinarias. Las valoraciones previas, ya sean como

http://bookmedico.blogspot.com

179

http://bookmedico.blogspot.com Preguntas: 5–13

paciente ambulatorio u hospitalizado no han permitido establecer con precisión la causa de la enfermedad. 9. ¿Cuál es el paso más apropiado en el tratamiento de este paciente? A) Hospitalización del niño y notificación a los servicios de protección al menor. B) Interrogar a la madre con respecto a las sospechas del médico. C) Solicitar la valoración de un psiquiatra para que hable con la madre. D) Valoración del paciente por el urólogo. E) Tratamiento para la infección de vías urinarias y enviarlo al hogar. 10. ¿Cuál es la explicación más probable para la conducta de la madre? A) La producción consciente de los síntomas para que el niño asuma el rol de enfermo. B) La producción consciente de los síntomas para obtener una ganancia secundaria. C) Es la reacción esperada en un padre preocupado. D) Reacción histérica por una preocupación exagerada de los padres. E) Producción inconsciente de los síntomas por conflicto inconsciente.

Preguntas 11 y 12 Una mujer de 19 años de edad, que se casó en fechas recientes, acude a la sala de urgencias acompañada de su esposo. Comenta que se despertó por la mañana y encontró que no podía mover las piernas. Niega dolor pero refiere que es incapaz de percibir estímulos por debajo de su abdomen. Niega antecedentes traumáticos o enfermedades. Tiene embarazo de 24 semanas que ha sido por lo demás normal; sólo toma vitaminas prenatales. Pregunta si los síntomas mejorarán y desea saber si “el bebé está haciendo presión sobre la médula espinal”. En la exploración neurológica llama la atención la fuerza muscular 0/5 en ambas extremidades inferiores, con disminución de la sensibilidad al tacto ligero y a la punción por debajo del nivel de la cicatriz umbilical. Los pares craneales y los reflejos osteotendinosos son normales y no muestra datos de síndrome de neurona motora superior. Se realiza una resonancia magnética nuclear que es reportada como normal.

181

11. ¿Cuál es la explicación más probable para los síntomas de la paciente? A) Producción consciente de los síntomas para asumir el papel de enferma. B) Producción consciente de los síntomas para obtener ganancia secundaria. C) Enfermedad que afecta el sistema nervioso central. D) Enfermedad que afecta el sistema nervioso periférico. E) Producción inconsciente de los síntomas por un conflicto inconsciente. 12. ¿Cuál es el método más apropiado para tratar a esta paciente? A) Administración de líquidos por vía intravenosa, e informarle que los síntomas desaparecerán. B) Hospitalización en el servicio de neurología para realización de pruebas adicionales. C) Interrogar a la paciente con respecto a la naturaleza de los síntomas. D) Valoración por un psiquiatra en la sala de urgencias. E) Tranquilizar a la paciente e informarle que los síntomas mejorarán. 13. Una mujer de 30 años de edad con antecedente de depresión acude a una cita para valoración del puerperio después del nacimiento de su primer hijo. Se encuentra asintomática y los exámenes de laboratorio no muestran problema de importancia. Parece ansiosa y cuando se le interroga describe que tiene pensamientos recurrentes de lastimar a su hijo, pero comenta con rapidez “yo no soy así, yo jamás haría nada que pudiera lastimarlo”. ¿Cuál es el paso más apropiado en el tratamiento de esta paciente? A) Valoración adicional en busca de síntomas de psicosis y buscar un sistema de apoyo. B) Tratamiento inmediato con antidepresivos. C) Llamar a los servicios de protección al menor para solicitar el retiro del niño. D) Hospitalización de la mujer de inmediato para valoración adicional. E) Tranquilización de la paciente e informarle que estos pensamientos son normales.

http://bookmedico.blogspot.com

http://bookmedico.blogspot.com 182

5: Psiquiatría

Preguntas 14 y 15 Una mujer de 80 años de edad es hospitalizada para tratamiento de una infección de vías urinarias. Durante la hospitalización se valora por confusión. En el examen del estado mental (MSE) se observa somnolienta en ocasiones, con estado de alerta fluctuante. No coopera, es agresiva y en ocasiones tiene alucinaciones. Tiene trastornos en la autocrítica y memoria. El diagnóstico diferencial incluye delirio y demencia. 14. ¿Cuál de los siguientes signos y síntomas son más específicos de delirio? A) B) C) D) E)

Agresividad. Nivel de conciencia fluctuante. Trastornos de la memoria. Psicosis. Falta de cooperación.

15. ¿Cuál es el tratamiento farmacológico más apropiado para el tratamiento de la conducta de la paciente? A) B) C) D) E)

Difenhidramina (Benadryl) en dosis bajas. Donepezilo (Aricepet) en dosis bajas. Haloperidol (Haldol) en dosis bajas. Lorazepam (Ativan) en dosis bajas. Risperidona (Risperdal) en dosis bajas.

Preguntas 16 y 17 Una mujer caucásica, joven (se desconoce su edad) es llevada a la sala de urgencias después de que se encontró inconsciente en la estación de autobuses. Tiene obnubilación y los signos vitales son temperatura de 36.5ºC, presión arterial (TA) de 94/60 mmHg, frecuencia cardíaca de 55 lpm y respiraciones de ocho por minuto. En la exploración física se observa a una mujer extremadamente delgada y mal arreglada. Está pálida, con piel y mucosas frías y secas. No coopera a la exploración física. Tiene pupilas puntiformes y con poca respuesta a la luz. La exploración física muestra bradicardia sin soplos o frotes pericárdicos. Los campos pulmonares están bien ventilados, con respiración superficial. Hay distensión abdominal leve.

16. ¿El consumo de cuál de las siguientes sustancias con mayor probabilidad explica las manifestaciones clínicas? A) Alcohol. B) Anticolinérgicos. C) Benzodiazepinas. D) Heroína. E) Fenciclidina (PCP) 17. El siguiente paso en el tratamiento de esta paciente consiste en la administración de: A) B) C) D) E)

Disulfiram (Antabuse). Flumazenil (Romazicon). Naloxona (Narcan). Fisostigmina. Tiamina.

Preguntas 18 y 19 Un varón de 30 años de edad, casado, con antecedentes de depresión acude a la unidad de medicina familiar. Parece avergonzado y un poco ansioso durante la consulta. Niega tristeza significativa o crisis de llanto. Duerme y come bien, sin cambios recientes en el peso corporal. Su energía y concentración son normales y niega ideación suicida u homicida. Refiere que ha seguido su tratamiento con citalopram (Celexa) que ha tomado para depresión, pero “tiene problemas durante las relaciones sexuales”. 18. ¿Cuál de los siguientes síntomas presenta con mayor probabilidad? A) B) C) D) E)

Disminución de la libido. Dolor al coito. Eyaculación prematura. Priapismo. Eyaculación retrógrada.

19. Se decide cambiar el antidepresivo que recibe el paciente para reducir los efectos secundarios. ¿Cuál es el medicamento más apropiado a elegir? A) B) C) D) E)

Bupropión (Wellbutrin). Desipramina (Norpramin). Fluoxetina (Prozac). Fenelzina (Nardil). Venlafaxina (Effexor).

http://bookmedico.blogspot.com

http://bookmedico.blogspot.com Preguntas: 14–24

Preguntas 20 y 21 Una mujer de 86 años de edad es llevada a la sala de urgencias por su hija. La paciente no proporciona bien sus antecedentes y tiene autocrítica limitada. La hija sabe que la paciente tiene antecedentes de hipertensión y recibe tratamiento con f{rmacos no especificados. La paciente vive en una comunidad de retiro. La hija está preocupada desde hace un año cuando observó que su madre parecía más confusa, lo cual atribuyó a la “edad avanzada”, pero hace dos semanas notó empeoramiento súbito del trastorno. Su madre tiene dificultad para reconocer a los familiares cercanos y para recordar información. En las últimas dos semanas se ha extraviado, ha dejado la estufa prendida y ha sido incapaz de bañarse ella misma. A la hija le preocupa que la paciente pueda lesionarse ella misma. 20. ¿Cuáles son los datos que con mayor probabilidad debe mostrar la resonancia magnética nuclear del encéfalo? A) B) C) D) E)

Atrofia del núcleo caudado. Dilatación de los ventrículos sin atrofia. Atrofia frontotemporal. Atrofia generalizada. Infartos de la sustancia blanca.

21. ¿Cuál es la evolución más probable de la enfermedad? A) Mejoría gradual. B) Deterioro rápido. C) Evolución estable. D) Empeoramiento estable. E) Deterioro progresivo.

183

encuentran en límites normales. En la exploración física llama la atención el temblor evidente de sus manos, más intenso en la mano izquierda que en la derecha. La marcha es lenta y de base amplia. 22. ¿Cuál es la estructura cerebral que con mayor probabilidad está afectada en este paciente? A) B) C) D) E)

Núcleo del rafe caudal. Hipocampo. Locus ceruleus. Núcleo basal de Meynert. Sustancia negra.

23. Poco después de iniciar el tratamiento con el medicamento apropiado, se torna agitado y sufre alucinaciones. ¿Cuál es el medicamento más apropiado para estos nuevos síntomas? A) B) C) D) E)

Clozapina (Clozaril). Haloperidol (Haldol). Risperidona (Risperdal). Quetiapina (Seroquel). Tioridazina (Mellaril).

Preguntas 24 y 25 Una mujer de 24 años de edad, estudiante de medicina es valorada en un centro de salud estudiantil. Acude porque iniciará la rotación en cirugía y tiene antecedentes de ansiedad extrema cuando observa sangre o agujas. 24. ¿Cuál es la respuesta hemodinámica que con mayor probabilidad ocurrirá cuando la paciente participe a futuro en procedimientos quirúrgicos?

Preguntas 22 y 23 Un varón de 67 años de edad es valorado en una clínica en una consulta programada. Refiere dificultades con la marcha, la cual ha empeorado en forma progresiva desde hace varios meses. Ha notado “temblor” de las manos lo que ocasiona que se le hayan caído objetos en forma ocasional. Se encuentra muy agobiado por sus problemas y admite que con frecuencia llora. Recibe medicamentos en forma crónica para el tratamiento del reflujo gastroesofágico e hiperlipidemia. A la fecha recibe un inhibidor de la bomba de protones y un fármaco hipocolesterolemiante. En su MSE resalta poca expresión o rango afectivo. Sus signos vitales se

A) Bradicardia inicial seguida de hipotensión. B) Bradicardia inicial seguida de taquicardia e hipertensión. C) Bradicardia inicial seguida de taquicardia e hipotensión. D) Taquicardia inicial seguida de bradicardia e hipotensión. E) Taquicardia inicial seguida por hipertensión.

http://bookmedico.blogspot.com

http://bookmedico.blogspot.com 184

5: Psiquiatría

25. ¿Cuál de las siguientes modalidades de tratamiento es más eficaz para esta mujer? Bloqueadores β. Tratamiento de exposición. Tratamiento orientado a la autocrítica. Inhibidores selectivos de la recaptación de serotonina. E) Terapia de apoyo.

A) B) C) D)

26. Un varón de 48 años de edad sin antecedentes psiquiátricos es valorado en un servicio de urgencias por preocupaciones con respecto a enfermedades de transmisión sexual. Niega disuria, secreción o lesiones en el pene. La exploración física es normal. Cuando se le proporciona esta información insiste en que se le realicen pruebas. Cuando se le interroga respecto a sus antecedentes sexuales responde que tiene una relación monógama con su esposa, que es la Senadora Hilary Clinton. Cuando se le confronta con el hecho de que ella ya está casada con alguien más y que vive en otro estado, responde que se casó con ella hace dos años “en una ceremonia secreta”. Añade que vuela los fines de semana para “visitas conyugales” pero teme que ella le sea infiel y que le haya transmitido una enfermedad venérea. No tiene antecedentes patológicos de importancia y no toma medicamentos a la fecha. El interrogatorio revela que tiene un trabajo estable como guardia de seguridad. Vive sólo en un departamento y niega el consumo de alcohol o drogas ilegales. En el MSE está bien vestido y aliñado. Es cooperador y su estado de ánimo y afecto muestran ansiedad. Sus pensamientos son lógicos, niega ideación suicida u homicida o trastornos de la percepción. ¿Cuál es el diagnóstico más probable? A) B) C) D) E)

Trastorno bipolar, maníaco. Ideas delirantes. Personalidad paranoide. Trastorno esquizoafectivo. Esquizofrenia.

que conoce las reglas del colegio. Nunca pone atención en la escuela o en el hogar cuando se le reprende. Por ejemplo, tiene problemas para obedecer órdenes, realizar sus deberes escolares y recoger su ropa sin importar las consecuencias. En tiendas departamentales, el niño corre por los pasillos, toma objetos y en varias ocasiones ha ocasionado deterioro de la mercancía. El padre comenta que su hijo se ha comportado de esta forma “desde que puede caminar” y está preocupado con respecto al futuro de su hijo. 27. ¿Cuál de los siguientes neurotransmisores con mayor probabilidad está implicado en la causa y tratamiento de este trastorno? A) B) C) D) E)

Acetilcolina. Ácido aminobutírico gamma (GABA) Glutamato. Noradrenalina. Serotonina.

28. Se amplían el interrogatorio y la exploración física y se obtienen los registros escolares. Se indica el inicio de un tratamiento de primera línea para los síntomas. El padre tiene dudas con respecto a este medicamento en particular. ¿Cuál de las siguientes aseveraciones es la más precisa con respecto a la farmacoterapia para este trastorno? A) Está absolutamente contraindicada en pacientes con tics concomitantes. B) El efecto terapéutico implica una sedación paradójica. C) Podría causar beneficios similares si lo toma un niño normal. D) Causará supresión del crecimiento si se utiliza durante la edad escolar. E) Incrementa el riesgo a futuro de farmacodependencia.

Preguntas 29 a 31

Preguntas 27 y 28 Un niño de siete años de edad es llevado a consulta por su padre. Está preocupado por la conducta de su hijo, pues le han comentado que en la escuela no permanece quieto y que se desplaza por todo el salón a pesar de

Una niña de nueve años de edad es llevada a la sala de urgencias por sus padres. En los últimos 18 meses han notado “h{bitos recientes”, lo que incluye mirar en forma repetida con los ojos entornados y gesticulaciones junto con gruñidos y limpieza frecuente de la garganta. Estas conductas aparecen casi todo el día y con frecuencia ocurren en conjunto. Ha sido objeto de burlas cada vez más frecuentes y su ansiedad empeora

http://bookmedico.blogspot.com

http://bookmedico.blogspot.com Preguntas: 25–34

los síntomas. No tiene otras enfermedades y no recibe medicamentos. La exploración física se encuentra en límites normales con excepción de las conductas estereotípicas mencionadas antes. 29. ¿Cuál de los siguientes diagnósticos revelará con mayor probabilidad el interrogatorio adicional? A) B) C) D) E)

Trastorno autista. Depresión mayor. Trastorno obsesivo-compulsivo. Trastorno de pánico. Trastorno de la conducta.

30. ¿Cuál es la farmacoterapia más eficaz para esta enfermedad? A) B) C) D) E)

Clonidina. Haloperidol (Haldol). Lorazepam (Ativan). Metilfenidato (Ritalin). Paroxetina (Paxil).

32. ¿Cuál es el tratamiento farmacológico más apropiado para este trastorno? A) B) C) D) E)

Alprazolam (Xanax). Bupropión (Wellbutrin). Citalopram (Celexa). Desipramina (Norpramin). Olanzapina (Zyprexa).

33. El paciente no desea tomar medicamentos pero está interesado en la psicoterapia. ¿Cuál de los siguientes es el tratamiento más eficaz para reducir los síntomas? A) Terapia conductual. B) Desensibilización con movimientos oculares y reprocesamiento (EMDR). C) Psicoanálisis. D) Psicoterapia psicodinámica. E) Terapia de apoyo.

Preguntas 34 y 35

31. En esta paciente existe una alta probabilidad de infección con uno de los siguientes microorganismos: A) B) C) D) E)

185

Virus del herpes simple. VIH. Virus de la influenza. Staphylococcus. Streptococcus.

Preguntas 32 y 33 Un varón de 26 años de edad, estudiante graduado, acude a una organización para la conservación de la salud. Ha tenido dificultades para completar su tesis y cuando trabaja en la computadora siente la necesidad de imprimir para resguardar cada cuartilla que escribe. Pese a que se da cuenta que es innecesario realizar esto, se siente obligado a leer y releer todas las versiones en busca de un error. Como consecuencia, es incapaz de avanzar en su trabajo. Tiene múltiples dudas con respecto a la tesis, al mismo tiempo que no puede deshacerse del material inútil. De hecho, en su departamento se encuentran cajas de papel por toda la habitación. Comprende que estas conductas y pensamientos son “irracionales” y siente una gran angustia por ellos y por los problemas que le han causado.

Un varón casado, de 29 años de edad es valorado en la sala de urgencias porque refiere “tengo temor de sufrir un ataque cardíaco”. Comenta que los últimos dos meses ha experimentado episodios recurrentes de dolor torácico y disnea con duración de 10 a 20 min. También describe como síntomas asociados taquipnea, mareo, parestesias en las extremidades, náuseas, diaforesis, ansiedad y temor de fallecer. Estos síntomas ocurren casi todo el día, pero no son provocados por situaciones o actividades como ejercicio o esfuerzo. Tiene una preocupación significativa con respecto a episodios futuros y una preocupación genuina con respecto a sufrir infarto miocárdico. Niega la presencia de enfermedades o el consumo de medicamentos. Bebe tres cervezas a la semana y no usa drogas ilegales. La exploración física muestra aumento leve de la presión arterial y de la frecuencia cardíaca y en el electrocardiograma se observa taquicardia sinusal. 34. ¿Cuál de los siguientes medicamentos es el más apropiado para el tratamiento agudo de los síntomas de este paciente? A) B) C) D) E)

Imipramina (Tofranil). Lorazepam (Ativan). Paroxetina (Paxil). Risperidona (Risperdal). Ácido valproico (Depakene).

http://bookmedico.blogspot.com

http://bookmedico.blogspot.com 186

5: Psiquiatría

35. ¿Cuál de los siguientes medicamentos es el más apropiado para el tratamiento a largo plazo de los síntomas de este paciente? A) B) C) D) E)

Imipramina. Lorazepam. Paroxetina. Risperidona. Ácido valproico.

hablar. Su familia ha notado que parece deprimido y a menudo llora. Se realizó MSE al paciente para ayudar a establecer si sufre demencia o depresión (seudodemencia). 37. ¿Cuáles son las características del MSE que son más compatibles con seudodemencia?

36. Una abogada de 55 años de edad sin antecedentes psiquiátricos acude con su médico internista por insomnio. Desde que su esposo falleció hace cinco semanas ha tenido dificultades para dormir, con despertares frecuentes durante la noche. Durante el día se encuentra cansada. Cuando se le interroga con respecto a su estado de ánimo, comenta que está “triste” y que a menudo tiene ataques de llanto cuando piensa en su esposo. La paciente cree que su trabajo ocupa su mente, pero comenta que se distrae con facilidad y comete errores menores en el trabajo. Su apetito ha disminuido pero su peso no ha cambiado. Se siente “perdida” y no disfruta de su vida sin su esposo; niega ideación suicida. Con dificultades admite que oye la voz de su esposo llamándola durante la noche. Comprende que no es real pero se siente cómoda con esta sensación. ¿Cuál es el paso más apropiado en el tratamiento de esta paciente? A) Hospitalización para tratamiento y valoración adicionales. B) Tratamiento con antidepresivos. C) Tratamiento con antidepresivos y antipsicóticos. D) Vigilancia de los síntomas en las siguientes semanas. E) Valoración por el psiquiatra para administración de tratamiento farmacológico.

Preguntas 37 y 38 Un varón de 70 años de edad es llevado a la clínica de medicina familiar porque su familia sospecha que tiene enfermedad de Alzheimer. Han notado empeoramiento de la memoria en los últimos seis meses. No desea salir de la cama y tiene dificultades para atender sus necesidades básicas como limpieza, vestido y cocinar para sí mismo. Está renuente a hablar, pero no está claro si es incapaz o no se encuentra motivado para

A) B) C) D) E)

Preocupación durante el examen. Atiende mal a las preguntas del MSE. Muestra poca conciencia de sus síntomas. Responde “no sé” a todas las preguntas. Realiza mal sus actividades.

38. Se amplían el interrogatorio y la exploración física, se realiza examen cognitivo y estudios de laboratorio y radiográficos. Los resultados son compatibles con demencia de Alzheimer. Al inicio la familia podía cuidarlo, pero lo regresaron a la clínica porque no puede permanecer en el hogar. La familia percibe que el paciente está mucho más agitado, que permanece despierto por la noche y desplaza los muebles buscando “las personitas que me molestan”. Los familiares han notado dificultades para la marcha, a menudo se mueve con lentitud y deja caer objetos de las manos. La familia ha intentado colocarlo en un asilo pero desean que se le administre algún medicamento para ayudarlo a dormir y mantenerlo calmado ¿Cuál de los siguientes medicamentos debe evitarse en este paciente? A) B) C) D) E)

Buspirona (Buspar). Donepezilo (Arizep). Lorazepam (Ativan). Trazodona (Desirel). Risperidona (Risperdal).

Preguntas 39 y 40 Un niño de 12 años de edad es llevado a consulta por su madre quien comenta “no puedo seguir lidiando con él”. Parece exasperada y comenta que su hijo se ha metido en más problemas en los últimos 15 meses desde la conclusión de un divorcio largo y particularmente difícil. Abandona el hogar por las noches sin notificar a su madre o decirle adonde va. Sospecha que ha participado en el aumento del vandalismo en el vecindario. En fechas recientes fue sorprendido robando en una tienda cercana. Sus calificaciones siempre han sido malas, pero ha sido suspendido por faltar a clases en el último

http://bookmedico.blogspot.com

http://bookmedico.blogspot.com Preguntas: 35–42

año. Con frecuencia hay datos de que ha participado en peleas. La madre sospecha que el niño es miembro de una pandilla y teme que termine en la cárcel y “se vuelva igual que su padre”. 39. ¿Cuál de los siguientes diagnósticos premórbidos podrían encontrarse en este paciente? A) Trastorno antisocial de la personalidad. B) Trastorno de hiperactividad con déficit de atención. C) Autismo. D) Esquizofrenia infantil. E) Retraso mental. 40. Si no se trata, ¿cuál es el diagnóstico que podría tener este paciente en el futuro? A) B) C) D) E)

Alcoholismo. Trastorno de oposición desafiante. Trastorno de pánico. Personalidad esquizoide. Esquizofrenia.

187

41. ¿Cuál de los siguientes es el principal factor de riesgo para el desarrollo de enfermedad crónica en este paciente? A) Antecedente de traumatismo en la infancia. B) Ausencia de trastornos psiquiátricos concomitantes. C) Género masculino. D) No hubo disociación durante el evento traumático. E) Rasgos esquizoides antes del evento traumático. 42. ¿Cuál de los siguientes medicamentos es útil como monoterapia para el tratamiento eficaz de estos síntomas? A) B) C) D) E)

Amobarbital (Amital). Haloperidol (Haldol). Lorazepam (Ativan). Sertralina (Zoloft). Trazodona (Desyrel).

Preguntas 41 y 42 Un varón de 19 años de edad, veterano de las fuerzas armadas estadounidenses, acude a una clínica de atención ambulatoria. En fechas recientes regreso del combate en Irak donde se le asignó a la infantería. Mientras patrullaba hace un mes, fue testigo de la muerte de varios de sus compañeros por una bomba colocada al lado del camino. Desde ese momento ha tenido dificultades para dormir con despertares frecuentes después de pesadillas recurrentes con respecto al evento. El paciente ha notado que en ocasiones “brinca”, en especial cuando percibe ruidos fuertes. Vive en la casa de sus padres desde las fiestas del 4 de julio, cuando tuvo un episodio de ansiedad aguda después de oír los fuegos artificiales. No pasa tiempo con amigos o familiares. Rechaza ver cualquier programa de televisión u oír la radio por el temor a escuchar noticias de más muertes. Refiere “insensibilidad” y se distrae con facilidad. Niega ideación suicida, pero en ocasiones siente que “mi vida terminó en Irak”.

http://bookmedico.blogspot.com

http://bookmedico.blogspot.com 188

5: Psiquiatría

43. Un varón de 18 meses de edad es llevado a la sala de urgencias por su madre quien refiere que “estaba comiendo suciedad”. Desde hace unos cuantos meses el niño empezó a caminar y se le ha encontrado tragando y deglutiendo sustancias extrañas como cabello, papel, pintura y cuerdas. La madre está preocupada porque en fechas recientes notó que come tierra y barro que se encuentra en los cimientos del departamento que está en construcción. El apetito del niño se ha visto afectado y a la madre le preocupa que pueda enfermarse a futuro. ¿Cuál de las siguientes sustancias podría estar disminuida en la sangre de este niño? A) B) C) D) E)

Ácido fólico y plomo. Ácido fólico y zinc. Hierro y plomo. Hierro y zinc. Plomo y zinc.

44. Una niña de 14 meses de edad es llevada a la clínica de medicina familiar por sus padres. Era una niña sana, los padres comentan que solía ser “extrovertida”, pero en fechas recientes se ha vuelto tímida y responde poco. Desde que empezó a caminar ha sufrido múltiples caídas y en fechas recientes es incapaz de mantenerse de pie. La madre notó que aplaude con las manos y que los sombreros para el sol se han vuelto demasiado grandes para ella. ¿Cuál es el diagnóstico más probable para esta paciente? A) B) C) D) E)

Trastorno de Asperger. Trastorno autista. Trastorno de desintegración infantil. Síndrome de cromosoma X frágil. Enfermedad de Rett.

Preguntas 45 y 46 Un varón de 60 años de edad con antecedente de esquizofrenia crónica y múltiples hospitalizaciones acude a la sala de urgencias por “movimientos raros”. Ha tomado en forma adecuada su tratamiento con risperidona (Risperdal), desde hace seis años, 3 mg cada 12 h. El paciente tiene sobrepeso, pero la higiene es adecuada. Sus pensamientos son un poco tangenciales, pero no hay desorganización mental evidente. Niega datos de paranoia, ideas de referencia o delirio, trastornos de la

percepción o ideación suicida/homicida. La exploración física es normal excepto por parpadeo y gesticulación involuntarios ocasionales, así como la rotación del tobillo izquierdo. Tiene una gran angustia con respecto a estos “h{bitos” y desea saber si puede hacer algo con respecto a ellos. 45. ¿Cuál de las siguientes medidas sería la más apropiada para el tratamiento de este paciente? A) Añadir benztropina a la risperidona. B) Continuación de la dosis actual de risperidona. C) Disminución de la dosis de risperidona. D) Interrupción del consumo de risperidona. E) Incrementar la dosis de risperidona. 46. El mismo paciente es llevado nuevamente a la sala de urgencias en ambulancia un mes después por “catatonía”. De acuerdo a su expediente clínico, ha recibido la misma dosis de risperidona por indicación de su psiquiatra, quien lo valoró en la consulta. En la exploración no responde a las preguntas. Sus signos vitales muestran temperatura de 39.7°C, TA de 180/95 mmHg, pulso de 105 lpm y respiraciones de 20 por minuto. En la exploración física los datos de importancia incluyen diaforesis significativa, rigidez muscular y falta de cooperación durante la exploración. ¿Cuál es el tratamiento más apropiado para este paciente? A) Añadir benztropina (Cogentin) a la risperidona. B) Continuación de la dosis actual de risperidona. C) Disminución de la dosis de risperidona. D) Interrupción del consumo de risperidona. E) Incremento de la dosis de risperidona. 47. Un varón de 22 años de edad es llevado a la sala de urgencias por la policía porque lo encontraron desnudo y gritando a mitad de la calle. En un cuarto silencioso tiene conducta impredecible durante la exploración. Tiene labilidad emocional extrema, que alterna entre la agitación, patea la cama y no pone atención. Al parecer responde a estímulos externos y tiene una conducta paranoide. La exploración física limitada

http://bookmedico.blogspot.com

http://bookmedico.blogspot.com Preguntas: 43–51

muestra elevación leve de la presión arterial y de la frecuencia cardíaca, nistagmo, ataxia y rigidez muscular. ¿Cuál es la intoxicación que con mayor probabilidad presenta este paciente? A) B) C) D) E)

Alcohol. Marihuana. Heroína. Dietilamida del ácido lisérgico (LSD). Fenciclidina (PCP).

Preguntas 48 y 49 Una viuda de 68 años de edad acude a la clínica de medicina familiar para una cita programada. Sus antecedentes personales patológicos de importancia incluyen hipertensión, obesidad y neumopatía obstructiva crónica. No tiene antecedentes de enfermedades psiquiátricas pero acudió a ocho sesiones con el psicólogo después de la muerte de su marido. No consume alcohol ni utiliza drogas ilegales. Fuma 30 cigarrillos por día desde hace 45 años y desea interrumpir el tabaquismo. Ha escuchado con respecto a algunas opciones pero no est{ segura cu{l es la m{s eficaz. 48. ¿Cuál de las siguientes estrategias tendrá más éxito para ayudar a la paciente a interrumpir el tabaquismo? A) B) C) D) E)

Interrupción súbita. Tratamiento conductual. Educación. Medicamentos para sustitución de nicotina. Medicamentos y terapia grupal.

49. Después de informarle de varias opciones, la mujer decide tomar medicamentos. ¿Cuál de los siguientes medicamentos es de mayor utilidad para la interrupción del consumo de tabaco? A) B) C) D) E)

Bupropión (Wellbutrin). Fluoxetina (Prozac). Mirtazapina (Remeron). Trazodona (Desyrel). Venlafaxina (Effexor).

189

Preguntas 50 y 51 Una mujer de 32 años de edad acude a consulta externa por cefaleas continuas. Desde hace ocho meses ha sufrido cefaleas recurrentes que describe como bilaterales, occipitales con una sensación de opresión que dura varias horas y se alivia con fármacos antiinflamatorios no esteroideos. El interrogatorio revela sensación de fatiga crónica y mala concentración. Admite que con frecuencia está preocupada con respecto a su desempeño laboral y a aspectos que incluyen su relación de pareja con su novio. De hecho estas preocupaciones interfieren con el sueño. Como consecuencia, a menudo presenta despertares por la noche con pánico extremo, temblores, diaforesis, náuseas y palpitaciones. Sus antecedentes personales patológicos incluyen reflujo gastroesofágico para el cual recibe tratamiento con famotidina. En ocasiones bebe un vaso de vino y niega el consumo de drogas. 50. ¿Cuál es el diagnóstico más probable? A) B) C) D) E)

Trastorno de ansiedad generalizada. Depresión mayor. Trastorno obsesivo-compulsivo. Trastorno de pánico Fobia social.

51. Una vez que se ha establecido el diagnóstico se valora el inicio del tratamiento con buspirona (Buspar), una benzodiazepina o ambos. ¿Cuál de las siguientes aseveraciones es más precisa con respecto al uso de buspirona en comparación con benzodiazepinas? A) Los pacientes que toman buspirona pueden desarrollar tolerancia y supresión con el uso a largo plazo. B) La buspirona es más eficaz para reducir los síntomas cognitivos de ansiedad. C) La buspirona a menudo es eficaz después de que se ha intentando el tratamiento con benzodiazepinas. D) El tiempo antes del inicio de la acción es aproximadamente el mismo para la buspirona y las benzodiazepinas. E) La buspirona debe utilizarse sólo como monoterapia.

http://bookmedico.blogspot.com

http://bookmedico.blogspot.com 190

5: Psiquiatría

Preguntas 52 y 53 Un varón de 18 años de edad es llevado a la sala de urgencias temprano por la mañana por sus amigos después de asistir a una fiesta. Está agitado y camina a mitad del pasillo, pero la marcha es inestable. A pesar de esto, el sujeto comenta que se siente “de maravilla” y que “todo se encuentra bien”. También parece que pone atención a imágenes multicolores y que percibe “conversaciones de varias personas a la vez”. No tiene antecedentes personales patológicos de importancia y no toma medicamentos. Admite que consumió una sustancia temprano por la noche y comenta que esa sustancia “le ayudaría a tolerar la fiesta toda la noche”. En la exploración física tiene hipertensión arterial y aumento de la frecuencia cardíaca, midriasis y diaforesis significativa. 52. ¿Cuál es el efecto farmacológico más probable de la sustancia que el paciente consumió? A) Bloqueo de la recaptación de dopamina. B) Bloqueo de los receptores de glutamato. C) Aumento de la actividad de los receptores de serotonina. D) Liberación de dopamina. E) Liberación de dopamina y serotonina. 53. Finalmente, el mismo paciente es admitido para destoxificación y completa con éxito un programa para el tratamiento de la farmacodependencia. El paciente acude a un colegio y tiene buen desempeño escolar. Más tarde regresa a la sala de urgencias con síntomas recurrentes similares a los que presentó cuando estaba “elevado”, como fosfenos, intensificación de los ruidos y halos, lo cual le causa gran molestia y considera que éstos interfieren con el estudio. La exploración física completa y los estudios de laboratorio (lo que incluye el estudio toxicológico) son normales. ¿Cuál de los siguientes medicamentos debe evitarse en este paciente? A) B) C) D) E)

Carbamazepina (Tegretol). Clonazepam (Klonopin). Fluoxetina (Prozac). Haloperidol (Haldol). Ácido valproico (Depakene).

Preguntas 54 y 55 Un veterano de la guerra de Vietnam, de 58 años de edad, es llevado a la sala de urgencias. Refiere sensa-

ción de depresión e ideación suicida de varios días de evolución. Admite que consume “crack” diariamente desde hace tres semanas, y no proporciona información clara con respecto a la forma en que obtiene la droga ni los esfuerzos que realiza para obtenerla. Bebe varias cervezas de 1.2 L tres o cuatro veces por semana y “en ocasiones” fuma marihuana. No tiene un lugar donde vivir y se queda con sus amigos en refugios. Utilizó cocaína esta mañana y desea ser hospitalizado para destoxificación. 54. ¿Cuál es el diagnóstico de la enfermedad que con mayor probabilidad presenta el paciente en forma simultánea? A) B) C) D) E)

Trastorno antisocial de la personalidad. Trastorno bipolar. Trastorno de ansiedad generalizada. Depresión mayor. Esquizofrenia.

55. Más tarde el sujeto fue hospitalizado en una unidad de cuidados mentales, pero al siguiente día es valorado por síntomas de supresión. Refiere insomnio, apatía, irritabilidad y empeoramiento de la disforia. ¿Cuál es el tratamiento más apropiado para este trastorno? A) Tratamiento con antidepresivos. B) Disminución de la dosis de benzodiazepinas. C) Educación y tranquilización. D) Destoxificación con metadona. E) Destoxificación con fenobarbital.

Preguntas 56 y 57 Una niña de seis años de edad es llevada por sus padres adoptivos a la clínica de medicina familiar para valoración, pues ellos están preocupados con respecto a que “algo pueda estar mal con la niña”. Han notado conductas extrañas con repetición de palabras y frases y dificultad para obedecer órdenes. Sus signos vitales son normales. Los datos de importancia en la exploración física incluyen circunferencia cefálica por arriba del porcentil 90, si bien el peso se encuentra por debajo del porcentil 30, tiene pabellones auriculares grandes y flexibilidad articular excesiva. 56. ¿Cuál de los siguientes cromosomas con mayor probabilidad se encontrará anormal en esta paciente?

http://bookmedico.blogspot.com

http://bookmedico.blogspot.com Preguntas: 52–62

A) B) C) D) E)

5 15 18 21 X

57. ¿Cuál es el diagnóstico concomitante más probable en esta paciente? A) Anorexia nerviosa. B) Trastorno de hiperactividad con déficit de atención. C) Trastorno obsesivo-compulsivo. D) Trastorno de oposición desafiante. E) Síndrome de Tourette.

Preguntas 58 y 59 Un varón de 17 años de edad es llevado contra su voluntad con su médico familiar por la madre, quien está molesta porque el paciente “anda en malas compañías”. La mamá cree que fuma marihuana todos los días después de la escuela y en los fines de semana con sus amigos. El joven se encuentra molesto con respecto a la cita pero niega el consumo de drogas y alcohol. La madre desea que se le informe al muchacho con respecto a los riesgos potenciales del consumo de drogas. 58. ¿Cuál de los siguientes efectos físicos es más compatible con intoxicación por marihuana? A) B) C) D) E)

Disminución de la frecuencia respiratoria. Sialorrea. Hiporexia. Función motora normal. Taquicardia.

59. ¿Cuál es la consecuencia más grave a largo plazo de fumar marihuana? A) B) C) D) E)

Síndrome amotivacional. Atrofia cerebral. Daño cromosómico. Cáncer pulmonar. Convulsiones.

191

de evolución desde que fue despedida de su empleo. Admite que “se siente triste todo el tiempo” y que tiene dificultad para dormir, poca energía, disminución del apetito, “no se siente capaz de disfrutar” y teme que su trastorno nunca mejore. La paciente ha empezado a pensar que “no importa si muere”, pero niega ideación o intentos suicidas. Bebe una o dos bebidas alcohólicas por semana y niega el uso de drogas. Se decide iniciar tratamiento antidepresivo con paroxetina (Paxil) 20 mg por las noches antes de dormir. 60. Si no hay mejoría significativa de los síntomas pero los medicamentos se toleran después de un tiempo, ¿en cuánto tiempo debe valorarse el incremento de la dosis? A) B) C) D) E)

Cuatro días. Una semana. Dos semanas. Cuatro semanas. Siete semanas.

61. ¿Cuál de los siguientes efectos secundarios ocurrirá con mayor probabilidad después de varios meses de tratamiento? A) B) C) D) E)

Cefalea. Inhibición del orgasmo. Evacuaciones disminuidas de consistencia. Náuseas. Sueños vívidos.

Preguntas 62 y 63 Un niño de ocho años de edad es llevado a consulta para valoración, sus padres están preocupados con respecto a la conducta del niño en la escuela; en fechas recientes cada vez se encuentra más renuente para acudir a la escuela. Desde que era pequeño ha disfrutado de la lectura, pero ahora parece que se frustra con facilidad cuando lee o cuando responde preguntas escritas. De hecho, durante dichas ocasiones a menudo interrumpe la clase y esto ha llevado a sus padres a valorar el retirarlo de esa escuela. 62. ¿Cuál de las siguientes pruebas es de mayor utilidad en la valoración de este niño?

Preguntas 60 y 61 Una mujer casada de 38 años de edad acude a la sala de urgencias por “crisis de llanto” de varias semanas

A) Prueba gestáltica visual-motora de Bender. B) Prueba Children Apperception. C) Prueba neuropsicológica de Reitan-Indiana.

http://bookmedico.blogspot.com

http://bookmedico.blogspot.com 192

5: Psiquiatría

D) Pruebas en bloque de Rorschach. E) Escala de inteligencia de Wechsler para niños. 63. ¿Cuál de los siguientes diagnósticos presentará con mayor probabilidad este paciente? A) Trastorno de hiperactividad con déficit de atención. B) Autismo. C) Depresión mayor. D) Retraso mental. E) Trastorno con tics.

Preguntas 64 y 65 Un varón de 52 años de edad acude a la sala de urgencias con dolor intenso en las piernas. El paciente comenta que ha tenido dolor en la rodilla izquierda de seis meses de evolución que no se alivia con los fármacos antiinflamatorios no esteroideos (NSAID). Niega traumatismos, pero refiere tener artritis. Sus signos vitales son estables. La exploración física de la rodilla no muestra datos significativos, excepto por disminución del arco de movimientos pero con poco esfuerzo. No hay aumento de volumen, eritema o datos de traumatismo. Se solicitó una radiografía que se reportó como “normal” sin datos de artritis. Cuando se le ofrece un nuevo ciclo de tratamiento con NSAID y envío con un especialista, el paciente se enfurece y abandona la sala de urgencias. 64. ¿Cuál de las siguientes es la motivación más probable para las manifestaciones clínicas de este paciente? A) Producción consciente de los síntomas para asumir el papel de enfermo. B) Producción consciente de los síntomas para obtener una ganancia secundaria. C) Creencia falsa de que tiene artritis. D) Temor de que sufra una enfermedad grave. E) Producción inconsciente de síntomas por un conflicto inconsciente. 65. ¿Cuál es el tratamiento más apropiado si el paciente regresa? A) Informar al paciente que se cree que su conducta tiene como fin la obtención de drogas. B) Hospitalización en una clínica psiquiátrica.

C) Interrogatorio y valoración adicionales. D) Notificación a la policía. E) Envío con el psiquiatra.

Preguntas 66 y 67 Una mujer de 54 años de edad es valorada en la sala de urgencias por dolor tor{cico con el fin de descartar infarto miocárdico. Describe dolor torácico de inicio súbito en el lado izquierdo del tórax, pungitivo, acompañado de náuseas, diaforesis, palpitaciones y temblor. En la exploración física se le observa desaliñada y ansiosa y con aliento alcohólico. Su presión arterial y frecuencia cardíaca están elevadas. Tiene eritema ocular y rubor facial. Hay un temblor evidente en ambas manos. Cuando se le pregunta cuando bebió por última vez ella responde “ayer, por la noche”. 66. ¿Cuál de las siguientes pruebas de laboratorio debe estar disminuida en esta paciente? A) B) C) D) E)

Aminotransferasa de alanina (ALT). Aminotransferasa de aspartato (AST). Gamma-glutamil transpeptidasa (GGT). Hemoglobina y hematócrito. Triglicéridos séricos.

67. ¿Cuál de los siguientes medicamentos es de mayor utilidad para disminuir el deseo de consumo de alcohol? A) B) C) D) E)

Disulfiram (Antabuse) Fluoxetina (Prozac). Litio. Naltrexona (ReVia). Risperidona (Risperdal).

68. Un varón de 26 años de edad es llevado a la sala de urgencias en ambulancia con mínima respuesta al interrogatorio y a la exploración. La novia informa que tiene antecedentes de trastorno depresivo mayor y alcoholismo. Se encuentra inconsciente con una nota de suicidio y con muchas botellas vacías de cerveza. La novia piensa que ha tomado “otras drogas” que compró con un traficante local. ¿Cuál de las siguientes sustancias encontradas en un examen toxicológico en orina sería la más peligrosa en este paciente? A) Barbitúricos.

http://bookmedico.blogspot.com

http://bookmedico.blogspot.com Preguntas: 63–72

B) C) D) E)

Canabinoides. Cocaína. Opioides. Fenciclidina.

se siente muy distraído y fatigado. Atribuye estos síntomas a que se despierta casi a las tres de la mañana y no puede volver a dormir. Pese a que se siente “peor que nunca” niega ideación suicida. No tiene antecedentes psiquiátricos o de enfermedades concomitantes. Él inició el consumo de mirtazapina (Remeron) 15 mg al ir a la cama por las noches.

Preguntas 69 y 70 Una mujer divorciada de 26 años de edad es llevada a la sala de urgencias después de que se le encontró vagando por las calles. La paciente responde bien al interrogatorio pero da respuestas espontáneas a las preguntas. Comenta que cree que desde hace un año es seguida por “agentes” del Vaticano que la vigilan de cerca para saber “si soy una buena católica”. Mientras dichos agentes la vigilan utilizan señales de radio para decirle “prostituta” y “sucia”. A causa de estas experiencias no puede trabajar. Teme que la asocien con otras personas por el temor de que sea “juzgada”. Niega antecedentes personales patológicos y no toma medicamentos. Sus padres se divorciaron cuando ella era niña. No sabe nada con respecto a su padre pero su madre sufre trastorno bipolar y toma litio. En su MSE se observa lentitud psicomotora, lenguaje pausado y aplanamiento afectivo. 69. ¿Cuál de las características antes mencionadas de esta paciente le confiere un mejor pronóstico? A) B) C) D) E)

193

Edad de inicio. Antecedente de enfermedad psiquiátrica. Estado marital. Datos durante el examen mental. Antecedentes sociales.

70. ¿Cuál es la probabilidad de que lleve una vida moderadamente adecuada desde el punto de vista funcional, con el tratamiento apropiado? A) 0 a 20% B) 20 a 40% C) 40 a 60% D) 60 a 80% E) 80 a 100%

Preguntas 71 y 72

71. ¿Cuál de los siguientes síntomas con mayor probabilidad será el último en mejorar? A) B) C) D) E)

Anergia. Desesperanza. Insomnio. Falta de concentración. Falta de apetito.

72. Su enfermedad fue tratada con éxito y presentó remisión durante casi un año. Regresa a la clínica y desea interrumpir los medicamentos. Si reduce la dosis de mirtazapina, ¿Cuál es la probabilidad de que sufra recurrencia? A) 0 a 10% B) 10 a 30% C) 30 a 50% D) 50 a 80% E) 80 a 100%

Preguntas 73 y 74 Una mujer de 25 años de edad fue dada de alta en fechas recientes del hospital después de su primer episodio maníaco. A la fecha toma 1200 mg de litio por día y sus concentraciones de este elemento son de 1.1 meq/L. Tiene un temblor ligero pero tolerable y ha aumentado 2.5 kg de peso, pero por lo demás tolera bien el medicamento. Comenta que su estado de ánimo ha permanecido “bastante bien”. Duerme casi 7 h por la noche. Su energía y concentración son adecuadas y niega pensamientos veloces, verborrea o incremento de la actividad. No tiene otras enfermedades de importancia y el único medicamento que toma son anticonceptivos orales. No consume alcohol o drogas. La paciente pregunta “¿Cuanto tiempo tomaré estos medicamentos por este problema?”

Un varón de 30 años de edad es valorado en la clínica de medicina familiar. Refiere “tristeza” de tres meses de evolución que empezó poco después de perder su trabajo, hace seis meses. Su apetito ha disminuido y ha notado que sus ropas le quedan grandes. El paciente

http://bookmedico.blogspot.com

http://bookmedico.blogspot.com 194

5: Psiquiatría

73. ¿Cuál es el tiempo recomendado para el tratamiento de mantenimiento de esta paciente? A) Seis meses. B) Un año. C) Dos años. D) Cinco años. E) De por vida.

75. ¿Cuál de los siguientes medicamentos debe evitarse en este paciente?

74. La paciente regresa a la clínica seis meses después. Continúa tomando litio y sus concentraciones continúan en 1.2 meq/L. Comenta que durante las últimas semanas ha presentado gran tristeza. Ahora duerme 10 h por las noches pero aún así se siente cansada. Tiene dificultades para concentrarse en la escuela y come una vez al día. No disfruta mientras participa en sus pasatiempos habituales y siente que “no vale la pena vivir la vida” aunque niega ideación suicida o planificación del suicidio. ¿Cuál es el paso más apropiado en el tratamiento de esta paciente? A) B) C) D) E)

molestos con respecto a su estado y desean que se le prescriba un medicamento para calmarlo y ayudarlo a dormir por las noches.

Añadir lamotrigina (Lamictal). Añadir sertralina (Zoloft). Añadir valproato (Depacon). Disminución de la dosis de litio. Aumento en la dosis de litio.

Preguntas 75 a 77 Un varón retirado de 68 años de edad es acompañado por sus hijos con el médico familiar. Están preocupados con respecto a la salud de su padre porque han notado que gradualmente se encuentra “m{s confuso” desde hace un año. El paciente siempre ha sido capaz de cuidarse solo pero no tiene la capacidad de pagar todas sus cuentas. El paciente explica que necesita sus bifocales pero los hijos lo interrumpen con rapidez y comentan que el paciente tiene anteojos pero que siempre los olvida. De hecho, con frecuencia pierde sus llaves y olvida cerrar la puerta. El conserje del condominio se ha quejado porque en fechas recientes lo encontraron vagando por el lobby y la piscina a media noche mientras vestía ropa interior. No tiene antecedentes personales patológicos de importancia y solamente toma ácido acetilsalicílico diariamente. En su MSE llama la atención su conducta defensiva a preguntas, mostrando cierta irritabilidad. Su examen de estado minimental es de 19-30, con déficit notable de la memoria y dificultad para encontrar las palabras apropiadas. Los hijos están

A) B) C) D) E)

Difenhidramina (Benadryl). Donepezilo (Aricept). Haloperidol (Haldol). Lorazepam (Ativan). Trazodona (Desyrel).

76. ¿Cuáles son los datos que podrían observarse en una resonancia magnética nuclear? A) Atrofia de los lóbulos frontal y temporal. B) Atrofia del núcleo caudado con atrofia cortical. C) Atrofia difusa con dilatación de los ventrículos. D) Dilatación de los ventrículos cerebrales sin atrofia cortical. E) Infartos subcorticales de la sustancia blanca. 77. ¿Cuál es la evolución más probable de la enfermedad? A) B) C) D) E)

Progresión gradual. Sin empeoramiento. Progresión rápida. Mejoría estable. Progresión gradual.

Preguntas 78 a 80 Una mujer de 82 años de edad es hospitalizada después de sufrir una fractura de cadera por una caída en las escaleras. La intervención quirúrgica y recuperación se llevaron a cabo sin problemas, pero tres días después las enfermeras están frustradas porque la paciente no les permite tomar los signos vitales u obtener muestras de sangre. Durante el interrogatorio la paciente parece somnolienta pero con períodos de agitación ocasional. Es incapaz de responder bien a las preguntas y únicamente está orientada en cuanto a persona. Intenta tomar objetos inexistentes en el aire y grita y se retuerce cuando la enfermera está presente.

http://bookmedico.blogspot.com

http://bookmedico.blogspot.com Preguntas: 73–80

78. Si se realiza un electroencefalograma (EEG) a esta paciente, los datos que con mayor probabilidad se observarán son: A) B) C) D) E)

Ondas difusas lentas. Espigas localizadas. Actividad rápida de bajo voltaje. Actividad aleatoria. Ondas delta trifásicas.

79. ¿Cuál es el tratamiento más importante en esta paciente?

195

C) Restricción suave para evitar lesiones. D) Técnicas para fomentar la orientación. E) Tratamiento de la enfermedad subyacente. 80. ¿Cuál es la tasa de mortalidad más probable seis meses después del alta? A) 0 a 20% B) 20 a 40% C) 40 a 60% D) 60 a 80% E) 80 a 100%

A) Haloperidol (Haldol) para disminuir la agitación. B) Lorazepam (Ativan) para regular el sueño.

http://bookmedico.blogspot.com

http://bookmedico.blogspot.com 196

5: Psiquiatría

Respuestas y explicaciones 1. E) 2. C)

Explicación de las preguntas 1 y 2 Esta mujer sufre un episodio de depresión mayor. Interrogar con respecto al abuso de sustancias, medicamentos que consume a la fecha, enfermedades y antecedentes de depresión son aspectos muy importantes para completar la valoración psiquiátrica y del intento suicida. El riesgo de suicidio en mujeres con depresión mayor es casi 20 veces más elevada que sin dicha enfermedad. En individuos con depresión mayor el riesgo calculado de por vida de suicidio es cercano a 15%. La elección del antidepresivo específico está influida por muchos factores lo que incluye respuesta previa en pacientes o en miembros de la familia y enfermedades simultáneas o abuso de sustancias en la familia (y por tanto potencialmente también para la paciente). Si un paciente o si un miembro de su familia tiene antecedentes de síntomas maníacos o trastorno bipolar debe valorarse el inicio de un estabilizador del estado de ánimo antes de iniciar el tratamiento con antidepresivos, porque los antidepresivos pueden cambiar a estos individuos a un estado de manía. (Practice Guideline for the Assessment and Treatment of Patients with Suicidal Behaviors, 2003, APA; Practice Guideline for the Treatment of Patients with Major Depressive Disorder, 2000, APA)

esquizoides y esquizotípicos de la personalidad aparecen indiferentes o extraños, por lo general con una actitud desprendida. Los pacientes con trastorno paranoide de la personalidad muestran una gran desconfianza hacia otros, se sienten vigilados continuamente y muestran resentimiento, pero por general no en la forma de demandas legales (DSM IV). Si se tiene una actitud defensiva únicamente servirá para “confirmar” las sospechas del paciente. La valoración por un psiquiatra, aunque en teoría sería de utilidad, podría no ser beneficiosa dados los mecanismos de defensa primitivos e inflexibles (proyección, negación, etc.) que muestran estos individuos. El paciente puede tener una paranoia progresiva como consecuencia de sus temores de tener cáncer, e informarle que es necesario enviarlo con el psiquiatra puede hacer peores sus temores. De hecho, el paciente puede sentirse más agredido e irritado. La valoración por otro médico está indicada como una segunda opinión para obtener confianza adicional, pero hacer esto con el fin únicamente de evitar problemas legales puede ser percibido como una conducta defensiva y que “hay algo que esconder”. Los individuos con trastorno paranoide de la personalidad responden mejor a una actitud empática, muy profesional y no excesivamente amigable. El responder todas las preguntas con información amplia con respecto al diagnóstico, tratamiento y pronóstico puede servir para fortalecer el vínculo con estos pacientes.

3. C) 5. C) 4. D) 6. B)

Explicación de las preguntas 3 y 4 El paciente presenta un trastorno paranoide, aunque no se encuentra en psicosis franca. Los individuos con trastornos antisociales de la personalidad suelen ser más deshonestos, agresivos y tener actitudes más explosivas. Los pacientes narcisistas se ofenden con facilidad por interacciones cuando no se sienten tratados como individuos “especiales” pero por lo general no muestran una gran desconfianza. Los pacientes con trastornos

196

http://bookmedico.blogspot.com

http://bookmedico.blogspot.com Respuestas: 1–10

Explicación de las preguntas 5 y 6 Los pacientes con depresión bipolar pueden presentarse con manifestaciones psicóticas pero este individuo no tiene antecedentes de episodios maníacos lo que hace difícil el diagnóstico en este momento. Los delirios bizarros (que no es posible que existan en la realidad) y las alucinaciones auditivas no son compatibles con trastorno delirante. Los trastornos esquizoafectivos, de tipo depresivo, incluyen los síntomas psicóticos y de depresión. Sin embargo, los síntomas psicóticos deben durar al menos un mes y ocurren en ausencia de depresión. Para el diagnóstico de esquizofrenia también se necesita al menos un mes de psicosis activa pero un total de seis meses de síntomas atenuados o residuales. Aunque la depresión es muy común en individuos con esquizofrenia, la duración total de la depresión es breve en comparación con los síntomas psicóticos. Esta paciente acude con datos de depresión mayor con características psicóticas, que consisten en depresión del estado de ánimo con síntomas neurovegetativos con duración de al menos dos semanas así como síntomas psicóticos, los cuales sólo se presentan con síntomas del estado de ánimo (DSM IV). Los estabilizadores del estado de ánimo solos o en combinación con fármacos antipsicóticos no son el tratamiento de primera línea para depresión mayor con características psicóticas, sino más bien para manía con o sin características psicóticas. Los estudios han demostrado que la combinación de antidepresivos y antipsicóticos es más eficaz en el tratamiento de la depresión mayor con características psicóticas que la farmacoterapia sola (Practice Guideline for the Treatment of Patients with Major Depressive Disorder, 2000, APA). 7. D) 8. E)

Explicación de las preguntas 7 y 8 Esta mujer sufre de bulimia, que se caracteriza por episodios recurrentes de comidas abundantes acompañadas de conductas de compensación lo que incluye la inducción del vómito y el abuso de diuréticos y laxantes. A causa del vómito repetido de contenido gástrico, los pacientes están propensos a desarrollar diversas anomalías electrolíticas como alcalosis hipoclorémica o hipopotasiemia.

197

La hipernatremia y leucopenia no se observan con frecuencia. En el tratamiento del trastorno bipolar concomitante pueden ser de utilidad los anticonvulsivantes como el ácido valproico y la carbamazepina y los estabilizadores del estado de ánimo como el litio pero no son por sí mismos eficaces para el tratamiento de la bulimia. De la misma forma, los antipsicóticos y las benzodiazepinas pueden utilizarse en trastornos de ansiedad o psicosis concomitantes, pero no son de utilidad en pacientes que comen de manera abundante o se inducen el vómito. Los antidepresivos, en especial los inhibidores de la recaptación de serotonina, han mostrado éxito para disminuir las conductas de consumo abundante de alimentos y el vómito (Synopsis, pp. 748, 750). 9. A) 10. A)

Explicación de las preguntas 9 y 10 La madre del niño muestra un trastorno fingido por el cuidador, en el cual un padre o un cuidador induce de manera intencional la enfermedad en una persona bajo su cuidado. El confrontar a la madre en la sala de urgencias podría dar origen a conductas defensivas, negación e ira. Tal vez la madre abandone el hospital súbitamente con el niño. El pedir la presencia de un psiquiatra en esta situación también puede crear un resultado similar. La valoración por un urólogo y el tratamiento para la infección urinaria pueden estar indicados y ser apropiados, pero no atienden la preocupación inmediata que consiste en el abuso del niño por parte de la madre. El trastorno fingido por un cuidador se considera una forma de abuso infantil y el médico tiene la obligación legal de notificar a los servicios de protección al menor. La hospitalización del niño permite iniciar el tratamiento para la enfermedad y proporciona el tiempo preciso para la intervención de las autoridades apropiadas, si esto es necesario. La producción consciente de síntomas por ganancia secundaria (p. ej., evitar acudir al trabajo, a la escuela, a prisión o a servicio militar) es la base para la simulación. La madre muestra la preocupación aparente que sería de esperarse, pero los métodos elaborados de abuso demuestran una enfermedad significativa. La

http://bookmedico.blogspot.com

http://bookmedico.blogspot.com 198

5: Psiquiatría

producción inconsciente de síntomas o signos por un conflicto inconsciente es la característica clásica de este trastorno. Se cree que la motivación para el trastorno fingido es la producción intencional de la enfermedad con el fin de asumir el papel de enfermo. 11. E) 12. E)

Explicación de las preguntas 11 y 12 A esta mujer se le debe diagnosticar trastorno de conversión. La producción consciente de síntomas para asumir el papel de enfermo es la motivación subyacente al trastorno fingido. La simulación no es un diagnóstico de enfermedad mental, sino la invención consciente o exagerada de síntomas físicos o psiquiátricos con el fin de obtener ganancia secundaria como beneficios por incapacidad, evitar actividades laborales o evitar cumplir sentencias en prisión. La resonancia magnética nuclear normal, los reflejos normales y la ausencia de reflejos patológicos aunado a anestesia por debajo de la cicatriz umbilical no es compatible con enfermedad del sistema nervioso central o periférico. El aparente estrés relacionado con el matrimonio y embarazo recientes tal vez tenga relación con el origen de los síntomas. Los síntomas de conversión se originan a través de la producción inconsciente de síntomas neurológicos por un conflicto inconsciente. La administración de un placebo, por ejemplo solución salina por vía intravenosa, puede resolver los síntomas pero es deshonesto y poco ético. Es innecesaria la hospitalización al área de neurología a menos que haya preocupación con respecto a una enfermedad real subyacente. También puede servir como refuerzo para la somatización de su conflicto. El confrontar a un paciente con trastorno conversivo a menudo provoca empeoramiento subsiguiente de la sintomatología. La valoración por un psiquiatra puede ayudar a la paciente a enfrentar la tensión emocional de su disfunción pero en la sala de urgencias también puede ocasionar que la paciente piense que no se le cree e incrementar los síntomas. Muchos casos de trastornos conversivos remiten en forma espontánea, pero la recuperación puede facilitarse en forma significativa a través de apoyo, tranquilización e información de que finalmente habrá mejoría (Synopsis, p. 650).

13. A) El tratamiento con antidepresivos puede ser apropiado si el paciente sufre de enfermedad depresiva, pero antes de establecer este diagnóstico debe ampliarse el interrogatorio. Los síntomas de depresión puerperal son frecuentes y tal vez no necesiten tratamiento. Si se considera que existe peligro inmediato para el niño está indicado llamar a los servicios de protección al menor. Los pensamientos persistentes no equivalen a la realización de éstos, y pensamientos similares a los que se comentan en este caso no son infrecuentes dada la tensión producida por un recién nacido. De nuevo, podría necesitarse más información. Por otra parte, la tranquilización prematura con respecto a sus pensamientos de lesionar al niño sin obtener información adicional podría ser peligrosa si la paciente tiene un plan o intenta lesionar a su hijo. Si la paciente sufre de psicosis puerperal o tiene ideación suicida podría ser necesaria la hospitalización. Sólo con el interrogatorio amplio y análisis de los síntomas, con especial atención a los sistemas de apoyo y a posibles síntomas psicóticos, el médico puede establecer si hay un motivo de preocupación. La psicosis puerperal se considera una urgencia psiquiátrica por el riesgo de lesionar al recién nacido y por lo general requiere hospitalización inmediata. 14. B) 15. E)

Explicación de las preguntas 14 y 15 Este caso demuestra la presentación clínica clásica de delirio, el cual puede manifestarse con diversos síntomas, lo que incluye agresividad, hostilidad, trastornos de la memoria, síntomas psicóticos (en especial alucinaciones visuales) y falta de cooperación como retirarse los catéteres intravenosos y permanecer fuera de la cama. Estos síntomas son comunes en pacientes delirantes, y no son específicos de delirio y pueden observarse en muchas enfermedades psiquiátricas, lo que incluye demencias, trastornos psicóticos, abuso de sustancias, trastornos de la personalidad y otros. La característica distintiva del delirio es el nivel fluctuante de conciencia con el paso del tiempo, que varía desde la agitación a la sedación. La difenhidramina puede tener un efecto sedante, pero por sus efectos anticolinérgicos tal vez empeore el delirio y cause retención urinaria y estreñimiento, sobre todo en ancianos. El donepezilo es otro inhibidor de la acetilcolinesterasa

http://bookmedico.blogspot.com

http://bookmedico.blogspot.com Respuestas: 11–19

que puede estar indicado para las demencias leves a moderadas, sobre todo enfermedad de Alzheimer. No está indicado para el tratamiento del delirio y sería difícil diagnosticar una demencia en el contexto de un estado delirante. La administración de benzodiazepinas como el lorazepam puede ser útil para la agitación causada por el delirio pero también puede causar pérdida de la inhibición del paciente e incrementar la agitación, sobre todo en individuos ancianos. Las benzodiazepinas son el tratamiento preferido para el delirio por supresión etílica (delirium tremens). Una dosis baja de antipsicóticos es la mejor opción para disminuir la agitación en un paciente delirante. Los medicamentos potentes como el haloperidol pueden utilizarse, pero es más probable que causen efectos secundarios extrapiramidales que los antipsicóticos de segunda generación (conocidos como atípicos), por ejemplo la risperidona. 16. D) 17. C)

Explicación de las preguntas 16 y 17 La intoxicación por alcohol y benzodiazepinas con frecuencia se manifiesta con conducta desinhibida, lenguaje farfullado, incoordinación y nistagmo pero no suele manifestarse con resequedad de mucosas o miosis. Los pacientes con sobredosis de anticolinérgicos por lo común muestran síntomas psicóticos y tienen resequedad cutánea similar al caso anterior. No obstante, en la exploración física por lo general muestran dilatación pupilar, piel caliente y taquicardia. La intoxicación por fenciclidina también se manifiesta como nistagmo vertical u horizontal, disartria e incluso coma, pero por lo general causa hipertensión o taquicardia (DSM IV). Este caso es una presentación típica de sobredosis por opioides (p. ej., heroína). La tríada clínica es coma o falta de respuesta a estímulos, pupilas puntiformes y depresión respiratoria. Otros signos incluyen hipotermia, hipotensión y bradicardia. El disulfiram es un medicamento oral que bloquea la deshidrogenasa de aldehído y que causa una reacción nociva en las personas que toman alcohol mientras lo consumen. Es útil para interrumpir el consumo de alcohol pero no está indicado para la sobredosis por alcohol u opioides. El flumazenil es un antagonista de los receptores de benzodiazepinas utilizado para corregir los

199

síntomas de sobredosis de benzodiazepinas, en especial la sedación y depresión respiratoria. Tal vez no tenga efectos en la sobredosis de opioides a menos que se ingiera en forma simultánea con benzodiazepinas. La tiamina por vía intravenosa está indicada para el tratamiento de la encefalopatía de Wernicke por deficiencia de tiamina en alcohólicos. La tríada clásica consiste de trastornos oculomotores, ataxia y delirio. Aunque los individuos con dependencia crónica de opioides a menudo se encuentran desnutridos, la tiamina no previene estas complicaciones con la sobredosis. La fisostigmina es un inhibidor de la acetilcolinesterasa utilizado en el tratamiento de urgencia de la intoxicación por anticolinérgicos, pero puede ser peligroso en sobredosis por opioides porque puede agravar la hipotensión. La naloxona es un antagonista de opioides que cuando se administra por vía intravenosa es el tratamiento de urgencia preferido para la sobredosis de heroína, porque corrige con rapidez la sedación, depresión respiratoria, hipotensión y bradicardia que se observan en casos similares al paciente comentado en la pregunta (Synopsis, pp. 454, 908, 1014, 1046). 18. A) 19. A)

Explicación de las preguntas 18 y 19 Muchos medicamentos psicotrópicos, lo que incluye la mayor parte de los antidepresivos, causan diversos síntomas de disfunción sexual. El coito doloroso y la eyaculación retrógrada no se observan en el tratamiento con antidepresivos. Estos trastornos suelen ser causados por medicamentos de otros grupos farmacológicos, otras enfermedades o por procedimientos quirúrgicos. Los antidepresivos no causan eyaculación prematura, de hecho en realidad los antidepresivos se utilizan para el tratamiento de este trastorno, sobre todo los inhibidores de la recaptación de serotonina. El priapismo es un efecto secundario poco común que se observa en pacientes tratados con trazodona y es incluso menos frecuente con el uso de otros antidepresivos. La disminución de la libido es un efecto secundario frecuente que se observa en individuos que toman antidepresivos, en especial inhibidores selectivos de la recaptación de serotonina. Otros problemas sexuales causados por estos medicamentos incluyen disminución de la erección y eyaculación tardía.

http://bookmedico.blogspot.com

http://bookmedico.blogspot.com 200

5: Psiquiatría

La mayor parte de todos los antidepresivos, lo que incluye los antidepresivos tricíclicos como desipramina y los inhibidores de la monoaminooxidasa como la fenelzina, puede causar disfunción sexual. La fluoxetina es un inhibidor selectivo de la recaptación de serotonina que con frecuencia causa este problema. La venlafaxina es un inhibidor de la recaptación de serotonina y noradrenalina que también causa problemas con el desempeño sexual. El bupropión es un antidepresivo eficaz con propiedades dopaminérgicas cuestionables, que no causa disfunción sexual y también ayuda a tratar la disfunción sexual inducida por antidepresivos (Synopsis, pp. 707, 709, 711, 1029). 20. E) 21. E)

Explicación de las preguntas 20 y 21 Éste es un caso de demencia de tipo vascular (causado por múltiples infartos cerebrales) por hipertensión mal controlada. La atrofia del núcleo caudado que se observa en la corea de Huntington explica el trastorno de los movimientos y la demencia que se presentan en dicha enfermedad. La dilatación de los ventrículos sin atrofia es característica de la hidrocefalia con presión normal, una de las causas de demencia potencialmente reversibles. La tríada consiste de demencia, trastornos de la marcha e incontinencia urinaria. La enfermedad de Pick es una demencia gradualmente progresiva que muestra atrofia marcada y preferencial de los lóbulos frontal y temporal del encéfalo. La atrofia generalizada puede observarse con estudios de neuroimagen en casos de demencia de Alzheimer. La demencia vascular por lo común muestra infartos lacunares en la sustancia blanca en los estudios de resonancia magnética nuclear. Con excepción de las causas reversibles (hidrocefalia con presión normal, causas metabólicas o intoxicación por metales pesados) la mejoría es poco común en las demencias. Una disminución rápida de la función es común en demencias por infección por priones, como la enfermedad de Creutzfeldt-Jakob. Las demencias estables también son poco comunes, y se observan con mayor frecuencia en casos de lesión cefálica. Las demencias de Alzheimer y de Pick muestran empeoramiento constante de la enfermedad con el paso de los años. Los infartos pequeños múltiples causan

demencia vascular que corresponde a deterioro gradual en el funcionamiento del paciente. 22. E) 23. D)

Explicación de las preguntas 22 y 23 El paciente sufre enfermedad de Parkinson, un trastorno ocasionado por disminución de la transmisión dopaminérgica. El núcleo del rafe caudal es el origen del sistema serotoninérgico del encéfalo. El hipocampo participa en los procesos de la memoria y emocional. El locus ceruleus es el sitio donde se ubican los cuerpos celulares de noradrenalina. El núcleo basal de Meynert es el sitio donde se origina el neurotransmisor acetilcolina. El sistema nigroestriado se origina en la sustancia negra. Es el haz dopaminérgico primario en el sistema nervioso central y se afecta de manera significativa en la enfermedad de Parkinson. La preocupación en el tratamiento de la agitación y psicosis en pacientes con enfermedad de Parkinson es que los antipsicóticos bloquean ciertos receptores dopaminérgicos lo que más tarde empeora los síntomas de parkinsonismo. La clozapina tiene pocos efectos extrapiramidales, pero por el riesgo de agranulocitosis se necesita la realización regular de estudios de sangre y por tanto no es práctica como tratamiento de primera línea. El haloperidol es un neuroléptico de gran potencia. Es eficaz en el tratamiento de los síntomas psicóticos y para reducir la agitación, pero su potencia también presenta un riesgo significativo de empeoramiento de la enfermedad de Parkinson. La risperidona es un antipsicótico atípico, de segunda generación. Aunque el riesgo de síntomas extrapiramidales con dosis bajas es menor que con el haloperidol, la risperidona tiende a ser más problemática cuando se compara con otros medicamentos atípicos. La tioridazina es otro antipsicótico antiguo. La menor potencia da origen a menos síntomas extrapiramidales, y por tanto existe menos probabilidad de que empeoren los síntomas de parkinsonismo pero tiene efectos secundarios anticolinérgicos que pueden empeorar la confusión. Un riesgo que causa más preocupación es la prolongación del intervalo QTc en el electrocardiograma, que en potencia puede causar arritmia ventricular. La quetiapina es un antipsicótico de segunda generación que prácticamente no causa síntomas extrapiramidales. Esto

http://bookmedico.blogspot.com

http://bookmedico.blogspot.com Respuestas: 20–28

le da una ventaja singular en el tratamiento de la psicosis, agitación o ambos en pacientes con enfermedad de Parkinson sin empeoramiento de los trastornos motores. 24. D) 25. B)

Explicación de las preguntas 24 y 25 Esta mujer experimenta síntomas compatibles con una fobia específica a la sangre, a las inyecciones y a las lesiones. Todas las fobias restantes se caracterizan por descarga simpática que causa taquicardia e hipertensión, pero las fobias causadas por sangre o lesiones por lo común causan una respuesta vasovagal. Esto puede manifestarse con taquicardia inicial seguida por bradicardia e hipotensión. A causa de la descarga simpática, los individuos con ciertas fobias, con mayor frecuencia la fobia social, en ocasiones pueden tratarse con bloqueadores β, en especial cuando se sabe el momento en que ocurrirá la exposición. Los bloqueadores β pueden empeorar los síntomas de fobia a lesiones y sangre por la naturaleza vasovagal de la respuesta. Las terapias de apoyo y orientadas al individuo no son de particular utilidad en el tratamiento de fobias, porque éstas por lo común requieren técnicas conductuales específicas. Los inhibidores selectivos de la recaptación de serotonina pueden ser eficaces en ciertos trastornos de ansiedad, como la fobia social, trastorno de pánico y trastorno de ansiedad generalizada, pero no son útiles para fobias como las inducidas por sangre y lesiones. La terapia de exposición se considera como el tratamiento óptimo para fobias en general, y en especial para fobias específicas. En esta terapia el paciente se expone a un estímulo fóbico particular de manera que cause ansiedad progresiva, acompañado de ciertas técnicas de relajación. 26. B) El trastorno bipolar por lo regular se presenta al final de la adolescencia o al inicio de la edad adulta. Un episodio maníaco consiste de síntomas como disminución en la necesidad de sueño, aumento en la energía, verborrea y estado de ánimo irritable o elevado. Los individuos con trastorno paranoide de la personalidad se muestran sospechosos y tienen desconfianza crónica. Aunque estos pacientes pueden distorsionar la realidad, no tienen ideas delirantes evidentes

201

como en el caso de esta pregunta. Los trastornos esquizoafectivos y la esquizofrenia muestran síntomas psicóticos evidentes, lo que incluye delirio, alucinaciones y desorganización del pensamiento. Además, los pacientes con trastornos esquizoafectivos tienen un episodio depresivo o maníaco mayor, en tanto que los pacientes con esquizofrenia presentan alteración significativa social u ocupacional. En este caso no se observan estos criterios. El paciente muestra un trastorno delirante, que consiste en ilusiones no bizarras (es decir, que en realidad pueden existir) sin alteración significativa de la función, conductas extrañas o presencia de trastornos mayores del estado de ánimo. La edad de inicio para el trastorno delirante por lo común es durante la edad madura, en tanto que otras enfermedades se presentan en edades más tempranas. 27. D) 28. C)

Explicación de las preguntas 27 y 28 Este niño sufre de trastorno de hiperactividad con déficit de atención (ADHD). La acetilcolina participa en algunos casos de delirio, al igual que en la enfermedad de Alzheimer. Es más probable que GABA desempeñe una función en los trastornos de ansiedad, porque es el neurotransmisor inhibidor más prevalente en el encéfalo. Por el contrario, el glutamato es el neurotransmisor excitador más común. Se ha estudiado su relación con la esquizofrenia, porque la fenciclidina (que afecta los receptores de glutamato) causa una psicosis similar a la esquizofrenia. La serotonina se asocia con diversas enfermedades mentales, lo que incluye trastornos de ansiedad, trastornos depresivos, impulsividad y esquizofrenia. La noradrenalina (y posiblemente la dopamina) es el neurotransmisor que tal vez participa en la fisiopatología y tratamiento de ADHD. La noradrenalina participa en la atención y los medicamentos estimulantes, que son muy eficaces para tratar ADHD incrementan la liberación de noradrenalina y dopamina. Los estimulantes como el metilfenidato se consideran tratamiento de primera línea para ADHD. Aunque pueden incrementar la frecuencia de tics en pacientes con trastornos subyacentes, no están contraindicados de manera absoluta en estos pacientes en particular. La razón de riesgo/ beneficio de utilizar estos medicamentos debe

http://bookmedico.blogspot.com

http://bookmedico.blogspot.com 202

5: Psiquiatría

analizarse con los cuidadores del paciente. Los fármacos estimuladores pueden inhibir, en cierto grado, el crecimiento de los niños pero el uso de los medicamentos durante los fines de semana proporciona el tiempo adecuado para permitir el crecimiento. La administración de estimulantes no ha demostrado incrementar el riesgo de farmacodependencia a futuro. De hecho, la farmacoterapia estimulante puede en realidad disminuir el riesgo cuando se compara con la ADHD no tratada. Antes se creía que los estimulantes ejercían sus efectos terapéuticos a través de sedación paradójica; no se cree que éste sea el caso. Incluso se ha observado mejoría similar en la atención y conducta en personas sin ADHD que toman estimulantes. Por tanto, los efectos positivos son de intensidad, más que por diferencias cualitativas (Synopsis, pp. 106, 107, 1224, 1228).

tratamiento de primera línea y de mayor eficacia para la enfermedad de Tourette es el uso de antagonistas de dopamina, con antipsicóticos como el haloperidol. Las causas de diversas enfermedades, entre éstas la enfermedad de Tourette y trastorno obsesivo-compulsivo pueden estar relacionadas con procesos autoinmunitarios. Se cree que la infección por ciertos microorganismos, específicamente infecciones por estreptococos, puede actuar de manera sinérgica con una vulnerabilidad genética para causar estas enfermedades mentales. El significado pleno de esto en términos de diagnóstico, prevención o tratamiento de estos trastornos aún debe establecerse (Synopsis, p. 1247). 32. C) 33. A)

29. C)

Explicación de las preguntas 32 y 33

30. B) 31. E)

Explicación de las preguntas 29 a 31 Esta niña tiene enfermedad de Tourette, que se caracteriza por la presencia de tics motores y vocales desde el primer año de edad. No hay incremento significativo en la frecuencia de trastornos de autismo, trastorno depresivo mayor, de pánico o de conducta. Sin embargo, aparece con gran frecuencia en forma simultánea con trastorno de hiperactividad con déficit de atención y trastorno obsesivo-compulsivo en individuos con enfermedad de Tourette. El lorazepam es una benzodiazepina que puede ser beneficiosa en el tratamiento a corto plazo de la ansiedad asociada con el síndrome de Tourette, pero no está indicada para el tratamiento de los tics por sí mismos. El metilfenidato es un estimulante que puede utilizarse si hay trastorno de hiperactividad con déficit de atención asociado con el tic, pero puede incrementar la frecuencia de éstos. La paroxetina es un inhibidor selectivo de la recaptación de serotonina que se utiliza para el tratamiento de trastornos depresivos y trastornos obsesivo-compulsivos, pero no está indicada para el tratamiento de enfermedad de Tourette. La clonidina es un agonista α2 adrenérgico que en ocasiones puede ser útil para reducir algunos síntomas de la enfermedad de Tourette. El

Este paciente tiene trastorno obsesivo-compulsivo (OCD). Las benzodiazepinas como el alprazolam pueden ser de utilidad para la ansiedad aguda relacionada con OCD, pero no son el tratamiento de primera línea para reducir las obsesiones o las compulsiones. En pacientes con OCD grave e intratable en ocasiones se utilizan antipsicóticos como la olanzapina junto con otros psicotrópicos pero no se recomiendan como monoterapia por su perfil de efectos secundarios significativos. Los antidepresivos que afectan principalmente a la noradrenalina como el bupropión y la desipramina no son particularmente eficaces para el tratamiento de OCD. Los fármacos serotoninérgicos como el citalopram y la clomipramina (un antidepresivo tricíclico) han demostrado mejorar las obsesiones y compulsiones. Por tanto, se cree que en el OCD participa el sistema serotoninérgico. La desensibilización con movimientos ocu lares y reprocesamiento (EMDR) es un tratamiento utilizado específicamente para el trastorno de estrés postraumático. Aunque las psicoterapias con psicoanálisis y psicodinamia pueden ser beneficiosas en algunos individuos con OCD, no se cuenta con estudios suficientes para documentar su eficacia. La psicoterapia de apoyo puede ayudar a los pacientes a enfrentar sus ansiedades y limitaciones graves, pero no es particularmente útil para las obsesiones y compulsiones en sí mismas. La terapia conductual

http://bookmedico.blogspot.com

http://bookmedico.blogspot.com Respuestas: 29–38

ha demostrado de manera consistente su éxito para el tratamiento de OCD y los estudios han demostrado que es tan eficaz como la farmacoterapia (Synopsis, p. 623). 34. B) 35. C)

Explicación de las preguntas 34 y 35 Es probable que este paciente experimente ataque de pánico como parte de un trastorno de pánico. Los antidepresivos tricíclicos como la imipramina y los inhibidores selectivos de la recaptación de serotonina como la paroxetina son eficaces para el tratamiento del trastorno de pánico, pero los beneficios terapéuticos podrían tardar varias semanas. Los medicamentos antipsicóticos como la risperidona no han mostrado eficacia en el trastorno de pánico. Sus efectos secundarios y su toxicidad a largo plazo parecen ser excesivamente elevados. El ácido valproico y otros anticonvulsivantes/estabilizadores del estado de ánimo no se han estudiado tan ampliamente en esta población de pacientes, y cualquier efecto positivo podría tardar varias semanas. Las benzodiazepinas como el lorazepam y el alprazolam han mostrado ser eficaces en el tratamiento de trastornos de pánico. Su inicio de acción más rápido (horas a días) los hace adecuados para el tratamiento inmediato y agudo de ataques de pánico. La imipramina y otros antidepresivos tricíclicos han demostrado su eficacia en el trastorno de pánico. Las desventajas son varias: la necesidad de incrementar la dosis terapéutica con el paso del tiempo, un perfil significativo de efectos secundarios y la letalidad de las sobredosis. Las benzodiazepinas como el lorazepam son eficaces en el tratamiento a largo plazo del trastorno de pánico, pero el riesgo de abuso y de síndrome de supresión cuando se reduce la dosis los hace poco ideales. No es inusual iniciar con benzodiazepinas para un alivio más inmediato de la ansiedad, junto con otro fármaco que requerirá mayor tiempo hasta que los beneficios sean aparentes. La risperidona y el ácido valproico no son tratamientos apropiados de primera línea para el trastorno de pánico. La paroxetina y otros inhibidores de la recaptación de serotonina son opciones más adecuadas para la farmacoterapia a largo plazo del trastorno de pánico por su eficacia probada, bajo perfil de efectos secundarios,

203

carencia de riesgo de abuso y seguridad en casos de sobredosis (Practice Guideline for the Treatment of Patients with Panic Disorder, 1998, APA). 36. D) Esta mujer sufre un duelo, que no puede ser diagnosticado como una enfermedad mental. El duelo se considera una reacción normal ante la muerte de un ser amado. La hospitalización estaría indicada si la paciente estuviera en riesgo inminente para sí misma (o para otros) o si es incapaz de cuidar de sí misma. No tiene ideación suicida y trabaja de manera adecuada, por tanto no es útil o necesaria la hospitalización. El inicio de tratamiento con antidepresivos sería apropiado si se tratara de trastorno depresivo mayor (MDD). La paciente muestra ciertos síntomas compatibles con MDD, pero tiene menos de dos meses desde la muerte súbita de su cónyuge y sus síntomas no son tan intensos como los observados en MDD (DSM IV). Otro factor que favorece la presencia de duelo sobre MDD es que la paciente no tiene antecedentes de depresión o ideación suicida. La farmacoterapia con antidepresivos y antipsicóticos podría ser el tratamiento preferido si sufriera MDD con características psicóticas. Aunque en ocasiones tiene trastornos de la percepción, este fenómeno no es inusual en el duelo complicado. Su conciencia y la falta de síntomas psicóticos, como ideas delirantes o desorganización mental, no son compatibles con la enfermedad psicótica mayor. Los individuos con duelo por lo general no requieren la valoración por un psiquiatra a menos que exista otro trastorno mental o el problema esté complicado. Dada la naturaleza limitada del duelo, la conducta más apropiada es la vigilancia de los síntomas con el paso del tiempo. También puede ser útil enviarla a terapia de duelo, ya sea individual o grupal. Si los síntomas de la paciente continúan por más de ocho semanas o hay una alteración en el funcionamiento o riesgo para ella, entonces está indicada la valoración por un psiquiatra con hospitalización. 37. D) 38. E)

Explicación de las preguntas 37 y 38 Los pacientes ancianos con disminución de la función cognitiva por depresión (que en ocasiones se denomina seudodemencia) muestran datos característicos en el examen minimental. Por lo

http://bookmedico.blogspot.com

http://bookmedico.blogspot.com 204

5: Psiquiatría

general tienen una gran preocupación con respecto a sus problemas, e incluso hacen énfasis en sus problemas a diferencia de los pacientes dementes, quienes intentan ocultar o minimizar su déficit y no parecen preocupados. Los pacientes con seudodemencias son capaces de responder bien a las preguntas pese a los síntomas cognitivos. Sin embargo, los individuos con demencia tienen dificultad significativa con la atención y concentración. Los pacientes con depresión están más propensos a demostrar su mala memoria en comparación con aquellos con demencia, quienes por lo común niegan tener la enfermedad. En las pruebas cognitivas, los individuos con seudodemencia presentan resultados inconsistentes, y en ocasiones presentan mejor desempeño que otros. Los pacientes con demencia muestran mal desempeño de manera consistente en varias pruebas que se dirigen a la misma función. Este paciente presenta características de enfermedad con cuerpos de Lewy, una demencia que puede relacionarse con la de Alzheimer. La tríada clásica de la demencia con cuerpos de Lewy es un curso fluctuante, alucinaciones pedunculares (alucinaciones visuales de personas pequeñas, animales u objetos) y síntomas de parkinsonismo. Estos pacientes suelen ser muy sensibles a los efectos secundarios extrapiramidales, y por tanto se evita el uso de antipsicóticos como la risperidona (Synopsis, pp. 333, 340). 39. B) 40. A)

Explicación de las preguntas 39 y 40 Este individuo presenta los criterios para trastorno de conducta. El trastorno antisocial de la personalidad puede diagnosticarse solamente en una persona de más de 18 años de edad. De hecho, el diagnóstico de trastorno antisocial de la personalidad requiere evidencia de trastornos de la conducta antes de los 15 años de edad (DSM IV). Los niños con autismo, esquizofrenia y retraso mental pueden mostrar conducta agresiva o trastornada, pero estas enfermedades no necesariamente predicen un trastorno futuro de la conducta. Los pacientes con trastorno de hiperactividad con déficit de atención y trastornos del aprendizaje se encuentran en mayor riesgo de desarrollar trastornos de la conducta en comparación con individuos de mayor edad.

Esto no es raro en sujetos con trastorno de la conducta que tienen antecedente de trastorno de oposición desafiante (ODD) como los niños pequeños. Los trastornos de la conducta a menudo se consideran como un espectro, con ODD en etapas tempranas seguidas por trastornos de la conducta y por último trastorno antisocial de la personalidad. Los trastornos de la conducta por si mismos no predicen el trastorno de pánico, trastorno esquizoide de la personalidad o el desarrollo de esquizofrenia. Si no se trata, hay un incremento significativo en el riesgo de desarrollar farmacodependencia, lo cual indica un mal pronóstico (Synopsis, p. 1237). 41. A) 42. D)

Explicación de las preguntas 41 y 42 Este paciente experimenta síntomas de trastorno de estrés postraumático (PTSD). El factor pronóstico de mayor importancia de síntomas futuros es la proximidad, gravedad y duración de la experiencia traumática. Sin embargo, hay otros factores de riesgo que incluyen enfermedades psiquiátricas simultáneas o que se presentaron antes de la enfermedad. Las mujeres están más propensas a desarrollar PTSD. Se cree que la disociación durante el traumatismo interfiere con la fijación de los recuerdos, lo que puede incrementar la probabilidad de síntomas de ansiedad en el futuro. La presencia de ciertas características de la personalidad como personalidades limítrofe, paranoide y antisocial también hacen al individuo más vulnerable a desarrollar PTSD. Los antecedentes de experiencia traumática durante la infancia, ya sea física, sexual o emocional son factores pronósticos fuertes de PTSD en el futuro. En ocasiones en conjunto con la psicoterapia se ha utilizado la administración de amobarbital o una entrevista después de la administración de amobarbital a fin de ayudar a los individuos a trabajar con su evento traumático. No se ha utilizado como tratamiento solo por su riesgo de adicción y la letalidad de la sobredosis. Se cuenta con pocos datos de la utilización de antipsicóticos como el haloperidol en síntomas de PTSD, pero pueden utilizarse en forma aguda para el tratamiento de la agitación o violencia. El lorazepam también se utiliza en forma similar, pero dada la alta comorbilidad de abuso de sustancias en pacientes con PTSD, no se recomienda como tratamiento único.

http://bookmedico.blogspot.com

http://bookmedico.blogspot.com Respuestas: 39–47

La trazodona, en dosis bajas, puede utilizarse para tratar el insomnio en estos individuos. El tratamiento de los síntomas de PTSD podría requerir dosis plenas de antidepresivos, lo que conlleva efectos secundarios significativos como sedación diurna e hipotensión ortostática. La sertralina y otros inhibidores selectivos de la recaptación de serotonina son tratamientos muy eficaces y bien tolerados para PTSD. Estos medicamentos mejoran los síntomas de PTSD (p. ej., recurrencia de los síntomas, evitación de estímulos y aumento del estado de alerta). Con base en su eficacia, tolerabilidad, falta de potencial de abuso y seguridad en sobredosis, se consideran el tratamiento de primera línea para el PTSD (Synopsis, pp. 624, 630). 43. D) Este preescolar ha desarrollado signo de pica, el consumo de sustancias no nutritivas. La disminución en las concentraciones de folato puede relacionarse con depresión y demencia en adultos, pero esto no se observa en casos de signo de pica. En niños con signo de pica puede observarse el consumo de pinturas con plomo, lo cual conduce a intoxicación por plomo y aumento en las concentraciones de éste. Las deficiencias de hierro y de zinc se consideran causas potenciales del signo de pica, en especial cuando se observa que los niños comen tierra y barro. A causa de esto, deben medirse las concentraciones sanguíneas de estos elementos (Synopsis, p. 1242). 44. E) El trastorno deAsperger es un trastorno penetrante del desarrollo (PDD) manifestado por alteración en la interacción social y conductas estereotipadas sin las anomalías adicionales del lenguaje que se observan en caso de autismo. Los trastornos de desintegración de la infancia también son un tipo de PDD que se caracteriza por desarrollo normal hasta los dos años de edad, seguido por una disminución rápida en el uso del lenguaje, habilidades motoras y de la interacción social. El síndrome de cromosoma X frágil es un síndrome genético que se caracteriza por retraso mental, características físicas típicas y un alta tasa de PDD. La paciente mencionada muestra datos compatibles con trastorno de Rett, un PDD con empeoramiento progresivo que se observa únicamente en mujeres. Las pacientes con síndrome de Rett por lo común muestran desarrollo normal hasta al menos los cinco años de edad, y más tarde sufren desaceleración del crecimiento cefálico, movimientos estereotipados de las manos, pérdida de la capacidad para las relaciones sociales, dificultades en la marcha y alteración del lenguaje (DSM IV).

205

45. C) 46. D)

Explicación de las preguntas 45 y 46 Es probable que este sujeto desarrolle discinesia tardía, un trastorno tardío de los movimientos relacionado con el uso crónico de antipsicóticos. Está indicado añadir un fármaco anticolinérgico para el tratamiento de la distonía aguda, pero no es eficaz para el tratamiento de la discinesia tardía. La continuación de las dosis actuales de antipsicóticos no reducirá los movimientos del paciente sino que los incrementará con el paso del tiempo. La interrupción de los fármacos psicotrópicos no reduce la discinesia y conlleva un riesgo elevado de recaída de la psicosis. Una vez que el individuo tiene discinesia tardía la reducción de la dosis (si existe indicación clínica) puede disminuir la progresión o incluso mejorar los movimientos anormales. El paciente muestra características compatibles con síndrome neuroléptico maligno (NMS) una enfermedad que pone en riesgo la vida y que se asocia con el tratamiento con antipsicóticos. El añadir benztropina al tratamiento no es de utilidad para el NMS. Se recomienda la interrupción inmediata de antipsicóticos. Para el tratamiento de este paciente también puede utilizarse dantroleno, un relajante muscular y bromocriptina, un agonista de los receptores de dopamina (Synopsis, pp. 1059-1060). 47. E) Este individuo cursa con agitación y psicosis, tal vez causada por intoxicación por abuso de sustancias. La intoxicación con alcohol puede causar inestabilidad y conducta agresiva, pero no se manifiesta con hipertensión y taquicardia. La marihuana puede causar taquicardia y sentimientos de paranoia, pero por si misma no ocasiona brotes de violencia o conducta desorganizada evidente. La intoxicación con heroína u otros opioides más a menudo se manifiesta con somnolencia o apatía más que con síntomas psicóticos evidentes y en el caso presentado en la pregunta, el individuo tiene datos de descarga simpática. La dietilamida del ácido lisérgico (LSD) y otros alucinógenos obviamente causan síntomas psicóticos, pero no son comunes el nistagmo y la rigidez muscular. Los pacientes por lo común no son hostiles como en este caso clínico. La fenciclidina por lo común se manifiesta con conducta impredecible, paranoia y agresividad similar a la

http://bookmedico.blogspot.com

http://bookmedico.blogspot.com 206

5: Psiquiatría

que se comenta en el caso clínico. Con frecuencia se confunde con esquizofrenia. Los datos en la exploración física incluyen nistagmo, hipertensión, taquicardia, ataxia y rigidez muscular (DSM IV). 48. E) 49. A)

Explicación de las preguntas 48 y 49 Las tasas de éxito para la interrupción del tabaquismo con interrupción súbita y educación/ recomendaciones son muy bajas cuando se utilizan solas. Las tasas se incrementan de manera significativa con intervenciones conductuales o con el uso de medicamentos, como la sustitución de nicotina. Las tasas más altas de éxito se observan en pacientes con la combinación de medicamentos más tratamiento conductual, en forma de terapia de grupo (Synopsis, p. 446). Los aspectos del refuerzo de la adicción a la nicotina parecen incluir el sistema dopaminérgico en el sistema nervioso central. Esto puede ser una razón por la cual el bupropión, que probablemente incrementa la actividad dopaminérgica, es muy eficaz para ayudar a los pacientes a interrumpir el tabaquismo. Los otros antidepresivos enumerados no tienen eficacia demostrada para dependencia a la nicotina. 50. A) 51. B)

Explicación de las preguntas 50 y 51 La paciente no refiere depresión significativa, anhedonía, problemas con el apetito o sentimiento de culpa compatible con trastorno depresivo mayor. Tampoco refiere obsesiones o compulsiones específicas necesarias para establecer el diagnóstico de trastorno obsesivo compulsivo (OCD) como temor a la suciedad, deseo de lastimar a alguien o necesidad de simetría. La paciente no tiene ataques de pánico, estos no son inesperados, y se relaciona con preocupaciones con respecto a aspectos cotidianos. Tampoco presenta temores continuos de tener más ataques característicos de trastorno de pánico. La fobia social consiste en el temor de participar en conductas incómodas o humillantes en público, lo cual no es aparente en el caso mencionado. La paciente refiere síntomas

de ansiedad excesiva y preocupación con respecto a diversas actividades asociadas con síntomas cognitivos y físicos. El caso satisface los criterios para trastorno de ansiedad generalizada (GAD) (DSM IV). La buspirona y las benzodiazepinas son muy eficaces para el tratamiento de GAD. La buspirona tiene la gran ventaja de que no causa adicción, como las benzodiazepinas. La buspirona ha mostrado menos eficacia en el tratamiento de GAD después de que se ha intentado el tratamiento con benzodiazepinas. Otra desventaja de la buspirona es que requiere varias semanas para mejorar los síntomas de ansiedad, y el inicio de acción de las benzodiazepinas es en lapso de horas. A causa de esto, una estrategia útil para el tratamiento de GAD es iniciar el tratamiento con buspirona y benzodiazepinas con el plan de reducir la dosis de benzodiazepinas conforme inicie el efecto de la buspirona. Este fármaco también ha mostrado utilidad para reducir los síntomas cognitivos de GAD más que los síntomas somáticos (Synopsis, p. 635). 52. E) 53. D)

Explicación de las preguntas 52 y 53 Este sujeto probablemente consumió MDMA (éxtasis) durante una fiesta. La cocaína tal vez produzca sus efectos a través del bloqueo de la recaptación de dopamina, que es la causante de la estimulación, y por tanto de su naturaleza adictiva. Los individuos intoxicados con fenciclidina (PCP) a menudo están agitados pero por lo común muestran nistagmo y con frecuencia tienen conducta violenta. La PCP actúa a través del bloqueo de los receptores de glutamato. Se cree que los alucinógenos incrementan la actividad del sistema serotoninérgico y que no necesariamente se presentan con los sentimientos de euforia que se observan en este caso. La intoxicación por anfetaminas, que causan liberación de dopamina, puede parecer similar al caso de la pregunta, pero los trastorno floridos de la percepción no son tan frecuentes. El éxtasis que por lo general se consume durante fiestas, actúa como una anfetamina y alucinógeno, creando sentimiento de bienestar o euforia y con producción de alucinaciones. Su naturaleza dual probablemente está en relación con sus efectos neurofisiológicos sobre la liberación de dopamina y serotonina en el encéfalo.

http://bookmedico.blogspot.com

http://bookmedico.blogspot.com Respuestas: 48–59

El sujeto tiene antecedente de trastorno persistente de la percepción con alucinaciones (recuerdos vívidos) que se caracteriza por la reexperimentación de trastornos de la percepeción después de la interrupción del consumo de la droga. No existe un medicamento que trate de manera definitiva los recuerdos vívidos, pero pueden ser útiles varios fármacos. Éstos incluyen anticonvulsivantes como la carbamazepina y ácido valproico o las benzodiazepinas como el clonazepam. Los antidepresivos como la fluoxetina podrían estar indicados si el paciente muestra además trastorno depresivo. Los antipsicóticos (incluido el haloperidol) se evitan porque en realidad empeoran los recuerdos vívidos (Synopsis, pp. 414, 440). 54. A) 55. C)

Explicación de las preguntas 54 y 55 Con frecuencia hay enfermedades simultáneas en individuos con dependencia a la cocaína. Los trastornos afectivos (incluida la depresión mayor y el trastorno bipolar) así como los trastornos de ansiedad se observan con frecuencia en pacientes adictos a la cocaína. No hay un incremento apreciable de esquizofrenia en esta población de pacientes. El trastorno antisocial de la personalidad es el diagnóstico que con mayor probabilidad se asocia con la dependencia a la cocaína. El tratamiento con antidepresivos podría estar indicado si hay alguna enfermedad depresiva concomitante, pero no alivia de manera específica los síntomas de supresión. Si el paciente muestra síntomas significativos de supresión etílica podría ser necesaria la reducción gradual de la dosis de benzodiazepinas. La destoxificación con metadona a menudo se utiliza para personas que tienen supresión grave a opioides pero no es apropiada para supresión de cocaína. La destoxificación con fenobarbital puede utilizarse para prevenir la supresión por benzodiazepinas y (con menos frecuencia) para la supresión etílica. A diferencia de la supresión por alcohol, benzodiazepinas o barbitúricos, la supresión por cocaína no pone en riesgo la vida y no requiere intervención farmacológica. La educación con respecto a la adicción y supresión a la cocaína, así como la tranquilización del paciente con respecto a la corta duración de los síntomas, es todo lo que se necesita (Synopsis, p. 429).

207

56. E) 57. B)

Explicación de las preguntas 56 y 57 Esta niña muestra el fenotipo clásico de síndrome de cromosoma X frágil: cabeza grande y alargada, pabellones auriculares grandes, talla baja, articulaciones hiperextensibles y macroorquidia (en varones). La falta de parte del cromosoma 5 es la causa del síndrome de maullido de gato que se caracteriza por microcefalia, implantación baja de los pabellones auriculares y retraso mental grave. Los cromosomas 15 y 21 participan en el síndrome de Down, la causa aislada más común de retraso mental. Muestran ojos rasgados, pliegues epicánticos y aplanamiento de la nariz. El síndrome de cromosoma X frágil es ocasionado por la mutación del cromosoma X. El síndrome del cromosoma X frágil es la segunda causa más común de retraso mental; los individuos afectados tienen retraso mental leve a grave. También se asocia con varias enfermedades simultáneas, lo que incluye trastornos del aprendizaje, autismo y trastorno de hiperactividad con déficit de atención en casi 75% de los casos (Synopsis, pp. 1163, 1165). 58. E) 59. D)

Explicación de las preguntas 58 y 59 Los canabinoides son una de las pocas sustancias de abuso que no afectan la frecuencia respiratoria. El consumo de marihuana por lo común produce síntomas de resequedad de boca y aumento del apetito. Contrario a lo que en ocasiones se asegura, la intoxicación con canabinoides altera de manera significativa la función motora y por tanto interfiere con la capacidad de conducir. También causa taquicardia (DSM IV). El síndrome amotivacional es un posible efecto a largo plazo del uso intensivo de canabinoides, pero permanece en controversia. Se caracteriza por apatía y aburrimiento. También se han reportado atrofia cerebral, daño cromosómico y convulsiones pero no se han confirmado en individuos con uso crónico de canabinoides. Las mayores preocupaciones médicas con respecto a fumar marihuana a largo plazo son similares a las

http://bookmedico.blogspot.com

http://bookmedico.blogspot.com 208

5: Psiquiatría

de fumar tabaco, como cáncer pulmonar y enfermedades respiratorias (Synopsis, p. 425). 60. D) 61. B)

Explicación de las preguntas 60 y 61 Esta mujer con mayor probabilidad sufre trastorno depresivo mayor. El tratamiento con inhibidores selectivos de la recaptación de serotonina (SSRI) se considera como el tratamiento de primera línea. Los síntomas neurovegetativos de la depresión (p. ej., insomnio, cambios en el apetito, anergia, mala concentración) en ocasiones mejoran después de varios días de inicio de la farmacoterapia, pero la sensación de depresión y desesperanza puede tardar hasta cuatro a seis semanas para mostrar una mejoría significativa. Mientras la paciente muestre tolerancia a los SSRI, debe evitarse la urgencia de incrementar la dosis con el fin de disminuir los efectos secundarios. Después de iniciar los SSRI debe proporcionarse educación y tranquilización a la paciente con respecto al tiempo esperado hasta que se observe la remisión. La mayoría de los pacientes tolera el tratamiento con SSRI, aunque hay efectos secundarios característicos que pueden presentarse al inicio, como cefaleas, trastornos gastrointestinales y sueños vívidos, éstos se resuelven con el paso de los días o semanas. Con frecuencia ocurre disfunción sexual en forma de impotencia o inhibición de los orgasmos, lo que suele ocurrir varias semanas o meses después de haber iniciado el tratamiento y pueden continuar en tanto éste persista. 62. E) 63. A)

Explicación de las preguntas 62 y 63 Este niño tiene un trastorno de la lectura, un tipo de trastorno del aprendizaje caracterizado por una capacidad de lectura que se encuentra por debajo de lo esperado, tomando en consideración la inteligencia y la edad (DSM IV). La prueba gestáltica visual-motora de Bender no es una prueba diagnóstica, pero puede utilizarse para identificar dificultades en el desempeño perceptual. Las pruebas de proyección psicológica, como la Children’s Apperception Test y la prueba de Rorschach no son útiles para pruebas de inteligencia y no se han

validado como instrumentos para su medición. La prueba neuropsicológica Reitan-Indiana es útil para niños en quienes se sospecha daño cerebral. Para diagnosticar trastornos de aprendizaje es esencial medir la inteligencia a fin de comparar los resultados con las discrepancias en los logros. La escala de inteligencia de Wechsler para niños es una de las utilizadas con mayor frecuencia para este propósito. Muchos pacientes con trastornos de aprendizaje, como el trastorno de lectura, tienen enfermedades simultáneas del eje I. Con frecuencia se encuentran otros trastornos de aprendizaje, como dificultad para aprender matemáticas. Las enfermedades como el trastorno autista y el retraso mental hacen difícil establecer el diagnóstico de un trastorno del aprendizaje. Si se observa otro déficit funcional, las dificultades de aprendizaje deben exceder a las asociadas con éste (DSM IV). Los síntomas de depresión son frecuentes en individuos con trastornos del aprendizaje dados los problemas en el desempeño escolar y la relación con los compañeros. El trastorno con tic no se incrementa de forma significativa en individuos con trastornos del aprendizaje. El trastorno de hiperactividad con déficit de atención se correlaciona en gran medida con los trastornos de la lectura, y hasta en 25% de los casos puede haber relación entre los dos trastornos (Synopsis, pp. 1158-1159,1181). 64. B) 65. C)

Explicación de las preguntas 64 y 65 Este caso tiene las manifestaciones clínicas características de simulación. La producción consciente de los síntomas en la función de enfermo es la motivación que se encuentra atrás del trastorno fingido. No hay evidencia de síntomas psicóticos ni de trastorno con ideas delirantes. La hipocondriasis incluye el temor de tener enfermedades graves más que centrarse en síntomas de dolor. La producción inconsciente de los síntomas por conflicto inconsciente es la característica distintiva del trastorno de conversión, el cual se manifiesta con déficit neurológico. La simulación no puede considerarse una enfermedad mental, y se define como la producción intencional de síntomas motivada por incentivos externos como evitar el trabajo, el servicio militar, la cárcel, o para obtener drogas (como en este caso) (DSM IV).

http://bookmedico.blogspot.com

http://bookmedico.blogspot.com Respuestas: 60–72

En casos de sospecha de simulación, la confrontación o las amenazas de acusación legal con probabilidad ocasionarán mayor hostilidad y dañan la relación médico-paciente. El envío para hospitalización en una institución psiquiátrica o el envío con un psiquiatra pueden tener el mismo efecto y no están indicados a menos que exista otra enfermedad mental o haya dudas con respecto a la seguridad del paciente. Es absolutamente necesario en estos individuos modificar los límites, pero debe mantenerse una conducta profesional. La confrontación suave, acompañada de comprensión de los problemas subyacentes (que han llevado al fingimiento de la enfermedad) son los métodos más útiles. Es necesaria una valoración cuidadosa para establecer si existe o no una enfermedad mental adicional o si es necesario el tratamiento por farmacodependencia. 66. D) 67. D)

Explicación de las preguntas 66 y 67 El alcohol es metabolizado en el hígado y el consumo intenso causa inflamación hepática. Con el abuso de alcohol con frecuencia se elevan las enzimas como ALT y AST, aunque no son específicas de daño hepático o de consumo de alcohol. La gamma-glutamil transferasa (GGT) es más específica para el consumo intenso de alcohol reciente. Los triglicéridos también se encuentran incrementados en alcohólicos. Las concentraciones bajas de hemoglobina y hematócrito por hemorragia a causa de gastritis, o bien la anemia macrocítica, son consecuencia del consumo crónico de alcohol. El disulfirám se utiliza en individuos con dependencia al alcohol. Inhibe la deshidrogenasa de acetaldehído y causa una reacción nociva cuando se combina con alcohol y se utiliza como medida disuasiva y no para disminuir el deseo de consumir alcohol. Los antidepresivos, el litio y los antipsicóticos no han mostrado reducir el deseo de consumo de alcohol. La naltrexona es un antagonista de opioides que ha mostrado ciertos resultados positivos para disminuir el deseo de consumir alcohol. Los supuestos mecanismos incluyen el bloqueo de los receptores opioides con lo que interfieren con el círculo vicioso de satisfacción.

209

68. A) Los canabinoides en grandes cantidades no causan la muerte. La cocaína, fenciclidina y opioides pueden ser letales en dosis excesivas, en particular cuando se combinan con alcohol. Sin embargo por sus efectos similares sobre los receptores GABA en el encéfalo, los barbitúricos (y benzodiazepinas) son especialmente letales cuando se combinan con sobredosis de alcohol. 69. B) 70. B)

Explicación de las preguntas 69 y 70 Esta paciente satisface los criterios para esquizofrenia. Los factores que indican mal pronóstico incluyen inicio temprano de los síntomas, estado civil soltera, síntomas negativos (p. ej., aplanamiento afectivo, limitación en el discurso, bloqueo de los pensamientos, mala atención) y mal funcionamiento psicosocial u ocupacional. Los factores que indican un pronóstico más positivo incluyen inicio tardío, buen funcionamiento premórbido, síntomas del estado de ánimo, estado civil casado y antecedentes familiares de trastornos del estado de ánimo (como trastorno bipolar en este caso). Incluso con el tratamiento antipsicótico apropiado, es baja la probabilidad de mantener un alto nivel de funcionamiento. Los estudios indican que casi 50% de los pacientes con tratamiento muestra sintomatología significativa y presenta mal funcionamiento. Sólo 20 a 40% de los pacientes con esquizofrenia puede llevar su vida sin alteraciones. 71. B) 72. D)

Explicación de las preguntas 71 y 72 Este paciente tiene un episodio depresivo mayor. Es apropiado iniciar con antidepresivos como la mirtazepina. Los primeros síntomas que mejoran en el lapso de días o semanas, con cualquier antidepresivo, son los síntomas neurovegetativos como el insomnio, anergia, trastornos del apetito y de la concentración. Por desgracia, el estado depresivo y la desesperanza a menudo son los últimos síntomas de depresión en ceder.

http://bookmedico.blogspot.com

http://bookmedico.blogspot.com 210

5: Psiquiatría

Al igual que muchas otras enfermedades psiquiátricas y enfermedades médicas, el trastorno depresivo mayor tiende a ser una enfermedad recurrente. Los episodios individuales son muy susceptibles de tratamiento, pero existe un alto riesgo de recurrencia sin el tratamiento continuo. Después de un episodio inicial de depresión, los estudios indican que casi 50 a 85% de los individuos desarrolla episodios subsecuentes de depresión mayor (Practice Guideline for the Treatment of Patients with Major Depressive Disorder, 2000, APA). 73. E) 74. A)

Explicación de las preguntas 73 y 74 Esta mujer tiene un trastorno bipolar, en fechas más recientes un episodio maníaco y ha sido estabilizada con litio. El trastorno bipolar es una enfermedad mental crónica con una alta probabilidad de varios episodios a futuro (manía y depresión). Como en este caso, después de un episodio de manía por lo general se recomienda continuar con tratamiento de mantenimiento en forma indefinida, con el fin de reducir el riesgo de recurrencias a futuro. La paciente ahora ha desarrollado un episodio de trastorno bipolar de tipo depresivo. La adición de antidepresivos como sertralina puede ayudar a la depresión, pero siempre existe el riesgo de provocar un episodio de manía. Hay pocos datos que apoyen el tratamiento con la adición de otro estabilizador del estado de ánimo con el fin de tratar la depresión. La disminución de litio no es de utilidad para la depresión de la paciente y podría permitir que fuera más vulnerable a cambiar a un estado de manía. Las dosis de litio se encuentran en concentraciones terapéuticas, y el incremento de la dosis con probabilidad incrementará los efectos secundarios y puede ocasionar efectos tóxicos. Se ha observado que la lamotrigina puede ser beneficiosa en el tratamiento de la depresión bipolar y no parece incrementar la probabilidad de episodio maníaco, como lo hacen otros antidepresivos. Se recomienda como tratamiento de primera línea para episodios de depresión en pacientes con trastorno bipolar. (Practice Guideline for the Treatment of Patients with Bipolar Disorder, APA). 75. A)

77. A)

Explicación de las preguntas 75 a 77 Este paciente presenta demencia, con mayor probabilidad de Alzheimer. La demencia de Alzheimer es causada, en parte, por deficiencia de acetilcolina. Los medicamentos con efectos anticolinérgicos significativos empeoran el déficit cognitivo y la confusión. El donepezilo puede ayudar en casos de demencia leve a moderada, mejorando el estado cognitivo y tal vez la agitación. En pacientes con trastornos del sueño, de la conducta o ambos, pueden ser beneficiosas las dosis bajas de haloperidol y lorazepam y trazodona pero debe evitarse la difenhidramina por sus efectos anticolinérgicos. La atrofia preferencial de la región frontotemporal es compatible con enfermedad de Pick, que se presenta de la misma forma que la enfermedad de Alzheimer. La enfermedad de Huntington es otra causa de demencia que se caracteriza por trastorno grave de los movimientos. Se ha demostrado atrofia notable del núcleo caudado, junto con posible atrofia cerebral. La dilatación de los ventrículos sin atrofia es la característica distintiva de la hidrocefalia con presión normal, una de las causas de demencia potencialmente reversible. La tríada clásica de hidrocefalia con presión normal es demencia, trastornos de la marcha e incontinencia urinaria. La segunda causa más común de demencia es la de tipo vascular, a menudo causada por hipertensión descontrolada que puede dar origen a múltiples infartos pequeños de la sustancia blanca que rodea a los ventrículos. La demencia de Alzheimer es la causa más común de demencia y se caracteriza por atrofia cerebral difusa y dilatación de los ventrículos. Las demencias estables sin progresión son inusuales. En ocasiones se observan en pacientes que sufrieron traumatismo cefálico. El deterioro rápido del funcionamiento por lo común se observa en enfermedades ocasionadas por priones, por ejemplo la enfermedad de CreutzfeldtJakob. El estado estable es posible solo en causas reversibles, como las metabólicas. La progresión de la enfermedad por lo común se observa en la demencia vascular, que se cree es causada por infartos lagunares. La mayor parte de las demencias, lo que incluye la de Alzheimer, muestra un deterioro gradual pero estable en el estado cognitivo y en el funcionamiento global.

76. C)

http://bookmedico.blogspot.com

http://bookmedico.blogspot.com Respuestas: 73–80

78. A) 79. E) 80. B)

Explicación de las preguntas 78 a 80 Este paciente muestra signos y síntomas de delirio. El electroencefalograma es muy sensible para el delirio. Las espigas localizadas pueden observarse en pacientes con actividad convulsiva. La actividad aleatoria es característica del estado normal de alerta. La actividad rápida de bajo voltaje es muy específica de delirio secundario a supresión de alcohol o de hipnóticos (sedantes). Las ondas delta trifásicas son características de estados de delirio causados por insuficiencia hepática. No obstante, las demás causas de delirio muestran alentamiento difuso de los trazos electroencefalográficos.

211

Los medicamentos como los antipsicóticos y benzodiazepinas pueden ser beneficiosos para reducir la agitación que a menudo se observa en individuos con delirio. La sujeción suave puede ser necesaria para permitir que el personal que participa en la atención médica realice las pruebas, exploraciones o procedimientos necesarios y evitar que el paciente se retire los catéteres intravenosos, sondas de alimentación etc. Pueden emplearse intervenciones conductuales como imágenes, luces, relojes, calendarios, etc. para reforzar la orientación a la persona en lugar y tiempo. El método principal y esencial en el tratamiento de pacientes con delirio es conocer y tratar la causa subyacente. La presencia de delirio es un signo de mal pronóstico. La tasa de mortalidad después de un año de delirio es de casi 50%. La tasa de mortalidad seis meses después de un episodio de delirio es cercana a 25% (Synopsis, p. 324).

http://bookmedico.blogspot.com

http://bookmedico.blogspot.com 212

5: Psiquiatría

BIBLIOGRAFÍA

American Psychiatric Association. Diagnostic and Statistical Manual of Mental Disorders, 4th ed., Text Revision. Washington, DC: American Psychiatric Association, 2000. American Psychiatric Association. Practice Guideline for the Assessment and Treatment of Patients with Suicidal Behaviors. Washington, DC: American Psychiatric Association, 2003. American Psychiatric Association. Practice Guideline for the Treatment of Patients with Bipolar Disorder, 2nd ed. Washington, DC: American Psychiatric Association, 2002. American Psychiatric Association. Practice Guideline for the Treatment of Patients with Borderline Personality

212

Disorder. Washington, DC: American Psychiatric Association, 2001. American Psychiatric Association. Practice Guideline for the Treatment of Patients with Major Depressive Disorder, 2nd ed. Washington, DC: American Psychiatric Association, 2000. American Psychiatric Association. Practice Guideline for the Treatment of Patients with Panic Disorder. Washington, DC: American Psychiatric Association, 1998. Sadock BJ, Sadock VA. Kaplan and Sadock’s Synopsis of Psychiatry, 9th ed. Philadelphia, PA: Lippincott Williams & Wilkins, 2003.

http://bookmedico.blogspot.com

http://bookmedico.blogspot.com CAPÍTULO 6

Medicina preventiva Preguntas Preguntas 1 a 3 Un varón de 50 años de edad acude a consulta para una revisión anual programada. En este momento el paciente no tiene síntomas específicos, sólo acude a su revisión. Su última consulta fue hace un año y la exploración física es normal. Se solicitan exámenes de laboratorio que se encuentran en límites normales, éstos incluyen colesterol de 172 mg/100 ml con lipoproteínas de alta densidad (HDL) de 45 mg/100 ml y lipoproteínas de baja densidad (LDL) de 100 mg/100 ml. El paciente recibió la vacuna contra el tétanos hace cinco años. Antecedentes personales patológicos: Antecedentes quirúrgicos: Medicamentos:

Sin importancia. 1. Apendicectomía a los 17 años. 2. Vasectomía a los 43 años. Consumo diario de multivitamínicos. Sin alergias conocidas a fármacos.

Alergias: Antecedentes heredofamiliares: Su padre murió a los 78 años de edad de un infarto cardíaco. La madre vive y tiene 76 años de edad. Padece hipertensión y osteoartritis. Tiene un hermano de 48 años de edad sin antecedentes de enfermedades crónicas conocidas. Sus hijos tienen 16, 14 y 8 años de edad y no tienen enfermedades crónicas conocidas. Antecedentes personales no patológicos: Casado, es empleado en el departamento de contabilidad; cuenta con estudios de licenciatura. Niega tabaquismo o uso de drogas con fines recreativos.

Consume una bebida alcohólica (ya sea cerveza o vino) al día. No hace ejercicio en forma regular.

1. ¿Cuál de las siguientes pruebas diagnósticas de consultorio se recomienda para este paciente? A) Medición de la presión arterial. B) Prueba de palpación abdominal para detectar aneurisma de la aorta abdominal. C) Exploración testicular para detectar cáncer testicular. D) Exploración de la totalidad de la piel para detectar cáncer cutáneo. E) Palpación de la glándula tiroides en busca de cáncer tiroideo. 2. ¿Cuál de las siguientes pruebas se recomienda para este paciente? A) B) C) D)

Medición de lípidos en ayuno. Radiografía de tórax. Electrocardiograma (ECG). Detección de glaucoma mediante la medición de la presión intraocular. E) Prueba de sangre oculta en heces. 3. ¿Cuál de las siguientes intervenciones se recomienda para este paciente? A) B) C) D)

Vacuna antineumocócica. Toxoide tetánico. Interrupción del consumo de alcohol. Complementos con betacarotenos para prevenir cáncer y cardiopatías. E) Prueba para depresión con un cuestionario completo al paciente.

http://bookmedico.blogspot.com

http://bookmedico.blogspot.com 214

6: Medicina preventiva

4. En la consulta se debe:

Preguntas 4 a 6 Una mujer caucásica de 65 años de edad acude a consulta para su exploración ginecológica anual. Ha sido su paciente por muchos años. También acude para tratamiento de hipertensión e hipotiroidismo. Su último Papanicolaou fue hace cinco años y nunca ha tenido un Papanicolaou anormal. Hace un año se realizó una mamografía que fue normal. La paciente no se realiza autoexploración mamaria. Actualmente se encuentra asintomática. Antecedentes personales patológicos:

Antecedentes gineco obstétricos:

Antecedentes quirúrgicos:

Medicamentos:

Alergias: Antecedentes heredofamiliares:

Antecedentes personales no patológicos:

1. Hipertensión de 15 años de evolución. 2. Enfermedad de Graves tratada con yodo radiactivo a los 50 años de edad. 1. Menarquia a los 14 años de edad. 2. Cuatro embarazos a término con partos vaginales (a los 22, 25, 27 y 32 años de edad). 3. Histerectomía total abdominal y salpingooforectomía bilateral por miomas uterinos a los 47 años de edad. 4. Tratamiento de sustitución con estrógenos de los 47 a los 55 años de edad. 1. Apendicectomía a los 16 años de edad. 2. Histerectomía total abdominal con salpingooforectomía bilateral como se mencionó antes. 1. Hidroclorotiazida, 25 mg al día. 2. Levotiroxina 0.1 mg al día. 3. Cloruro de potasio 20 meq por día. Ninguna. Se desconocen los antecedentes de padres y hermanos porque la paciente fue adoptada. De niña fue sana sin enfermedades crónicas conocidas. Enviudó hace cinco años y no ha vuelto a tener una relación sexual desde la muerte de su esposo; es una profesora retirada; estudios a nivel licenciatura; no fuma cigarrillos, no bebe alcohol ni utiliza drogas; camina 30 a 45 minutos al día para hacer ejercicio.

A) Realizar estudio de Papanicolaou. B) Recomendar que reinicie su tratamiento de sustitución con estrógenos. C) Comentar que puede reducir el riesgo de morir de cáncer de mama al realizarse autoexploración mamaria una vez al mes. D) Solicitar densitometría ósea para diagnóstico de osteoporosis. E) Enviar una muestra para cultivo de orina por bacteriuria asintomática. 5. ¿Cuál de los siguientes trastornos se encuentra en la mayor parte de las muertes de mujeres estadounidenses a los 65 años de edad? A) Cáncer de mama. B) Cáncer de ovario. C) Cáncer pulmonar. D) Enfermedad cardiovascular. E) Neumonía. 6. ¿Cuál de las siguientes vacunas debe recomendarse en forma sistemática a esta paciente? A) Vacuna contra la hepatitis B. B) Vacuna contra sarampión, rubéola, varicela (MMR) si la paciente no ha recibido el refuerzo de la vacuna contra el sarampión. C) Vacuna conjugada antineumocócica (PCV7). D) Vacuna de polisacáridos de neumococo (PPV-23). E) Vacuna contra la hepatitis A.

Preguntas 7 a 9 Un varón de 40 años de edad acude a consulta como un paciente nuevo, porque en fechas recientes se mudó a la ciudad. El paciente ha estado en tratamiento médico por diabetes mellitus tipo 2 por tres años y ha tenido un buen control de la glucemia. Toma metformina, 500 mg cada 12 h y reporta tener concentraciones de glucosa en ayuno menores de 100 mg/10 ml con las mediciones en casa. El paciente cuenta con información de sus valoraciones médicas anteriores con un estudio de fondo de ojo con dilatación pupilar hace seis meses, que fue normal y las concentraciones de hemoglobina glucosilada (HgbA1c), realizadas hace tres meses, eran de 6.2%. No tiene antecedentes conocidos de coronariopatía. Su última medición de lípidos en ayuno fue hace

http://bookmedico.blogspot.com

213

http://bookmedico.blogspot.com Preguntas: 4–12

14 meses. Se le solicita perfil de lípidos en ayuno y se obtienen los siguientes resultados: Colesterol total: 235 mg/100 ml. Triglicéridos: 210 mg/100 ml. Colesterol HDL: 45 mg/100 ml. Colesterol LDL: 162 mg/100 ml. 7. El tratamiento debe incluir: A) Dieta con bajo contenido de carbohidratos. B) Incremento de la dosis de metformina. C) El paciente debe de iniciar tratamiento con insulina. D) No hacer ningún cambio. E) Prescripción de un inhibidor de la reductasa de hidroximetil glutaril-coenzima A (HMGCoA) (una estatina). 8. El paciente de la pregunta anterior regresa en dos meses y ha estado bien con sus indicaciones. Se repite el perfil de lípidos en ayuno y los resultados son: Colesterol total: 160 mg/100 ml. Triglicéridos: 140 mg/100 ml. Colesterol HDL: 48 mg/100 ml. Colesterol LDL: 98 mg/100 ml. Sus recomendaciones en ese momento son: A) Continuar con el régimen actual sin ningún cambio. B) Agregar ácido nicotínico. C) Agregar ácido fíbrico. D) Valoración del paciente por un nutriólogo para asesoría. E) Incremento de la dosis del inhibidor de la reductasa HMG-CoA. 9. Las concentraciones ideales recomendadas de LDL en un paciente adulto con diabetes mellitus tipo 2 son: A) B) C) D) E)

160 mg/100 ml 130 mg/100 ml 100 mg/100 ml 70 mg/100 ml 50 mg/100 ml

215

Preguntas 10 a 13 Un varón de 42 años de edad acude a consulta para una nueva prescripción de esteroides nasales que utiliza cada verano para controlar su alergia. Se observa que los signos vitales, tomados por la enfermera, son: presión arterial (TA) de 150/95 mmHg. El paciente se encuentra asintomático excepto por los estornudos y congestión nasal. No tiene otros antecedentes personales patológicos de importancia y el único medicamento que recibe son los esteroides nasales. No fuma, no consume alcohol y no realiza ejercicio. Su índice de masa corporal es de 24 kg/m2. 10. El tratamiento inicial para el paciente debe incluir: A) Tratamiento con diuréticos tiazídicos o bloqueadores β. B) Medición de la presión arterial en cada brazo después de que el paciente se encuentre en reposo por 5 min. C) Recomendar el inicio de ácido acetilsalicílico una tableta infantil al día. D) Realizar prueba de esfuerzo para establecer el riesgo de arteriopatía coronaria. E) Interrupción de los esteroides nasales. 11. El paciente regresa para una consulta y su TA es de 146/94 mmHg. Se establece el diagnóstico de: A) B) C) D) E)

Presión arterial elevada sin hipertensión. Prehipertensión. Hipertensión en etapa 1. Hipertensión en etapa 2. Hipertensión en etapa 3.

12. La valoración adicional en este punto debe incluir: A) Recolección de orina de 24 h para medición de proteínas y depuración de creatinina. B) Estudios de Doppler de la arteria renal para valorar estenosis de arteria renal. C) Electrocardiograma (ECG). D) Ecocardiograma para valorar posible hipertrofia ventricular. E) Medición sérica de hormona estimulante de tiroides (TSH).

http://bookmedico.blogspot.com

http://bookmedico.blogspot.com 216

6: Medicina preventiva

13. De las opciones enumeradas, el tratamiento más apropiado es: A) Recomendar un bajo consumo de sal en la dieta y nueva consulta en nueve a 12 meses. B) Incrementar la dosis del medicamento antihipertensivo que se inició previamente. C) Tratamiento con bloqueadores de los conductos de calcio. D) Tratamiento con diuréticos tiazídicos. E) No se recomienda ninguna intervención en este momento.

Preguntas 14 a 17 Un estudio reciente comparó dos fármacos (exemestano y tamoxifeno) para el tratamiento del cáncer de mama positivo para receptores de estrógenos en mujeres posmenopáusicas. Al final del estudio, 91.5% de las mujeres tratadas con el exemestano y 86.8% de las mujeres tratadas con tamoxifeno no sufrieron enfermedad (P 350 U/L. En el segundo grupo se encuentran los criterios a las 48 h (segundo día de hospitalización) e incluyen: disminución del hematocrito en más de 10%, aumento de las concentraciones de nitrógeno ureico sanguíneo en más de 5 mg/100 ml, disminución en las concentraciones de calcio a menos de 8 mg/100 ml, presión parcial de oxígeno en sangre arterial menor de 60 mmHg, déficit de base >4 meq/L y secuestro de líquidos de más de 6 L. Si el paciente satisface de cero a dos de los criterios, la tasa de mortalidad es de sólo 2%, en tanto que la presencia de tres a cuatro criterios conlleva una tasa de mortalidad de 15%. Si el paciente satisface cinco o seis criterios, la tasa de mortalidad es de 40% y con siete criterios, la tasa de mortalidad se acerca a 100%. Los pacientes con tres o más criterios de Ranson deben ser hospitalizados en la unidad de cuidados intensivos para tratamiento enérgico, reposo intestinal,

http://bookmedico.blogspot.com

http://bookmedico.blogspot.com Respuestas: 47–50

descompresión con sonda nasogástrica y exámenes de laboratorio y exploración física seriados. La cirugía se reserva para complicaciones de la pancreatitis, lo que incluye formación de absceso pancreático, pancreatitis hemorrágica, necrosis pancreática infectada o enfermedad resistente al tratamiento con deterioro del estado general del paciente. (Schwartz, pp. 1476-1478) 49. D) Las causas comunes de pérdida súbita e indolora de la visión que dura más de 24 h incluyen desprendimiento de retina, oclusión venosa o arterial de la retina y neuritis isquémica del nervio óptico. Las cataratas del glaucoma de ángulo abierto se encuentran entre las causas más comunes de pérdida indolora de la visión que es gradual en el lapso de meses o años. El glaucoma de ángulo agudo es una causa de pérdida aguda de la visión, que se acompaña de dolor. Las abrasiones corneales por lo general se manifiestan con eritema ocular. (Wills Eye Manual, Chap 1) 50. C) La esferocitosis hereditaria es una anemia hemolítica que se caracteriza por flexibilidad y forma anormales de los eritrocitos por deficiencia

337

o disfunción de varias proteínas de membrana. Desde el punto de vista clínico, los pacientes con esferocitosis hereditaria varían desde trastorno asintomático con concentraciones casi normales de hemoglobina a hemólisis y anemia graves, que da origen a ictericia potencialmente letal e insuficiencia cardíaca congestiva en recién nacidos. Los pacientes con anemia leve por lo general se diagnostican en etapas avanzadas de la vida durante la valoración por trastornos no relacionados. Algunos pacientes pueden desarrollar cálculos vesiculares en edades tempranas, que son indicación para colecistectomía. La característica distintiva para el diagnóstico de esferocitosis hereditaria es la presencia de esferocitos en un extendido de sangre periférica. Además de los índices de hemólisis (reticulocitosis, aumento en las concentraciones de LDH y de bilirrubinas no conjugadas), la fragilidad osmótica de los eritrocitos se incrementa en forma característica. En estos individuos la esplenectomía es el tratamiento principal, que produce una mejoría en la supervivencia de los eritrocitos y aumento en las concentraciones de hemoglobina. (Hoffman, pp. 584-585)

http://bookmedico.blogspot.com

http://bookmedico.blogspot.com 338

10: Examen de práctica 3

BIBLIOGRAFÍA

American Psychiatric Association. Diagnostic and Statistical Manual of Mental Disorders, 4th ed. Text Revision. Washington, DC: American Psychiatric Association, 2000. Beckmann CRB, Ling FW, Laube DW, et al. Obstetrics and Gynecology, 4th ed. Baltimore, MD: Lippincot Williams & Wilkins, 2002. Braunwald E, et al. Harrison’s Principles of Internal Medicine, 15th ed. New York, NY: McGraw-Hill, 2001. Braunwald E. Heart Disease: A Textbook of Cardiovascular Medicine, 6th ed. Philadelphia, PA: W.B. Saunders, 2001. Cameron JL (ed.). Current Surgical Therapy, 8th ed. St. Louis, MO: C.V. Mosby, 2004. Cotran RS, Kumar V, Collins T. Robbins Pathologic Basis of Disease, 6th ed. Philadelphia, PA: W.B. Saunders, 1999. Cunningham FG, et al. Williams Obstetrics, 21st ed. New York, NY: McGraw-Hill, 2001. Hoffman R, et al. Hematology: Basic Principles and Practice, 3rd ed. New York, NY: Churchill Livingstone, 2000. Jones KL (ed). Smith’s Recognizable Pattern of Human Malformations, 5th ed. Philadelphia, PA: W.B. Saunders, 1997.

338

Jones KL (ed). Smith’s Recognizable Pattern of Human Malformations, 5th ed. Philadelphia, PA: W.B. Saunders, 1997. Kunimoto DY, et al. The Wills Eye Manual: Office and Emergency Room Diagnosis and Treatment of Eye Disease, 4th ed. Philadelphia, PA: Lippincott Williams & Wilkins, 2004. Larsen PR, et al. Williams Textbook of Endocrinology, 10th ed. Philadelphia, PA: W.B. Saunders, 2003. Ruddy S, et al. Kelley’s Textbook of Rheumatology, 6th ed. Philadelphia, PA: W.B. Saunders, 2001. Rudolph CD, Rudolph AM, Hostetter MK, Lister GE, Siegel NJ, et al. (eds.). Rudolph’s Pediatrics, 21st ed. New York, NY: McGraw-Hill, 2003. Sadock BJ, Sadock VA. Kaplan and Sadock’s Synopsis of Psychiatry, 9th ed. Philadelphia, PA: Lippincott Williams & Wilkins, 2003. Schwartz SI, Shires GT, Spencer FC, et al. (eds.). Principles of Surgery, 7th ed. New York, NY: McGraw-Hill, 1999. Townsend CM, Beauchamp RD, Evers BM, et al. (eds.). Sabiston Textbook of Surgery: The Biological Basis of Modern Surgical Practice, 17th ed. Philadelphia, PA: W.B. Saunders, 2004.

http://bookmedico.blogspot.com

http://bookmedico.blogspot.com CAPÍTULO 11

Examen de práctica 4 Preguntas Lea cuidadosamente cada pregunta en el orden en que se presenta. Después elija la mejor respuesta de las opciones mostradas. Más de una opción puede ser parcialmente correcta. Se debe elegir la MEJOR respuesta.

Situación I: consultorio/centro de salud Se valora a los pacientes en dos ubicaciones: en su consultorio, que se encuentra adyacente al hospital y en el centro de salud comunitario. Se atienden consultas de medicina familiar. La mayoría de los pacientes es de la propia práctica que acude para consultas regulares programadas. En ocasiones se valora a pacientes cuyo tratamiento primario corresponde a otro médico. Puede hacerse referencia a los registros médicos del paciente. Los pacientes conocidos pueden ser tratados por vía telefónica. Se deben responder las preguntas con respecto a la información que aparece en los medios de comunicación masiva, los cuales requerirán interpretación de las publicaciones médicas. Los departamentos de laboratorio y radiología tienen todos los servicios disponibles.

Preguntas 1 a 4 Un varón de raza negra de 19 años de edad acude a consulta para una revisión antes de participar en actividades deportivas. Desea ingresar al equipo de básquetbol de su colegio. Juega básquetbol cinco días por semana durante 1 o 2 h al día. En general se encuentra sano pero el día de hoy refiere molestias leves e intermitentes en el flanco derecho del abdomen. Niega fiebre, escalofríos, disuria o poliuria. No tiene antecedentes personales patológicos significativos de traumatismos o hematuria previa. Fuma una cajetilla de cigarrillos cada dos a tres días, bebe cuatro a seis cervezas durante los fines de semana y niega el consumo de drogas ilegales. La última dosis de vacuna contra difteria-tétanos (dT) fue a los 14 años de edad. La exploración física es normal.

Se solicita análisis de orina que muestra ocho eritrocitos por campo de alta resolución. En la revisión de su expediente se observa un análisis de orina realizado hace seis meses con seis eritrocitos por campo de alta resolución, pero nunca ha acudido para una consulta de revisión. 1. ¿Cuál es el paso más apropiado en ese momento? A) Observación. B) Repetir el análisis de orina en tres meses. C) Antibioticoterapia empírica para el tratamiento de la infección de vías urinarias. D) Urografía excretora. E) Radiografía simple de abdomen. 2. Durante esta consulta el médico debe: A) Asesorar al paciente con respecto a la prevención del dolor lumbar. B) Asesorar al paciente con respecto a la prevención del cáncer cutáneo. C) Proporcionar información con respecto al consumo de alcohol. D) Administrar un refuerzo de la vacuna dT. E) Recomendar al paciente la exploración frecuente de los testículos para la detección de cáncer testicular. 3. La principal causa de muerte en este grupo de edad es: A) Síndrome de inmunodeficiencia adquirida (sida) B) Homicidio. C) Suicidio. D) Accidente en vehículo motorizado. E) Enfermedad cardiovascular.

http://bookmedico.blogspot.com

http://bookmedico.blogspot.com 340

11: Examen de práctica 4

4. Cuando se pregunta al paciente por qué fuma, la respuesta más probable es: A) B) C) D) E)

Todos a mi alrededor fuman. Soy adicto. Me relaja. Tengo el hábito. Estoy aburrido.

Preguntas 5 y 6 Una mujer de 42 años de edad es enviada a la clínica después de haberse mudado a la ciudad. Refiere múltiples molestias, lo que incluye cefaleas intensas desde la infancia, dolor articular, dolor torácico atípico y lumbalgia crónica. También refiere diarrea intermitente e intolerancia inusual a los alimentos. Ha tenido períodos menstruales irregulares desde la menarquia y a menudo se marea y se tambalea al ponerse de pie. Se realiza una revisión general de su expediente médico, y se detectan citas frecuentes como paciente ambulatorio, diversas hospitalizaciones, numerosas pruebas y procedimientos y varias intervenciones quirúrgicas. Pese a las valoraciones amplias, no se ha establecido un diagnóstico definitivo ni se ha dado un tratamiento suficiente para atender de manera adecuada todos sus síntomas. La paciente tiene angustia evidente con respecto al sufrimiento y percibe que no se le pone atención. 5. ¿Cuál es el diagnóstico más probable? A) B) C) D) E)

Trastorno conversivo. Hipocondriasis. Trastorno doloroso. Trastorno de somatización. Trastorno somatiforme indiferenciado.

6. ¿Cuál es el método más útil en el tratamiento de la paciente? A) Hospitalización para estudio diagnóstico amplio. B) Comentar con la paciente la naturaleza psicológica de los síntomas. C) Tranquilizar a la paciente e informarle que al final se establecerá el diagnóstico. D) Envío de la paciente con varios especialistas. E) Citas regulares para vigilancia.

Preguntas 7 a 9 Una mujer caucásica de 60 años de edad acude a consulta por dolor en los hombros y en la cara externa de la cadera. Estos síntomas han persistido desde hace cuatro semanas y le han impedido acudir a sus clases de ejercicios aeróbicos en el deportivo local. Ha perdido 7.5 kg en las últimas cuatro semanas, pero niega síntomas articulares. La exploración física muestra que el arco de movimientos está conservado, no hay sinovitis en ninguna articulación pero hay dolor en hombros y cuello. La fuerza motora en las extremidades superiores e inferiores es normal. Las pruebas de laboratorio fueron positivas únicamente para la tasa de eritrosedimentación, con 99 mm/h, con resultado negativo en la prueba de anticuerpos antinucleares y cifras normales de cinasa de creatina. 7. Los datos del interrogatorio y exploración física sugieren uno de los siguientes diagnósticos: A) B) C) D) E)

Polimiositis. Hipotiroidismo. Polimialgia reumática. Osteoartritis de cuello y hombros. Fibromialgia.

8. ¿Cuál es el tratamiento médico más apropiado para esta paciente? A) Tratamiento de sustitución de hormona tiroidea. B) Analgésicos como paracetamol. C) Prednisona en dosis de 20 mg por día. D) Antidepresivos (amitriptilina, 50 mg/día). E) Relajantes musculares (ciclobenzaprina, 10 mg al día). 9. La paciente regresa a consulta cinco días después, comentando que aún persisten los síntomas. Además, reporta que ha tenido cefaleas y visión borrosa. Los estudios de laboratorio muestran tasa de eritrosedimentación de 100 mm/h. Estos síntomas adicionales sugieren uno de los siguientes diagnósticos:

http://bookmedico.blogspot.com

339

http://bookmedico.blogspot.com Preguntas: 4–14

A) B) C) D) E)

Artritis reumatoide. Arteritis de células gigantes. Lupus eritematoso sistémico. Hipotiroidismo. La paciente está deprimida y eso amplifica sus síntomas.

10. Una niña de seis meses de edad es llevada a consulta para revisión de niño sano. El peso y la talla se encuentran por debajo del porcentil 50, pero la niña está por lo demás sana. La niña tiene buena coloración y no hay datos positivos en la exploración física. En la auscultación se percibe un soplo intenso, áspero, soplante, holosistólico en el borde esternal izquierdo. ¿Cuál es la causa más probable de estas manifestaciones clínicas? A) B) C) D) E)

Comunicación interventricular. Tetralogía de Fallot. Transposición de grandes vasos. Tronco arterioso. Atresia pulmonar.

Preguntas 11 a 13 Una mujer previamente sana de 27 años de edad acude a consulta por cefaleas recurrentes de unos cuantos días de evolución. La paciente se alarmó cuando en fechas recientes visitó una feria de la salud en donde le informaron que su presión arterial se encontraba elevada. La paciente inició el consumo de anticonceptivos orales después del nacimiento de su último hijo, hace un año. En el año anterior incrementó su peso en 10 kg y desarrolló edema bilateral de tobillos que empeora por la noche. No ha menstruado en los últimos dos meses y esta mañana despertó con hemorragia transvaginal escasa. Desea saber qué hacer, porque en este momento de su vida “no puede sobrellevar un embarazo”. 11. ¿Cuál es el paso más apropiado en el tratamiento?

341

D) Hacer énfasis en que las cefaleas tal vez sean ocasionadas por estrés y que es probable que su peso se estabilice con el paso del tiempo. E) Inicio de fármacos diuréticos para controlar la presión arterial y reducir el edema. 12. ¿Cuál es una contraindicación absoluta para el consumo de anticonceptivos orales? A) B) C) D)

Diagnóstico de depresión mayor. Antecedente de migraña. Antecedente de oligomenorrea. Tabaquismo después de los 35 años de edad. E) Antecedente de anemia hemolítica. 13. ¿Cuál se considera un efecto adverso de los anticonceptivos orales? A) B) C) D) E)

Trombocitopenia. Cáncer ovárico. Colelitiasis. Infertilidad irreversible. Cáncer endometrial.

14. A una estudiante de 19 años de edad se le encuentran concentraciones elevadas de calcio sérico en una revisión programada. Hay antecedentes heredofamiliares de hipercalcemia, que no produce síntomas. El estudio diagnóstico revela aumento leve en las concentraciones de hormona paratiroidea con disminución en las concentraciones de fosfato sérico. Se cuantificó la excreción de calcio en orina de 24 h, la cual resultó baja. ¿Cuál es el diagnóstico correcto? A) B) C) D) E)

Hipercalcemia hipocalciúrica familiar. Hiperparatiroidismo primario. Hiperparatiroidismo secundario. Hiperparatiroidismo terciario. Cáncer óseo metastásico.

A) Interrupción de anticonceptivos orales y realizar una prueba de embarazo en orina. B) Prueba de embarazo en orina pero continuar con la administración de anticonceptivos orales. C) Tranquilizar a la paciente e informarle que los síntomas tal vez sean ocasionados por el inicio de la menstruación.

http://bookmedico.blogspot.com

http://bookmedico.blogspot.com 342

11: Examen de práctica 4

Preguntas 15 a 17 Un varón divorciado de 44 años de edad es enviado a la clínica para tratamiento de hipertensión e hipercolesterolemia. Al iniciar la consulta comenta “vine porque escuché que usted es el mejor médico. Los otros médicos no hicieron mucho por mí, de hecho, ellos no sabían qué hacer”. Más tarde admitió falta de apego terapéutico pues interrumpía las dosis del medicamento, pero disminuyó el consumo de sal y grasas. Se expresa muy mal de su jefe, de quien dice: “es un explotador... abusa de mí y no tengo tiempo para comer en forma apropiada”. Niega síntomas de depresión pero expresa que se siente solo y aburrido. Describe dificultades durante toda su vida con su temperamento y con frecuencia rompe objetos de valor cuando está enojado. No tiene relaciones estables pero admite conductas promiscuas repetitivas con mujeres, en cuyo caso compra joyería costosa para obtener favores sexuales de ellas. 15. ¿Cuál es el diagnóstico más probable? A) B) C) D) E)

Trastorno de evitación de la personalidad. Trastorno limítrofe de la personalidad. Trastorno de dependencia. Trastorno paranoide de la personalidad. Trastorno esquizoide de la personalidad.

16. El paciente comentado en la pregunta anterior muestra como tipo prominente de mecanismo de defensa: A) B) C) D) E)

Negación. Desplazamiento. Intelectualización. Desdoblamiento. Sublimación.

17. ¿Cuál es el medicamento que sería más apropiado como tratamiento de primera línea para estos síntomas? A) Clonazepam. B) Haloperidol. C) Litio. D) Sertralina. E) Ácido valproico.

A) Adolescente de 16 años con carcinoma papilar con metástasis a tres ganglios linfáticos cervicales. B) Varón de 42 años de edad con carcinoma papilar. C) Mujer de 52 años de edad con carcinoma folicular confinado a la cápsula tiroidea. D) Varón de 44 años de edad con un foco oculto de carcinoma medular que se encuentra de manera incidental después de una tiroidectomía subtotal. E) Mujer de 53 años de edad con carcinoma de células de Hurthle. 19. Una mujer de 36 años de edad acude a consulta solicitando una receta por anticonceptivos orales. ¿Cuál es el antecedente personal que constituye una contraindicación absoluta para la prescripción de anticonceptivos orales? A) B) C) D) E)

Hipertensión. Diabetes mellitus tipo 2. Epilepsia. Tabaquismo. Migraña menstrual (sin aura).

Preguntas 20 a 21 Se valora a una niña de cuatro meses de edad por pérdida persistente de peso. La madre comenta que la niña “come muy bien” pero no parece recuperar peso, además de que parece producir demasiada saliva. La niña fue producto de un embarazo a término sin antecedentes perinatales de importancia. En la exploración se le observa con buen estado general pero caquéctica, con disminución de la grasa corporal. Su peso y talla se encuentran por debajo del porcentil 3. La exploración física es básicamente normal. Los estudios de laboratorio incluyen lo siguiente: Na 135 Nitrógeno ureico sanguíneo 15 Análisis de orina:

K 5.2 Creatinina 0.9

Cl 111 Glucosa

Bicarbonato 16 79

Densidad urinaria: 1.010

PH 6.0

Sangre y proteínas ()

20. ¿Cuál es la explicación más probable para la falta de aumento de peso de esta niña?

18. ¿Cuál de los pacientes mostrados a continuación con cáncer tiroideo tiene el mejor pronóstico?

A) Estenosis pilórica. B) Intolerancia a la fórmula láctea. C) Estenosis duodenal.

http://bookmedico.blogspot.com

http://bookmedico.blogspot.com Preguntas: 15–25

D) Reflujo gastroesofágico. E) Acidosis tubular renal. 21. ¿Cuál es el estudio que con mayor probabilidad confirmará el diagnóstico? A) Serie esofagogastroduodenal. B) Endoscopia del tubo digestivo proximal. C) Tratamiento con bicarbonato de sodio para valorar el pH urinario. D) Tratamiento con metoclopramida. E) Tratamiento con fórmula elemental. 22. Una mujer de 55 años de edad acude a consulta con su médico internista por polaquiuria y disuria de dos meses de evolución. La semana anterior otro médico indicó trimetoprim/sulfametoxazol por una supuesta infección de vías urinarias. Los síntomas persisten y además la paciente refiere resequedad vaginal e irritación durante el coito. Estos problemas se han exacerbado desde que interrumpió el tratamiento de sustitución hormonal hace un año. ¿Cuáles son los datos que esperaría encontrar en la exploración? A) Secreción vaginal blanquecina, con grumos. B) Piel vulvar y mucosa vaginal delgadas y pálidas. C) Lesiones fungoides en el cuello uterino. D) Tampón retenido. E) Fístula vesicovaginal. 23. Con respecto a las categorías de medicamentos durante el embarazo de acuerdo a la Food and Drug Administration, ¿cuál de las siguientes aseveraciones es verdadera? A) Una categoría más alta indica el mayor nivel de riesgo de anomalías congénitas. B) En la categoría B se incluyen los fármacos que muestran riesgo de teratogénesis en estudios en animales, pero no existe tal riesgo en seres humanos. C) Los fármacos de la categoría C muestran evidencia consistente de riesgo en animales y seres humanos gestantes. D) Los fármacos de la categoría D conllevan un riesgo significativo de causar anomalías congénitas en mujeres que los toman y no se recomienda su uso durante el embarazo. E) Los fármacos de la categoría X siempre causan defectos congénitos.

343

24. Una mujer de 25 años de edad acude a consulta por galactorrea bilateral, fatiga y aumento de peso. Su menarquia ocurrió a los 12 años de edad y tuvo ciclos menstruales normales hasta los 20 años, cuando tomó píldoras anticonceptivas hasta hace 18 meses. Más tarde, ella tuvo un embarazo normal con parto vaginal hace seis meses; amamantó a su hijo por tres meses y reinició el consumo de píldoras anticonceptivas. También inició con ejercicios vigorosos para perder peso. Sin embargo, en los últimos tres meses ha sentido fatiga y ha incrementado 7.5 kg de peso. Ha notado calambres musculares durante el ejercicio, resequedad cutánea y pérdida persistente de cabello. Se ha sentido deprimida. En la exploración física la presión arterial (TA) es de 130/90 mmHg y la frecuencia cardíaca es de 60 latidos por minuto (lpm). La piel es áspera y los reflejos osteotendinosos profundos tardan en relajarse.

Exámenes de laboratorio

T4 total

4.5 (5–12 µg/dl)

Captación de T3 por resinas Índice de tiroxina libre TSH Prolactina

20% (25–35%) 0.9 (1.2–4.2) 35 (0.4–5 uU/ml) 40 (5–15 ng/ml)

¿Cuál es el paso más apropiado en el tratamiento? A) Resonancia magnética nuclear de la hipófisis. B) Gammagrafía tiroidea. C) Interrupción de las píldoras anticonceptivas orales y repetir los estudios en seis semanas. D) Tratamiento con levotiroxina. E) Prueba de estimulación con tratamiento de sustitución de hormonas tiroideas. 25. Una primigesta de 18 años de edad tuvo una mola hidatiforme que fue evacuada con legrado por aspiración. Antes del procedimiento, las concentraciones de gonadotropina crónica humana (hCG) eran de 125000 mUI/ml y el útero tenía el tamaño de un embarazo de 22 semanas. Después del legrado por aspiración las concentraciones séricas de hCG se midieron cada dos semanas. En las primeras dos semanas la concentración de hCG disminuyó a 25000 mUI/ml, pero se incrementó

http://bookmedico.blogspot.com

http://bookmedico.blogspot.com 344

11: Examen de práctica 4

a 32000 mUI/ml después de cuatro semanas. En la exploración física sólo se encuentra útero disminuido de consistencia. ¿Cuál es el paso más apropiado en el tratamiento en este momento? A) Repetir la dilatación y legrado. B) Medición de concentraciones de hCG una vez por semana. C) Tomografía computadorizada (CT) de cabeza, tórax, abdomen y pelvis. D) Histerectomía. E) Ecografía pélvica.

Situación II: pacientes hospitalizados El médico tiene privilegios para hospitalizar pacientes, lo que incluye servicios de pediatría y ginecología. En ocasiones el médico valora pacientes en la unidad de cuidados intensivos. Los pacientes posoperados por lo general se valoran en su habitación del hospital, a menos que se especifique que se revisan en la sala de recuperación posquirúrgica. Tal vez se llame al médico para valorar pacientes en la unidad psiquiátrica. Se dispone de una unidad de corta estancia donde podrían verse pacientes dados de alta el mismo día de la operación u hospitalizar a los pacientes para observación. También, el médico puede visitar pacientes en asilos, instituciones de cuidados por períodos prolongados y unidades de destoxificación.

27. Un varón de 42 años de edad con enfermedad de Crohn extensa fue sometido a una resección casi total del íleon. ¿Cuál es la deficiencia que con probabilidad será consecuencia de la operación? A) B) C) D) E)

Niacina. Tiamina. Vitamina B12. Vitamina C. Vitamina B6.

Preguntas 28 a 30 Una mujer de 55 años de edad acude con padecimiento actual de tres semanas de evolución con una tumoración en la mama, que se detectó durante una autoexploración mamaria. La tumoración es indolora y la paciente no ha detectado cambios cutáneos, secreción a través del pezón o retracción cutánea. No hay antecedentes familiares de cánceres mamario u ovárico. La paciente por lo demás está sana. En la exploración física se encuentra una tumoración de consistencia firme en el cuadrante superior externo de la mama de casi 2 cm de diámetro. Después de una mamografía y de la realización de pruebas de laboratorio fue sometida a biopsia excisional. En la figura 11-1 se muestra una microfotografía representativa del aspecto histológico de la tumoración.

26. Una mujer de 74 años de edad con antecedente de hipertensión e hipotiroidismo fue hospitalizada por equimosis fáciles, heces con resultado positivo en la prueba de guayaco y anemia (hemoglobina de 8.1 g/100 ml). Las pruebas de coagulación muestran prolongación del tiempo de tromboplastina parcial activada (aPTT) con tiempo de protrombina (PT) y recuento plaquetario normales. ¿Cuál es el siguiente paso en el diagnóstico de la enfermedad esta mujer? A) Endoscopia de tubo digestivo alto y bajo con toma de biopsia. B) Medición de las concentraciones de los factores de coagulación II, VII, IX y X. C) Verificar las concentraciones de factor VII. D) Medición de las concentraciones de los factores XI, VII, IX y VIII. E) Medición de nuevo de aPTT con una mezcla 1:1 con plasma normal.

FIG. 11-1 (Cortesía de Edison Catalano, MD.)

http://bookmedico.blogspot.com

http://bookmedico.blogspot.com Preguntas: 26–34

28. ¿Cuál es el diagnóstico de la tumoración palpable, cuyo aspecto histológico se muestra en la figura? A) Carcinoma ductal infiltrante mal diferenciado (grado III). B) Carcinoma ductal infiltrante de baja malignidad. C) Carcinoma ductal microinvasor, variante papilar. D) Hiperplasia ductal atípica. E) Carcinoma coloide (secretor de mucina). 29. ¿Cuál es el factor pronóstico de mayor importancia en el carcinoma de mama? A) B) C) D)

Presencia de receptores de estrógenos. Invasión a linfáticos. Anaplasia celular. Metástasis a los ganglios linfáticos regionales. E) Retracción del pezón. 30. ¿En cuál de los siguientes tumores mamarios se encontrará metástasis axilares? A) B) C) D) E)

Hiperplasia lobulillar atípica. Carcinoma lobulillar in situ. Hiperplasia ductal atípica. Adenoma tubular. Carcinoma ductal infiltrante.

31. Una paciente con diabetes mellitus mal controlada e insuficiencia renal crónica fue sometida a desbridamiento de una úlcera venosa necrótica. En el segundo día posoperatorio se observa hemorragia persistente y prolongación del tiempo de sangrado. ¿Cuál es la mejor intervención en este momento? A) Apósitos compresivos y administración subcutánea de vitamina K. B) Transfusión de plaquetas. C) Desmopresina intravenosa (DDAVP). D) Transfusión de plasma fresco congelado. E) Corrección del estado de hiperglucemia. 32. Durante el embarazo ocurren alteraciones fisiológicas en casi todos los aparatos y sistemas corporales. Con respecto a la sangre, ¿cuál de las siguientes aseveraciones es verdadera?

345

A) El volumen sanguíneo materno se incrementa en 100% hacia el final del embarazo. B) La masa de eritrocitos maternos se incrementa en 50% hacia el final del embarazo. C) Las demandas totales de hierro para un embarazo a término para satisfacer las necesidades de la madre y del feto son de 2500 mg. D) La administración de 30 mg de hierro por día en forma de complementos durante el embarazo previene de manera eficaz la deficiencia de hierro. E) El recuento plaquetario promedio en mujeres embarazadas y no embarazadas es prácticamente el mismo. 33. ¿Cuál de las siguientes es una indicación para resección de divertículo de Meckel encontrado de forma incidental durante una apendicectomía laparoscópica en un varón de 52 años de edad? A) Todo divertículo de Meckel que se encuentra en forma incidental debe ser extirpado. B) Ubicación a 35 cm o menos de la válvula ileocecal. C) Antecedente de hemorragia rectal. D) Presencia de cuello del divertículo de base ancha. E) Presencia de tejido firme en la base del divertículo.

Preguntas 34 y 35 34. Un varón de 34 años de edad fue sometido a una extirpación sin complicaciones de un adenoma paratiroideo. En el primer día posoperatorio el paciente refiere adormecimiento alrededor de los labios. ¿Cuál es la causa más probable de estos síntomas? A) Hipocalcemia secundaria a hipomagnesemia. B) Hipocalcemia por insuficiencia renal aguda. C) Hipocalcemia por “síndrome del hueso hambriento”. D) Hipocalcemia por lesión inadvertida del nervio laríngeo recurrente. E) Hematoma posoperatorio del cuello.

http://bookmedico.blogspot.com

http://bookmedico.blogspot.com 346

11: Examen de práctica 4

35. ¿Cuál es el tratamiento más apropiado? A) Administración oral de gluconato de calcio. B) Rehidratación intravenosa con solución salina isotónica. C) Administración de sulfato de magnesio por vía intravenosa. D) Hemotransfusión. E) Tranquilizar al paciente y vigilancia estrecha.

Situación III: sala de urgencias La mayoría de los pacientes valorados en este contexto es nueva para el médico, pero en ocasiones puede concertarse una cita en la sala de urgencias cuando un paciente conocido llama por teléfono al médico. Por lo general, los pacientes son valorados en situaciones de urgencia. También se dispone de una amplia gama de servicios sociales, lo que incluye intervención después de una violación, apoyo familiar y asistencia de seguridad por la policía local.

A) B) C) D) E)

Púrpura trombocitopénica inmunitaria. Púrpura de Henoch-Schönlein. Síndrome de Evans. Meningococemia. Síndrome hemolítico-urémico.

Preguntas 38 a 40 Una mujer de 78 años de edad es transferida desde un asilo al hospital por náuseas, vómito, distensión abdominal e incapacidad para evacuar desde hace tres días. En la exploración se le encuentra somnolienta y taquicárdica. Hay distensión abdominal, timpanismo y dolor difuso a la palpación de todo el abdomen. En la figura 11-2 se muestra la radiografía abdominal de la paciente.

36. A un paciente con fiebre de 38.9°C y antecedente de aneurisma de la aorta abdominal, se le realiza una CT en la que se observa la presencia de líquido alrededor del injerto. ¿Cuál es el tratamiento más apropiado? A) Antibióticos intravenosos y repetir la CT en 24 a 48 h. B) Drenaje del líquido acumulado mediante la colocación de un catéter guiado por CT. C) Exploración quirúrgica con retiro del injerto, irrigación y colocación de un nuevo injerto. D) Exploración quirúrgica con retiro del injerto y construcción de un injerto axilobifemoral. E) Exploración quirúrgica con desbridamiento y drenaje del líquido acumulado. 37. Una niña de siete años de edad es llevada a la sala de urgencias por fatiga y palidez. Fue tratada por su médico familiar hace unos cuantos días y se diagnosticó infección de vías respiratorias altas. En la exploración física se observa esplenomegalia y equimosis en las extremidades superiores. Los exámenes de laboratorio solicitados muestran recuento de leucocitos de 11000, hemoglobina de 6 g/100 ml, recuento de plaquetas de 40000. La prueba de Coombs es positiva. ¿Cuál es la causa más probable?

FIG. 11-2 (Reproducido con autorización de Chen MYM, Pope TL Jr, Ott DJ, Basic Radiology, New York, NY: Mc Graw-Hill, 2004, p. 220.)

38. Los datos radiológicos son compatibles con: A) B) C) D) E)

Íleo biliar. Pancreatitis. Colangitis. Neoplasia de intestino delgado. Apendicitis perforada.

http://bookmedico.blogspot.com

http://bookmedico.blogspot.com Preguntas: 35–46

39. La paciente se encuentra en riesgo de: A) B) C) D) E)

Colangitis ascendente. Perforación de intestino delgado. Perforación colónica. Síndrome de Ogilvie. Hemorragia masiva del intestino delgado.

40. ¿Cuál es la intervención más apropiada? A) Líquidos intravenosos, descompresión gástrica, antibióticos y vigilancia estrecha. B) Colangiopancreatografía endoscópica retrógrada con descompresión de la vía biliar. C) Colecistectomía. D) Exploración quirúrgica con enterotomía para aliviar la obstrucción intestinal. E) Exploración quirúrgica con enterotomía para aliviar la obstrucción, junto con colecistectomía. 41. ¿Cuál es la principal causa de muerte en Estados Unidos de lactantes entre el primero y duodécimo meses de vida? A) B) C) D) E)

Meningitis bacteriana. Cardiopatía congénita. Malformaciones congénitas. Envenenamientos accidentales. Síndrome de muerte súbita infantil.

Preguntas 42 y 43 Una mujer caucásica de 84 años de edad acude a la sala de urgencias con amaurosis súbita que inició hace varias horas. Tiene antecedentes de hipertensión y fibrilación auricular, las cuales están controladas. Refiere fatiga crónica y no come bien. 42. ¿Cuál es la primera prueba que debe solicitarse? A) Resonancia magnética nuclear de cráneo y órbitas con medio de contraste. B) Doppler carotídeo. C) Tasa de eritrosedimentación. D) Ecocardiograma. E) Biometría hemática completa con recuento diferencial de leucocitos y de plaquetas.

347

43. ¿Cuál es el dato que con mayor probabilidad se encontrará durante la exploración del fondo de ojo? A) B) C) D) E)

Hemorragias en la retina. Exudados de lípidos. Manchas de color rojo cereza. Muescas arteriovenosas. Exudados cotonosos.

44. Un varón de 62 años de edad acude a la sala de urgencias con dolor subesternal intenso después de un episodio de vómito intenso y prolongado. La radiografía de tórax muestra escaso aire en el mediastino. ¿Cuál es el tratamiento más apropiado en este momento? A) Endoscopia de tubo digestivo alto. B) Radiografía con trago de material de contraste hidrosoluble. C) Colocación de sonda nasogástrica y sonda pleural, ayuno y administración de antibióticos de amplio espectro. D) Toracotomía izquierda de urgencia. E) Toracotomía derecha de urgencia. 45. ¿Cuál es el agente infeccioso que con mayor probabilidad ocasionará secreción nasal intensa, sibilancias y tos en invierno a una niña de cuatro meses de edad? A) Estreptococo del grupo B. B) Estreptococo del grupo A. C) Virus sincitial respiratorio. D) Enterovirus no relacionado con polio. E) Haemophilus influenzae. 46. Un varón de 35 años de edad acude a la sala de urgencias con quemadura de tercer grado por flama en la extremidad superior derecha, cara anterior del tronco y lado derecho de la extremidad inferior, sin afección de genitales. ¿Cuál es el porcentaje del área quemada? A) B) C) D) E)

15% 25% 35% 45% 55%

http://bookmedico.blogspot.com

http://bookmedico.blogspot.com 348

11: Examen de práctica 4

Preguntas 47 a 50 Un varón de 50 años de edad acude a la sala de urgencias después de tres episodios de hematemesis grave. No hay antecedentes de episodios similares y al momento de la revisión la hemorragia no está activa. Los signos vitales son: temperatura 37.1°C, TA en decúbito dorsal de 98/72 mmHg, 75/50 mmHg en posición sedente, frecuencia cardíaca de 110 lpm, frecuencia respiratoria de 17 por minuto, saturación de oxígeno de 100% con aire ambiental. En la exploración física se le encuentra pálido, mareado y débil. Tiene ictericia de escleróticas, telangiectasias en la cara anterior del tórax, esplenomegalia y matidez a la percusión de todo el abdomen. El hematócrito inicial es de 30%. El tiempo de protrombina es de 31 s con una razón internacional normalizada (INR) de 3.1. El paciente niega el consumo de fármacos anticoagulantes. Se solicita la valoración por un gastroenterólogo. 47. Antes de realizar la endoscopia de tubo digestivo alto, el mejor paso inicial en el tratamiento de este paciente es: A) B) C) D)

Solicitar estudios serológicos para hepatitis. Lavado gástrico con sonda nasogástrica. CT para descartar enfermedad hepática. Administración intensiva de líquidos intravenosos. E) Corrección de la coagulopatía del paciente.

48. La endoscopia de tubo digestivo alto muestra múltiples varices grado 3/4 en el esófago. Sobre la várice más grande se observa un tapón de fibrina que parece estar roto con salida de sangre fresca a través de la mucosa. ¿Cuál es el paso inicial en el tratamiento de este paciente? A) Colocación de bandas esofágicas a través de endoscopia para el tratamiento de las varices. B) Escleroterapia de las varices esofágicas por endoscopia. C) Tratamiento quirúrgico para ligadura definitiva de las varices. D) Inicio de tratamiento con inhibidores de la bomba de protones. E) Colocación de una derivación portosistémica intrahepática transyugular por radiología intervencionista.

49. El paciente evoluciona bien y no presenta nuevos episodios de hemorragia en su hospitalización. ¿Cuál es el tratamiento adicional que puede administrarse en forma ambulatoria y que es útil para evitar la recurrencia de la hemorragia en esta paciente? A) Colocación de bandas esofágicas a través de endoscopia para el tratamiento de las varices. B) Escleroterapia de las varices esofágicas por endoscopia. C) Tratamiento quirúrgico para ligadura definitiva de las varices. D) Inicio de tratamiento con inhibidores de la bomba de protones. E) Colocación de una derivación portosistémica intrahepática transyugular por radiología intervencionista. 50. El paciente regresa seis meses después a la sala de urgencias luego de otro episodio de hematemesis grave. Se realiza endoscopia de tubo digestivo alto que muestra las varices gástricas con hemorragia por debajo de la unión esofagogástrica. La hemorragia es intensa y el paciente tiene inestabilidad hemodinámica. ¿Cuál es el siguiente paso en el tratamiento de este paciente? A) Colocación de bandas esofágicas a través de endoscopia como tratamiento de las varices. B) Escleroterapia de las varices esofágicas por endoscopia. C) Tratamiento quirúrgico para ligadura definitiva de las varices. D) Inicio de tratamiento con inhibidores de la bomba de protones. E) Colocación de una derivación portosistémica intrahepática transyugular por radiología intervencionista.

http://bookmedico.blogspot.com

http://bookmedico.blogspot.com Preguntas: 35–46

349

Respuestas y explicaciones 1. D) 2. C) 3. D) 4. C)

Explicación de las preguntas 1 a 4 Se define a la hematuria microscópica como la presencia de más de tres eritrocitos por campo de alta resolución en el estudio microscópico del sedimento urinario en dos de tres muestras de orina con recolección adecuada. El grado de hematuria no guarda relación con la gravedad de la enfermedad subyacente, y por tanto la hematuria debe considerarse como un síntoma de enfermedad grave hasta que se demuestre lo contrario (la observación es una opción incorrecta). El paso inicial para la valoración de la hematuria microscópica es el interrogatorio y exploración física amplios. La valoración de las vías urinarias es el siguiente paso recomendado y las opciones incluyen pielografía, ecografía y CT. (Grossfeld GD, et al. Asymptomatic microscopic hematuria in adults: Summary of the AUA best practice policy recommendations. Am Fam Physician 2001;63:1145-1154) Durante la consulta debe proporcionarse asesoría con respecto al abuso de alcohol. La U.S. Preventive Services Task Force (USPSTF) emitió una recomendación B (detección sistemática) para el asesoramiento conductual y detección con el fin de reducir el consumo de alcohol en adultos durante la valoración médica primaria. La USPSTF emitió una recomendación I (evidencia insuficiente) para la detección de cáncer cutáneo y lumbalgia, mientras que emitió una recomendación D (en contra) de la detección sistemática de cáncer testicular en adolescentes y adultos jóvenes asintomáticos. El paciente recibió su última dosis de vacuna dT a los 14 años de edad, hace cinco años, de forma que no es necesario administrar una nueva dosis de la vacuna. (USPSTF:www.preventiveservices. ahrq.gov) La principal causa de muerte en este grupo de edad son lesiones relacionadas con vehículos motorizados. La tasa de mortalidad para individuos entre 15 y 24 años de edad por lesiones

accidentales es de 38.7/100000, la mayor parte de los cuales es ocasionada por accidentes en vehículo motorizado. La segunda causa de muerte son otros traumatismos, entre los que se incluye el homicidio, suicidio y ahogamiento. (Maxcy-Rosenau-Last.Public Health and Preventive Medicine, 14th ed.) En un estudio transversal de 354 adolescentes del medio urbano, con edades entre 12 y 21 años se preguntó la razón por la que continuaban fumando. La respuesta m{s común fue “me relaja”, la cual se reportó en 73% de los casos. La segunda causa fue “tengo el h{bito”, 56%; “soy adicto”, 29%; “estoy aburrido”, 22% y “todos a mi alrededor fuman”, 17%. A fin de fomentar la interrupción del tabaquismo en esta población, es necesario realizar acciones dirigidas a disminuir el estrés percibido y la dependencia a la nicotina (Siqueira LM, Rolnitzky LM, Rickert VI. Smoking cessation in adolescents. Arch Pediatr Adolesc Med 2001;155:489-495) 5. D) 6. E)

Explicación de las preguntas 5 y 6 El trastorno conversivo se diagnostica por la presencia de síntomas neurológicos o déficit asociados con factores psicológicos o conflictos en su mayor parte inconscientes, sin la gama de síntomas que se observan en este caso. La hipocondriasis se caracteriza por preocupación, con el temor de tener una enfermedad grave, con base en una interpretación inadecuada de las sensaciones corporales. El trastorno doloroso implica angustia significativa o alteración funcional como consecuencia del dolor. Se cree que los factores psicológicos participan en el inicio, gravedad, empeoramiento o mantenimiento del dolor. La sintomatología no relacionada con el dolor, como la que muestra la paciente, no es una característica de este trastorno. El trastorno somatiforme indiferenciado se diagnostica cuando hay síntomas físicos inexplicados que no satisfacen los criterios para trastorno de somatización. Este caso representa un trastorno de somatización, que se caracteriza por múltiples síntomas o patrones

http://bookmedico.blogspot.com

http://bookmedico.blogspot.com 350

11: Examen de práctica 4

recurrentes, que inician antes de los 30 años de edad y que no se explican por completo por una enfermedad conocida (DSM-IV). Si a un paciente se le diagnostica trastorno de somatización, la hospitalización para estudio diagnóstico amplio y las valoraciones por diversos especialistas pueden reforzar el proceso de somatización y empeorar los síntomas. Es probable que confrontar al paciente origine ira y cause incremento de los síntomas somáticos. Informar al paciente que se establecerá el diagnóstico es impreciso, confuso y puede por último dar origen a una demanda legal. El tratamiento más apropiado para estos individuos es programar consultas regulares, breves, dirigidas a la valoración de nuevos síntomas físicos. Las pruebas de laboratorio, estudios y procedimientos serán de naturaleza limitada. La exploración física debe tranquilizar al paciente en el sentido de que ha sido revisado con meticulosidad. Después de que se fortalezca la relación médico-paciente tal vez esté indicada la valoración por un psiquiatra para ayudar a disminuir el uso de los servicios de salud y para el tratamiento de la psicopatología concomitante (Synopsis, p. 647). 7. C) 8. C) 9. B)

Explicación de las preguntas 7 a 9 La polimialgia reumática es un síndrome inflamatorio de individuos en edad avanzada que se caracteriza por dolor y rigidez en las cinturas escapular y pélvica. Los pacientes por lo común tienen más de 50 años de edad, presentan síntomas bilaterales y la tasa de eritrosedimentación con frecuencia se encuentra por arriba de 40 mm/h. La ausencia de debilidad muscular y las concentraciones normales de creatinfosfocinasa son factores en contra de una posible polimiositis. El hipotiroidismo puede causar dolor musculoesquelético, pero no se asocia con tasas de eritrosedimentación de 99 mm/h o pérdida de peso (por lo común hay aumento de peso). La osteoartritis de cuello y hombros muestra anomalías en el arco de movimientos, acompañada de dolor; este paciente únicamente tiene dolor muscular a la palpación. La fibromialgia es un síndrome doloroso no inflamatorio ni autoinmunitario difuso, de causa desconocida, con inicio

entre los 35 y 40 años de edad, con puntos dolorosos característicos (es necesario que se encuentren 11 de los 18 puntos por al menos tres meses y que se encuentren por arriba y por abajo de la cintura). Dada la edad avanzada de esta paciente (60 años), la presencia de síntomas de un mes de evolución y datos de enfermedad inflamatoria sistémica (tasa de eritrosedimentación de 99 mm/h), es poco probable el diagnóstico de fibromialgia. La prednisona en dosis de 10 a 20 mg/día suele ocasionar respuesta rápida y espectacular en casos de polimialgia reumática. La mayoría de los pacientes mejora en forma significativa en el lapso de uno o dos días y la tasa de eritrosedimentación disminuye en forma estable. La medición de hormona liberadora de tirotropina (TRH) es inapropiada si los síntomas de este paciente fueron ocasionados por hipotiroidismo. Los analgésicos no son de utilidad para la inflamación de la polimialgia reumática (indicada por el incremento en la tasa de eritrosedimentación). Aunque esta paciente podría tener depresión, el incremento en la tasa de eritrosedimentación es un argumento en contra de dicho diagnóstico y es poco probable que el uso de relajantes musculares para enfermedades inflamatorias sistémicas sea de utilidad. La arteritis de células gigantes (o temporal) ocurre en forma simultánea o secuencial en pacientes con polimialgia reumática. Los síntomas incluyen cefalea, claudicación de la mandíbula, trastornos visuales y dolor a la palpación de la piel cabelluda. El hecho de que la prednisona (con dosis de 20 mg/día) no mejore de manera significativa los síntomas o normalice la tasa de eritrosedimentación, sugiere la presencia de arteritis temporal y es indicación para biopsia de la arteria temporal. Es poco probable la artritis reumatoide porque no hay datos de sinovitis en las articulaciones. Es posible la presencia de lupus eritematoso sistémico, pero es altamente improbable en un paciente con títulos negativos de anticuerpos antinucleares. Los síntomas de este paciente no son típicos de hipotiroidismo (ausencia de debilidad muscular, pérdida de peso en lugar de aumento de peso e incremento en la tasa de eritrosedimentación ~100 mm/h). No es prudente asumir que la paciente se encuentra deprimida y que está amplificando sus síntomas, con base en la evidencia de los exámenes de laboratorio que apoyan inflamación sistémica (tasa de eritrosedimentación ~100 mm/h). (Ruddy, pp. 1155-1162)

http://bookmedico.blogspot.com

349

http://bookmedico.blogspot.com Respuestas: 7–17

10. A) Por lo común las cardiopatías congénitas cianóticas ocasionan cortocircuito de derecha a izquierda. Esto evita el paso de sangre a través de la circulación pulmonar, con ausencia de intercambio gaseoso. La tetralogía de Fallot, transposición de los grandes vasos, tronco arterioso y atresia pulmonar son causas de cardiopatías congénitas cianóticas. El niño comentado en la pregunta no tiene una lesión cianótica y de las cardiopatías enumeradas, sólo la comunicación interventricular ocasiona cortocircuito de izquierda a derecha. (Fleisher and Ludwig, pp. 187-191) 11. A) 12. D) 13. C)

Explicación de las preguntas 11 a 13 Lo más probable es que esta paciente tenga incremento de la presión arterial como consecuencia del consumo de anticonceptivos orales. Ésta es la causa más común de hipertensión secundaria y debe interrumpirse la administración de estos medicamentos. Además, ha desarrollado otros efectos adversos por el consumo de anticonceptivos orales, lo que incluye aumento de peso y edema. Dada la presencia de síntomas al momento de la valoración, es razonable asumir que la presión arterial está elevada y por tanto debe medirse durante esta consulta. La hemorragia transvaginal y la amenorrea apuntan a un posible embarazo. Por tanto, se debe realizar prueba de embarazo en orina e interrumpir de inmediato los anticonceptivos orales hasta que se descarte un posible embarazo. La única contraindicación absoluta de las opciones mencionadas es el tabaquismo después de los 35 años de edad, aunque el tabaquismo intenso a cualquier edad se considera una contraindicación relativa. Con excepción de la anemia hemolítica, el resto de las opciones se consideran contraindicaciones relativas para prescribir anticonceptivos orales. La única anemia hemolítica que se considera contraindicación absoluta es la drepanocitosis, por incremento en el riesgo de trombosis. La colelitiasis es un efecto secundario establecido de los anticonceptivos orales. Otros efectos secundarios incluyen enfermedad tromboembólica, hipertensión, aumento de peso, retención de líquidos, edema, intolerancia a la

351

glucosa, cefaleas, melasma y náuseas. El riesgo de desarrollar cáncer endometrial y ovárico puede reducirse con el uso de anticonceptivos orales. La infertilidad temporal inducida por los anticonceptivos orales puede corregirse con la interrupción de éstos. La infertilidad prolongada después de interrumpir los anticonceptivos orales es indicación para estudio diagnóstico en busca de otras causas. No se espera que la trombocitopenia sea un efecto secundario del consumo de anticonceptivos orales. (Larsen, pp. 672-691) 14. A) La hipercalcemia hipocalciúrica familiar es un trastorno familiar benigno de hipercalcemia, poco frecuente, que se caracteriza por hipercalcemia asintomática o con síntomas leves. Es un trastorno que se hereda con rasgo autosómico dominante y en el que las glándulas paratiroides por lo general tienen un tamaño normal. La base para el desarrollo de hipercalcemia hipocalciúrica familiar parece ser una mutación en el gen de los receptores sensibles al calcio, que regula el punto de ajuste de la glándula paratiroides y que modula la concentración de calcio extracelular. El trastorno puede confundirse con hiperparatiroidismo primario porque en ambos trastornos las concentraciones de calcio sérico y de hormona paratiroidea se encuentran elevadas con disminución de las concentraciones séricas de fosfato. El diagnóstico diferencial se hace al obtener una medición de la excreción de calcio en orina de 24 h. En pacientes con hipercalcemia hipocalciúrica familiar la excreción de calcio en orina es baja, en tanto que en el hiperparatiroidismo primario las concentraciones se encuentran elevadas. Es importante la diferenciación porque los pacientes con hiperparatiroidismo primario se benefician de la intervención quirúrgica, a diferencia de los individuos con hipercalcemia hipocalciúrica familiar (Cameron, p. 602) 15. B) 16. D) 17. D)

Explicación de las preguntas 15 a 17 El trastorno de evitación de la personalidad se caracteriza por un patrón de inhibición en situaciones sociales y sentimientos de incomodidad. Estos individuos con probabilidad se culpan a sí mismos por sus dificultades en vez de criticar a

http://bookmedico.blogspot.com

http://bookmedico.blogspot.com 352

11: Examen de práctica 4

otras personas, como lo ilustra el caso de la pregunta. Los pacientes con trastorno dependiente de la personalidad muestran una conducta con la necesidad intensa de ser cuidados, lo que conduce a una conducta de sumisión. Es poco probable que adquieran una conducta hostil o que sufran labilidad emocional como en el ejemplo mencionado. Los individuos paranoides sufren desconfianza crónica y sospechan de otras personas. Asumen que los demás individuos desean lesionarlos. No participan en numerosas relaciones ni admiten sentimientos de soledad como se observa en el paciente del caso clínico. El trastorno esquizoide de la personalidad se diagnostica en pacientes con un patrón marcado de desprendimiento y expresión restringida durante las relaciones interpersonales. Los individuos esquizoides no buscan relacionarse con otras personas, prefieren las actividades solitarias. No tienen conductas tan explosivas como la del paciente mencionado en el caso clínico. El ejemplo antes citado representa a un varón con un trastorno limítrofe de la personalidad, que se caracteriza por inestabilidad de las relaciones personales, afectos e imagen personal, así como de impulsividad, problemas para el control de la ira y sensación de vacío (DSM-IV). La negación se clasifica como un mecanismo de defensa primitivo o psicótico, en el cual los individuos evitan percibir la realidad externa al negar los hechos. El desplazamiento se considera un mecanismo de defensa neurótico, que se caracteriza por desplazar los sentimientos de una persona o situación a otra persona o situación y que provocan menos angustia. La intelectualización es otro proceso defensivo de tipo neurótico, que utiliza un procesamiento intelectual para evitar la experiencia afectiva. La sublimación es un mecanismo de defensa maduro, en el que una persona dirige su impulso hacia una conducta socialmente aceptable. El paciente presentado en la pregunta muestra un mecanismo defensivo de separación. Es un mecanismo de defensa primitivo que suele observarse en trastornos graves de la personalidad, en especial en el trastorno limítrofe de la personalidad. En la separación, los pacientes dividen los sentimientos, personas y conductas en categorías de “bueno y malo” en vez de enfrentar ambivalencias. (Synopsis, pp. 207-208, 802) Las benzodiazepinas, como el clonazepam, pueden ser de utilidad para el tratamiento de la ansiedad y agitación en individuos con trastorno limítrofe de la personalidad, pero no se consideran tratamientos de primera línea por la falta de datos, riesgo de abuso y la posibilidad de empeo-

rar los trastornos conductuales. Los antipsicóticos como el haloperidol pueden ser beneficiosos para el control de la ira y para las lesiones infligidas por el propio individuo, pero los efectos son inespecíficos; éstos no se consideran para la monoterapia por los efectos secundarios significativos, en especial el riesgo de discinesia tardía. Los estabilizadores del estado de ánimo como el litio y el ácido valproico en ocasiones se utilizan para el tratamiento de la agresión impulsiva y para la inestabilidad emocional que es común en pacientes con trastorno limítrofe. No obstante, existe una preocupación legítima del uso de litio en estos pacientes impulsivos, a menudo suicidas, dada su toxicidad en casos de sobredosis. El ácido valproico es más seguro en caso de sobredosis, pero hay pocos datos que apoyen su eficacia en esta población de pacientes. Los antidepresivos en general, en especial los inhibidores de la recaptación de serotonina, como sertralina, han mostrado su eficacia para el tratamiento de los síntomas de inestabilidad del estado de ánimo con impulsividad, agresión, hostilidad y mutilación en individuos con trastorno limítrofe de la personalidad. Éstos fármacos son los medicamentos más estudiados y son muy seguros en casos de sobredosis. Estos factores favorecen su uso como farmacoterapia de primera línea en dichos pacientes, aunque a la fecha la FDA no ha probado tales fármacos para el tratamiento de esta enfermedad. (Practice Guideline for the treatment of Patients with Borderline Personality Disorder, 2001, APA) 18. A) El cáncer tiroideo se clasifica como papilar, folicular, de células de Hurtle, medular o anaplásico. De todos los subtipos, el carcinoma papilar tiene el mejor pronóstico general. Además del subtipo de cáncer, hay otros indicadores importantes del pronóstico, lo que se conoce como escala AGES. Dicha escala toma en consideración la edad del paciente, grado histopatológico del tumor, extensión de la enfermedad y tamaño del tumor (Age, Grade, Extent, Size). Otros factores importantes incluyen la presencia de metástasis a distancia, extensión de la resección quirúrgica original y la presencia de invasión extratiroidea. De todos los factores, la edad es la de mayor importancia (edad 16 semanas antes de la evacuación; 2) aumento de volumen de ambos ovarios antes de la evacuación (quistes tecaluteínicos); 3) edad >40 años; 4) concentraciones de hCG superiores a 100000 mUI/ml antes de la evacuación del útero; 5) antecedente de mola hidatiforme; 6) complicaciones del embarazo molar, entre las que se incluyen toxemia, hipertiroidismo y embolización con tejido trofoblástico. Las indicaciones para el tratamiento del tumor trofoblástico posmolar incluyen: 1) concentraciones estables de hCG en tres mediciones consecutivas, 2) aumento en las concentraciones de hCG en dos mediciones consecutivas, 3) concentraciones de hCG >20000 mUI/ml cuatro semanas o más después de la evacuación, 4) detección de metástasis o 5) diagnóstico histopatológico de coriocarcinoma. Después de la evacuación de un embarazo molar rara vez está indicado repetir la dilatación y legrado; no es curativo en casos de GTN y se acompaña de aumento en el riesgo de perforación uterina. (Diagnosis and management of gestational trophoblastic neoplasia. Best Pract Res Clin Obstet Gynaecol 2003;17(6):893-903) 26. E) El paso inicial en la identificación de la causa de la prolongación de los tiempos de tromboplastina parcial y protrombina es establecer si la adición de plasma corrige el déficit. Si no se corrige la prolongación de estos tiempos con la adición de 50% de plasma normal, indica la presencia de un inhibidor específico (contra uno de los factores de la coagulación) o inespecífico (p. ej., anticoagulante lúpico). Si después de una mezcla 1:1, se corrigen el PT o aPTT, la causa más probable es la deficiencia de factores de coagulación. El inhibidor más común en individuos sin hemofilia es una sustancia contra el factor VIII, como en el caso de esta mujer. (Hoffman, p. 1843) 27. C) La porción distal del intestino delgado (íleon) es el sitio de absorción de las vitaminas liposolubles (vitaminas A, D, E, y K) así como de vitamina B12. Esta última se une al factor intrínseco, una glucoproteína secretada por las células parietales del fondo y cuerpo del estómago. Receptores específicos en la porción terminal del íleon captan el complejo de factor intrínseco-vitamina B12. La deficiencia de vitamina B12 ocasiona anemia megaloblástica. Este paciente necesita la aplicación mensual de inyecciones de vitamina B 12. (Shwartz, p. 1255)

http://bookmedico.blogspot.com

http://bookmedico.blogspot.com Respuestas: 24–32

28. A) 29. D) 30. E)

Explicación de las preguntas 28 a 30 La microfotografía muestra células tumorales malignas que infiltran entre los haces de colágeno y en ciertas áreas focales, con la formación de “nidos” sólidos. El tumor muestra un alto grado de anaplasia con núcleos prominentes y citoplasma escaso. La formación de estructuras ductales es mínima y, por las razones antes mencionadas, es un tumor mal diferenciado (grado III). El carcinoma de mama afecta a una de cada nueve mujeres estadounidenses y 33% de dichas pacientes fallece a causa de la enfermedad. El cáncer de mama rara vez se observa antes de los 25 años de edad, excepto en personas con predisposición familiar. Los antecedentes heredofamiliares son un factor de riesgo para el desarrollo de cáncer mamario en 5 a 10% de los casos. Se han implicado a dos genes en la herencia del cáncer mamario, BRCA-1 y BRCA-2; sin embargo, menos de 20% de las mujeres con antecedentes heredofamiliares de cáncer mamario porta estos genes. La mastopatía proliferativa (hiperplasia ductal atípica) también se acompaña con incremento en el riesgo de desarrollar cáncer mamario. Otros factores que se acompañan de aumento en el riesgo de carcinoma de mama incluyen edad avanzada, carcinoma en la mama contralateral o cáncer endometrial, exposición a radiación, duración de la edad reproductiva, paridad, obesidad y los estrógenos (aunque son cuestionables). Los carcinomas invasores o ductales infiltrantes son la principal variedad de carcinomas (70 a 80%). La mayor parte de estos cánceres muestra un incremento marcado en el tejido fibroso del estroma, lo que le da una consistencia pétrea. En el examen histopatológico el tumor consiste de células malignas que forman cordones, nidos sólidos y túbulos. El aspecto citológico de estos tumores varía desde baja malignidad a alta malignidad. (Cotran, 6th ed., pp. 1104-1111) El pronóstico de cáncer mamario se relaciona directamente con su etapa, utilizando el sistema TNM. En ausencia de metástasis a distancia, la etapa del tumor se establece por el tamaño del tumor primario y la presencia o ausencia de linfadenopatía regional. Otros factores pronósticos, como la presencia de invasión linfática o el estado

355

de los receptores hormonales, puede influir en la supervivencia, pero no está claro si añaden información a la estadificación histopatológica. La hiperplasia lobulillar atípica, hiperplasia ductal atípica y el adenoma tubular son enfermedades mamarias benignas y no causan adenopatías. El carcinoma lobulillar in situ, por definición, no es un cáncer invasor y por tanto no causa adenopatía axilar. No obstante, casi 30% de las pacientes sometidas a ablación de neoplasia lobulillar pueden desarrollar carcinoma invasor en los siguientes 15 a 20 años, por lo general carcinoma ductal infiltrante. En las opciones enumeradas, sería de esperarse que sólo el carcinoma ductal infiltrante cause metástasis axilares. (Harrison’s Principles of Internal Medicine, 15th ed., pp. 574-577) 31. C) La prolongación del tiempo de sangrado es la característica distintiva de las enfermedades plaquetarias. La alteración en la adhesión plaquetaria se observa en casos de insuficiencia renal aguda y crónica. No está claro el mecanismo, aunque los datos implican que las toxinas producidas en la uremia ocasionan desagregación del factor VIII, que es necesario para la función adecuada de las plaquetas. DDAVP (1-desamino-8D-arginina vasopresina) es un derivado sintético de la vasopresina y su administración ocasiona incremento dependiente de la dosis del factor VIII por las células endoteliales. Por tanto, se utiliza para el control de la hemorragia posoperatoria en pacientes con uremia. Otro método consiste en la transfusión de crioprecipitados, que es útil porque contiene altas concentraciones de factor VIII. Por último, tal vez está indicada la corrección del estado urémico con diálisis. La transfusión de plaquetas no corrige el trastorno, porque serán desactivadas por las toxinas circulantes. La vitamina K se utiliza para corregir la hemorragia causada por trastornos en la vía extrínseca de la coagulación. El plasma fresco congelado no contiene concentraciones lo suficientemente elevadas de factor VIII. La hiperglucemia no se relaciona con la tendencia hemorragípara. (Jarrell BE, et al. National Medical Series for Independent Study, Surgery, 4th ed. Philadelphia, PA: Lippincott Williams & Wilkins, 2000, pp. 130-131; Schwartz, p. 82) 32. D) Los cambios fisiológicos en los aparatos cardiovascular y circulatorio son importantes y pueden afectar una enfermedad materna de fondo. El volumen sanguíneo se incrementa en 40 a 50% hacia el término del embarazo, y la masa de

http://bookmedico.blogspot.com

http://bookmedico.blogspot.com 356

11: Examen de práctica 4

eritrocitos se incrementa en 20 a 25%; esta diferencia se conoce como anemia fisiológica del embarazo. El consumo total de hierro en el embarazo a término para satisfacer las necesidades maternas, fetales y placentarias es de 1000 mg. Esto puede satisfacerse con las reservas preexistentes de hierro en mujeres con buenas reservas, pero existe el riesgo de agotamiento de las mismas, lo que da origen a anemia puerperal. El agotamiento de las reservas de hierro puede evitarse con la administración de 30 mg de hierro elemental por día, que se obtiene al tomar una tableta de gluconato ferroso de 325 mg/día. (Romslo I, Haram K, Sagan N, et al. Iron requirement in normal pregnancy is assessed by serum ferritin, serum transfer and saturation and erythrocyte protoporphyrin determinations. Br J Obstet Gynecol 1983;90:101) 33. E) El divertículo de Meckel es el divertículo verdadero más común del tubo digestivo; es un trastorno congénito ocasionado por el cierre incompleto del conducto onfalomesentérico o vitelino. Por lo común se encuentra a menos de 60 cm de la válvula ileocecal en el borde antimesentérico del íleon. Un divertículo de Meckel puede ser asintomático y encontrarse de manera incidental durante la exploración quirúrgica. En tal caso, la resección está indicada únicamente si hay signos de posible complicación, como la presencia de tejido firme palpable en la base. Esto representa la presencia de tejidos gástrico o pancreático heterotópicos, que pueden dar origen a úlcera y hemorragia. Si existe una comunicación o adherencia entre el divertículo de Meckel y la cicatriz umbilical está indicada la resección para evitar el riesgo de rotación intestinal, vólvulo y obstrucción alrededor de la adherencia. Por último, la resección se valora si el cuello del divertículo de Meckel es estrecho, porque puede ocasionar obstrucción a futuro por un fecalito, dando origen a diverticulitis y perforación. La morbilidad por la ablación incidental puede ser de hasta 12%, y por tanto se limita a las indicaciones antes mencionadas. (Schwartz, p. 1249) 34. C) 35. A)

Explicación de las preguntas 34 y 35 El síndrome de hueso hambriento se refiere a la hipocalcemia después de la corrección quirúrgica de hiperparatiroidismo en pacientes con enferme-

dad grave y prolongada, porque el calcio sérico es retirado con rapidez de la circulación hacia el hueso. Los síntomas por lo general se observan en las primeras 24 a 48 h después de la paratiroidectomía, cuando las concentraciones séricas de calcio alcanzan su punto más bajo. Los síntomas precoces incluyen parestesias y hormigueo en la región peribucal, dedos de las manos o de los pies. Los síntomas avanzados incluyen nerviosismo, ansiedad y aumento de la transmisión neuromuscular, evidenciado por signos positivos de Chvostek y Trousseau, espasmo carpopedal e hiperreflexia osteotendinosa. En casos graves se observa prolongación del intervalo QT en el electrocardiograma. Los pacientes con síntomas o signos de hipocalcemia requieren tratamiento. En pacientes con síntomas graves el tratamiento inicia con gluconato de calcio por vía intravenosa. Los pacientes con síntomas leves pueden recibir calcio por vía oral en forma de lactato, carbonato o gluconato de calcio. Si persiste la hipocalcemia pese a los complementos de calcio, tal vez se necesite tratamiento adicional con vitamina D. El tratamiento con complementos de calcio y vitamina D se continúa hasta que las concentraciones de calcio regresan a cifras normales. (Schwartz, p. 1701) 36. D) La tríada de fiebre, acumulación de líquido intraabdominal y antecedente de cirugía de injerto abdominal indica la aparición de infección en el injerto. El microorganismo aislado con mayor frecuencia es Staphylococcus aureus. Es una complicación poco frecuente pero con alta morbilidad y cifras de mortalidad de hasta 36%. La infección puede ocasionar con rapidez septicemia, choque hemorrágico y embolización séptica. El tratamiento estándar es la detección precoz y extirpación quirúrgica del injerto infectado con cierre primario de la aorta y creación de una reconstrucción extraanatómica, principalmente una derivación axilofemoral. Este tipo de derivación conlleva sus propias complicaciones, lo que incluye riesgo de pérdida de las extremidades, rotura del muñón aórtico e isquemia pélvica. (Am J Surg 1999;178(2):136-140) 37. C) El síndrome de Evans es la combinación de púrpura trombocitopénica inmunitaria y anemia hemolítica autoinmunitaria. Los factores desencadenantes de estos trastornos se desconocen. Los niños con síndrome de Evans tienen anemia con reacción de Coombs positiva con varios niveles de trombocitopenia. Está indicada la administración

http://bookmedico.blogspot.com

http://bookmedico.blogspot.com Respuestas: 33–44

de corticoesteroides para controlar la hemólisis aguda o la trombocitopenia. (Rudolph, p. 1546) 38. A) 39. B) 40. D)

357

acuesten sobre su espalda o de costado (no sobre el abdomen). La meningitis bacteriana es una enfermedad devastadora, pero rara vez es letal cuando se da tratamiento. La muerte por malformaciones congénitas por lo general ocurre en el primer mes de vida neonatal. (Rudolph, pp. 1935-1936) 42. C)

Explicación de las preguntas 38 a 40 El íleo biliar causa 1 a 2% de las obstrucciones mecánicas de intestino delgado. Es ocasionado por la erosión de un cálculo impactado a través de la pared de la vesícula biliar hacia la pared duodenal. El cálculo avanza a través del intestino delgado (a través de la pared duodenal) y más tarde el cálculo pasa al intestino delgado. El tamaño del cálculo es de importancia, porque aquellos menores de 2 cm por lo general son evacuados, en tanto que los de mayor tamaño pueden causar obstrucción. Cuando ésta ocurre, por lo general sucede en el íleon terminal, que es la porción más estrecha del intestino delgado. La obstrucción ocasiona grandes pérdidas de líquido y anomalías electrolíticas. Hay edema, ulceración y por último necrosis de la pared intestinal con perforación. Es poco frecuente hacer el diagnóstico en el preoperatorio. No obstante, el estudio radiográfico es diagnóstico si se observa aire en la vía biliar aunado a obstrucción del intestino delgado. En 20% de los casos se observan cálculos radioopacos. El tratamiento definitivo es la intervención quirúrgica urgente. Se realiza exploración quirúrgica y se realiza un corte sobre el intestino delgado (enterotomía) justo proximal al sitio en que se encuentra el cálculo. Éste se extrae y se realiza cierre primario del intestino. El prolongar la intervención quirúrgica para realizar una colecistectomía en un paciente anciano, grave incrementa el riesgo quirúrgico, y por lo general no está indicado. (Schwartz, pp. 1450-1452) 41. E) El síndrome de muerte súbita infantil (SIDS) es la causa más común de muerte en niños entre uno y 12 meses de edad. En el primer mes de vida la causa más común de muerte son las complicaciones perinatales (p. ej., premadurez). El diagnóstico de SIDS requiere el análisis de la historia clínica, investigación del sitio del fallecimiento y la autopsia. La tasa de SIDS ha disminuido en forma espectacular con el inicio de la campaña “dormir de espaldas”, que recomienda que los lactantes se

43. C)

Explicación de las preguntas 42 y 43 Esta paciente tiene síntomas compatibles con oclusión de la arteria central de la retina. Este trastorno por lo común causa amaurosis aguda, indolora y unilateral. En ocasiones se asocia con antecedente de amaurosis fugaz. Los signos característicos son las manchas de color rojo cereza en el centro de la mácula y opacificación superficial o coloración blanquecina de la retina en el polo posterior. Las causas comunes de este trastorno incluyen embolia desde las cavidades cardíacas o desde la arteria carótida, trombosis, arteritis de células gigantes u otras colagenopatías y estados de hipercoagulabilidad. Esta paciente tiene fibrilación auricular, lo que incrementa el riesgo de enfermedad embólica y por tanto la prueba más importante es la tasa de eritrosedimentación, la cual ayudará a establecer si la causa es embólica o está relacionada con arteritis de células gigantes. Si hay un incremento notable en la tasa de eritrosedimentación se inician de inmediato esteroides en dosis elevadas. Las otras pruebas enumeradas son parte del estudio diagnóstico posterior, pero no son tan importantes para establecer si se requiere tratamiento urgente. (Wills Eye Manual, Section 11.5) 44. B) La perforación esofágica tiene mal pronóstico, en especial si el diagnóstico se retrasa. Cuando se sospecha se solicitan de inmediato radiografías urgentes posteroanterior y laterales de tórax. Los datos positivos incluyen enfisema en el mediastino, ensanchamiento del mediastino como consecuencia del edema, neumotórax y derrame pleural; sin embargo estas manifestaciones pueden aparecer después de varias horas. El diagnóstico se confirma con un esofagograma con medio de contraste hidrosoluble. Si dicho estudio es negativo se puede realizar un estudio con pequeñas cantidades de bario. La presencia de bario en el mediastino puede dar origen a mediastinitis grave. (Cameron, p. 8)

http://bookmedico.blogspot.com

http://bookmedico.blogspot.com 358

11: Examen de práctica 4

45. C) En época de invierno, más de 90% de los niños menores de tres años de edad sufren infección por virus sincitial respiratorio, que puede causar bronquiolitis en lactantes y que en ocasiones causa hipoxia. La mayoría de los lactantes sanos no necesita hospitalización. Los factores de riesgo para complicaciones de infección por virus sincitial respiratorio incluyen displasia broncopulmonar, cardiopatías congénitas y premadurez. (Red Book, pp. 523-528) 46. D) Hay dos métodos utilizados con frecuencia para calcular el área de superficie corporal quemada: el método de la palma y la regla de los nueve. El método de la palma se basa en el hecho de que la palma del paciente corresponde a casi 1% de su superficie corporal total. Este método es particularmente útil en personas con quemaduras aisladas. La regla de los nueve divide la superficie corporal en varias áreas, de las cuales cada una corresponde a 9% o a un múltiplo de 9%. La cabeza y cuello representan el 9% y cada extremidad superior corresponde a 9%. Las caras anterior y posterior del tronco representan el 18%, al igual que cada extremidad inferior; los genitales corresponden a 1%. En el caso clínico mencionado, el paciente tiene una quemadura que afecta la extremidad superior derecha (9%), cara anterior del tronco (18%) y la extremidad inferior derecha (18%) para un total de 45%. (Schwartz, p. 228) 47. D) 48. A) 49. A) 50. E)

Explicación de las preguntas 47 a 50 El paciente tiene síntomas de hipotensión ortostática cuando se cambia de posición supina a sentado después de una hematemesis masiva y necesita la reposición urgente de volumen intravenoso para evitar la lesión a órganos terminales por hipovolemia. Los estudios serológicos para hepatitis, lavado gástrico, corrección de coagulopatía y estudios radiológicos son importantes, pero deben retrasarse hasta que el paciente se encuentre hemodinámicamente estable.

La colocación de bandas esofágicas por endoscopia es el procedimiento preferido para este paciente. En términos de eficacia la colocación de bandas es superior a la escleroterapia para interrumpir una hemorragia por varices, aunque la escleroterapia puede ser una segunda opción si la colocación de bandas falla o se incrementa el sangrado. La intervención quirúrgica en pacientes con varices esofágicas hemorrágicas conlleva un riesgo muy elevado por la hipertensión portal y coagulopatía concomitante. El tratamiento con inhibidores de la bomba de protones es de poca utilidad en este paciente porque la enfermedad no es consecuencia de reflujo gastroesofágico. La derivación portosistémica intrahepática transyugular no es el tratamiento de primera línea para la hemorragia aguda por varices. La persona tuvo una recurrencia de hemorragia por varices esofágicas que se trató en forma exitosa con colocación de bandas. La colocación de bandas rara vez elimina por completo las varices en una sola sesión. A menudo se necesitan varias sesiones para completar la trombosis y para cicatrizar las varices hasta el grado en que no haya un nuevo cuadro de hemorragia. Por lo general la escleroterapia no se realiza en pacientes que responden a la colocación de bandas. El tratamiento con dosis elevadas de inhibidores de la bomba de protones no ayuda a evitar la recurrencia de la hemorragia por varices esofágicas. La derivación portosistémica intrahepática transyugular no es necesaria en pacientes en quienes se controló la hemorragia por vía endoscópica y han evolucionado bien. El tratamiento endoscópico es preferible a la intervención quirúrgica en el tratamiento ambulatorio del paciente con hipertensión portal. En individuos con hipertensión portal a menudo se observan varices gástricas; responden mal al tratamiento endoscópico con escleroterapia o colocación de bandas. Es posible la intervención quirúrgica en estos pacientes, pero tiene un riesgo elevado dada la inestabilidad hemodinámica. La derivación portosistémica intrahepática transyugular puede realizarse en forma urgente y podría descomprimir el sistema portal y tiene la mejor posibilidad de interrumpir la hemorragia por varices gástricas. (Harrison’s Principles of Internal Medicine, pp. 252-253, 1760-1761)

http://bookmedico.blogspot.com

http://bookmedico.blogspot.com

BIBLIOGRAFÍA

American Academy of Pediatrics. The Red Book, 2003 Report of the Committee on Infectious Diseases, 26th ed. Elk Grove Village, IL: American Academy of Pediatrics, 2003. American Psychiatric Association. Diagnostic and Statistical Manual of Mental Disorders, 4th ed., Text Revision. Washington, DC: American Psychiatric Association, 2000. American Psychiatric Association. Practice Guideline for the Treatment of Patients with Borderline Personality Disorder. Washington, DC: American Psychiatric Association, 2001. Beckmann CRB, Ling FW, Laube DW, et al. Obstetrics and Gynecology, 4th ed. Baltimore, MD: Lippincot Williams & Wilkins, 2002. Braunwald E, et al. Harrison’s Principles of Internal Medicine, 15th ed. New York, NY: McGraw-Hill, 2001. Cameron JL (ed.). Current Surgical Therapy, 8th ed. St. Louis, MO: C.V. Mosby, 2004. Cameron JL (ed.). Current Surgical Therapy, 8th ed. St. Louis, MO: C.V. Mosby, 2004. Cotran RS, Kumar V, Collins T. Robbins Pathologic Basis of Disease, 6th ed. Philadelphia, PA: W.B. Saunders, 1999.

Fleisher GR, Ludwig S (eds.). Textbook of Pediatric Emergency Medicine, 4th ed. Baltimore, MD: Lippincott Williams & Wilkins, 2000. Hoffman R, et al. Hematology: Basic Principles and Practice, 3rd ed. New York, NY: Churchill Livingstone, 2000. Kunimoto DY, et al. The Wills Eye Manual: Office and Emergency Room Diagnosis and Treatment of Eye Disease, 4th ed. Philadelphia, PA: Lippincott Williams & Wilkins, 2004. Larsen PR, et al. Williams Textbook of Endocrinology, 10th ed. Philadelphia, PA: W.B. Saunders, 2003. Ruddy S, et al. Kelley’s Textbook of Rheumatology, 6th ed. Philadelphia, PA: W.B. Saunders, 2001. Rudolph CD, Rudolph AM, Hostetter MK, Lister GE, Siegel NJ, et al. (eds.). Rudolph’s Pediatrics, 21st ed. New York, NY: McGraw-Hill, 2003. Sadock BJ, Sadock VA. Kaplan and Sadock’s Synopsis of Psychiatry, 9th ed. Philadelphia, PA: Lippincott Williams & Wilkins, 2003. Schwartz SI, Shires GT, Spencer FC, et al. (eds.). Principles of Surgery, 7th ed. New York, NY: McGraw-Hill, 1999.

http://bookmedico.blogspot.com

359

http://bookmedico.blogspot.com 360

11: Examen de práctica 4

http://bookmedico.blogspot.com

http://bookmedico.blogspot.com

Índice alfabético

El número de página seguido por f o c en cursiva indica figuras o cuadros, respectivamente.

A Abdomen agudo, 140, 158 Abdominal, aneurisma de la aorta complicaciones posoperatorias, 275, 287, 346, 356 diagnóstico, 84, 84f, 105f factores de riesgo, 84, 105 tratamiento, 80, 102 Ablación electroquirúrgica con asa, procedimiento (LEEP), 165 Aborto, 139, 157, 286 Absorciometría con rayos X de energía dual (DEXA), 8, 41, 220, 238 Acalasia, diagnóstico, 77, 97, 320-321, 330-331 tratamiento, 73, 73f, 92 Acantosis nigricans, 140, 158 Acatisia, 325, 335 ACE, inhibidores. Véase Angiotensina, inhibidores de la enzima convertidora de (ACE) Acetilsalicílico, ácido, síndrome de Reye y, 125, 136 Aciclovir, 16-17, 50 Acidobásicos, trastornos, 76, 95-96, 276-277, 288 Acné, 25, 57 Acrodermatitis enteropática, 131, 131f, 134 Actínica, queratosis, 244 Adenocarcinoma, colon, 255, 255f, 264 esófago, 7, 40 gástrico, 85, 106 pulmonar, 254-255, 254f, 263-264 rectal, 80, 101 Adenoma, hepático, 300, 310-311 paratiroideo, 73, 93 velloso, 81, 102 Adenosina, 31, 63 ADHD. Véase Hiperactividad con déficit de atención, trastorno (ADHD) Adolescentes, causas de muerte, 339, 349 conservación de la salud, 217, 234 exámenes antes de competencias deportivas, 217, 233, 296, 307, 339, 349 muerte cardiaca súbita, 296, 307 tabaquismo, 340, 350 Adrenalina, anafilaxis, 27, 58 anestésicos locales, 26, 57-58 AGES, escala, 352 Agua, seguridad, 223, 243 Aguda, fiebre reumática. Véase Reumática, fiebre insuficiencia renal. Véase Renal, insuficiencia necrosis tubular (ATN), 324, 333-334 Agudo, glaucoma de ángulo cerrado, 10, 43-44 Alcohol, abuso, 192, 199, 205, 209 interacciones tóxicas con paracetamol, 5, 38 supresión, 301, 311 Alimentación, trastornos, 147-148, 169, 180, 197, 318, 328 Alimentos, alergia, 320, 330 seguridad, 223, 243 Alopurinol, 32, 65 Alprazolam, 311 Alquilantes, fármacos, 32, 65

Alucinógenos, 190, 206-207 Alzheimer, enfermedad, datos en resonancia magnética nuclear, 194, 210 tratamiento farmacológico, 11, 44-45, 194, 210 Amamantamiento, 149, 170 Amaurosis, 326, 337, 347, 357 Amenorrea, anorexia nerviosa, 328 hipotalámica, 141, 158 primaria, 140, 158, 273, 284 Amiloidosis, 252, 252f, 261 Aminoglucósidos, 18, 51 Amobarbital, 204 Amotivacional, síndrome, 191, 207 Anafilactoide, púrpura, 324, 334 Anafilaxis, 26-27, 57, 58, 320, 330 Anal, carcinoma, 80, 101 Andrógenos, exceso, síndrome, 139, 157 insensibilidad, 140, 158 Anemia, 20-21, 53-54, 121, 134, 326, 337 Anestésicos locales con adrenalina, 26, 57-58 Anfetaminas, intoxicación, 206 Angina, 28, 60 Angiotensina, inhibidores de la enzima convertidora de (ACE), efectos secundarios, 31, 63 embarazo 151, 171-172 hipertensión, l, 35, 329 mecanismo de acción, 63 Animales, mordeduras por, 20, 53 Anorexia nerviosa, 318, 328 Anorrectal, absceso y fístula, 79, 100 Ansiedad, trastorno, 189, 206 Antibióticos, C. difficile, enterocolitis, 79, 99 mordeduras de gato, 20, 53 resistencia, 129 sensibilidad cruzada, 27, 58 Anticoncepción, oral. Véase Orales, anticonceptivos puerperio, 149, 170 de urgencia, 140, 157, 321, 331 Antidepresivos, disfunción sexual, 182-183, 199200 selección, 179, 196 trastornos de pánico, 186, 203 Antígeno prostático específico (PSA), 225, 245, 282 Antineoplásicos, 32, 65. Véase también Quimioterapia Antipsicóticos, 295, 305-306, 325, 335. Véanse también fármacos específicos Antirreumáticos modificadores de la enfermedad, fármacos (DMARD), 328-329 Antisocial, trastorno de la personalidad, 15 Aorta, lesiones por desaceleración, 75, 94-95 Aórtica, estenosis, 29-30, 62 Apendicitis, 78, 78f, 98, 98f Apendicular, carcinoide, 78, 82, 99, 104 Apoptosis, 253, 261-262 Aprendizaje, trastornos, 191-192, 208 aPTT. Véase Tromboplastina parcial activada, tiempo (aPTT)

http://bookmedico.blogspot.com

ARR. Véase Riesgo absoluto, reducción (ARR) Arteria mesentérica superior, síndrome, 312 Arteriopatía carotídea, 84, 105 Arteritis de células gigantes, 341, 350 Artritis, gonocócica. Véase Gonocócica, artritis reactiva. Véase Reactiva, artritis reumatoide. Véase Reumatoide, artritis séptica. Véase Séptica, artritis Ascarosis, 120, 134 Asesoramiento genético, 2, 36 Asherman, síndrome, 141, 158-159 Asma, clasificación de la gravedad, 63-64, 64c diagnóstico, 27, 31, 58-59, 63 embarazo, 152, 172 tratamiento, 27, 31, 59, 64 Asperger, trastorno, 205 Ateroembólica, enfermedad, 14, 47 Ateroesclerosis, arterias de miembros inferiores, 84-85, 105 Ativan. Véase Lorazepam ATN. Véase Aguda, necrosis tubular (ATN) Auricular, fibrilación, 28, 59 Autoinmunitaria, anemia hemolítica, 356 Autosómicas, enfermedades, 2, 36 Avascular, necrosis, 75, 95 B Bacillus anthracis, 225-226, 245-246 Bacteriana, endocarditis, profilaxis, 218-219, 235236, 236c Bacteriano, absceso, 25, 25f, 57 Bacteriemia en lactantes, 113, 128 Bandas elásticas, ligadura, 79, 100 Barbitúricos y alcohol, 192-193, 209 Barrett, esófago, 40, 78, 85, 98, 104, 106 Basocelular, carcinoma, 25, 56-57, 224, 244 Bazo, amiloidosis, 252, 252f, 261 traumatismos, 77, 77f, 97 BCG, vacuna, 123, 136 Beckwith-Wiedemann, síndrome, 117, 131 Bender, prueba gestáltica visual-motora, 208 Benzodiazepinas. Véase también fármacos específicos ansiedad generalizada, trastorno, 189, 206 efectos hipnóticos, 11, 44 supresión alcohólica aguda, 301, 311 Betalactámicos, antibióticos, 16, 50 Biliar, cólico, 76, 96 Bilioso, vómito, 70, 89-90, 114, 128 Biopsia, endometrial, 143, 163 mama, 71, 90 Bioterrorismo, 225-226, 246 Bipolar, trastorno, manifestaciones clínicas, 180, 184, 197, 201, 277, 289 tratamiento, 180, 193-194, 197, 210 Bisfosfonatos, 146, 168 Bloqueadores beta, después de infarto miocárdico, 28, 60 fobias, 184, 201 Bocio nodular tóxico, 324, 334

361

http://bookmedico.blogspot.com 362

Índice alfabético

Boerhaave, síndrome, 98 Borrelia burgdorferi, 19, 52 BRCA1, mutación, 153, 174 Bronceado, cabinas de, 224, 244 Bronquiolitis, 132 Buerger, enfermedad, 256, 265 Bulimia, 180, 197, 318, 328 Bupivacaína, 45 Bupropión, 182, 200, 206 Burkitt, linfoma de, 252, 261 Buspirona, 11, 45, 189, 206 C C1, deficiencia, 26, 58 Cabeza femoral, necrosis avascular, 75, 95 Cabinas de bronceado, 224, 244 Cadera, artritis séptica, 74, 94 Cálculos biliares, anticonceptivos orales, 341, 351 diagnóstico, 87, 87f, 109f íleo, 346-347, 346f, 357 indicaciones quirúrgicas, 87, 108 manifestaciones clínicas y diagnóstico, 76, 96 pancreatitis, 4-5, 5f, 37-38 Cambios mínimos, enfermedad, 301, 311 Campanario, signo, 119, 132-133 Canal medular, estenosis, 271, 281 Capacidad vital, 75, 94 Carboplatino, 153, 174 Carbunco, 225-226, 245-246 Carcinoembrionario, antígeno (CEA), 310 Carcinoide, 78, 82, 99, 104, 264 síndrome, 86, 108 Cardíaco, taponamiento, 278, 289 Carotideo-subclavia, derivación, 83, 105 Casos y testigos, estudio, 240, 284 Catéter, septicemia, 79, 99-100 Causas de muerte, adolescentes, 339, 349 lactantes, 347, 357 CCK. Véase Colecistocinina (CCK) CEA. Véase Carcinoembrionario, antígeno (CEA) Cefálicas, lesiones, 74-75, 94, 94c Cefalosporinas, 16, 50 Ceguera y diabetes, 10, 43 Celecoxib, 21, 54 Celexa. Véase Citalopram Células gigantes, arteritis de, 341, 350 tumores, 256, 265 Centinela, ganglio linfático, biopsia, 71, 90-91 Cerebral, edema, 12, 45-46 Cervicales, lesiones, 144, 164 Cervicouterino, cáncer, 142-143, 144, 162, 164-165, 270, 281 Cetoacidosis, 276-277, 288 Cimetidina, interacciones medicamentosas, 11, 44 Cimitarra, signo, 127 Cirrosis, 70, 90 Ciruela, abdomen en, síndrome, 111, 126 Cistitis, 74, 93 Citalopram, 185, 202 Citocromo P-450, enzimas, 60 Claudicación intermitente, 85, 105-106 Clonazepam, 311 Clonidina, 202 Clostridium difficile, enterocolitis, 79, 99, 270, 280 Clozapina, 21, 54, 200 Coagulación, factores de, 21, 54, 344, 345, 354, 355 infrarroja, 79, 100 Cobalamina, deficiencia, 21, 53-54 Cocaína, dependencia, 190, 207, 277, 289 Coccidioides immitis, 17-18, 51 Cohorte, estudio, 240, 284 Colangitis esclerosante, 297, 297f, 308 primaria (PSC), 6, 39 Colecistitis, diagnóstico, 76, 88, 96, 109 manifestaciones clínicas, 76, 96 Colecistocinina (CCK), 83, 104 Colectomía, 72-73, 92 Colelitiasis. Véase Cálculos biliares Colesterol, concentraciones, arteriopatía coronaria, factores de riesgo, 272, 283 diabetes mellitus, 215, 230

Colestiramina, 30, 62 Cólico, biliar, 76, 96 ureteral, 75, 95 Colitis isquémica, 275, 287 Colon, embriología, 88, 109 Colonoscopia, 6-7, 39 Colorrectal, cáncer, adenocarcinoma, 255, 255f, 264 diagnóstico, 81, 81f estadificación, 102, 102c factores genéticos, 102 recomendaciones para detección, 6-7, 39, 213, 228 tratamiento, 80-81, 101, 102 Colostomía o ileostomía, 86, 108 Columna cervical, traumatismos, 74, 93 Compartimental, síndrome, 72, 91 Competencias, examen médico antes de, 217, 233, 296, 307, 339, 349 Conducta, trastornos, 187, 204 Conductual, tratamiento, 185, 202 Confianza, intervalo de (CI), 218, 221, 226-227, 235, 240, 246-247 Congénitas, anomalías, frecuencia, 153, 173 in utero, 141, 159 cardiopatías, 341, 351 Congestiva, insuficiencia cardiaca, manifestaciones clínicas, 325-326, 335-336 tratamiento, 29, 61, 325-326, 336 Conización cervical “en frío”, 144, 165 Consentimiento informado, 272-273, 283-284 Conservación de la salud, consultas, 36 años de edad, varones, 271, 282 50 años de edad, varones, 213, 228 65 años de edad, mujeres, 214, 229 Constitucional, retraso en el crecimiento, 119-120, 133 Consultorio, examen en, 36 años de edad, varones, 271, 282 50 años de edad, varones, 213, 228 65 años de edad, mujeres, 214, 229 Conversivo, trastorno, 181, 198, 283, 349 Convulsiones, embarazo, 152, 173 fiebre, 121-122, 134-135 COPD. Véase Neumopatía obstructiva crónica (COPD) Coriocarcinoma, 256, 265 Córnea, infecciones, 294, 304-305 Coronaria, derivación arterial (CABG), indicaciones, 29, 61 selección del vaso, 82, 104 Corporal, área de superficie, 347, 358 Coxsackie, infección por virus, 132 Crecimiento, intrauterino, retraso, 286 retraso, 119-120, 133 Crohn, enfermedad, complicaciones, 296, 306 diagnóstico diferencial, 38, 100-101, 295-296, 306 tratamiento quirúrgico, 79, 100 Cronkhite-Canada, síndrome, 39 Crowe, signo, 269, 280 Cuadriplejía, 74, 93 Cuello, cirugía de, 73, 92 masas en, 300, 311 Cuello uterino, insuficiencia, 286 Curling, úlceras, 74, 93 Cushing, síndrome, 320, 330 Cutáneo, cáncer, 224, 243-244 carbunco, 225-226, 245-246 CH CHARGE, asociación, 117, 131 Chlamydia, 112, 126 Chlamydia trachomatis, 115, 130, 310 Choque, 277, 288-289 D Dactilitis, 296, 306 De Quervain, tiroiditis de, 322, 331-332 Decisiones, capacidad de toma de, 318, 327-328 Dehiscencia, 72-73, 92 Delirantes, ideas, trastorno, 283 Delirio, 182, 194-195, 198-199, 211

http://bookmedico.blogspot.com

Demencia, capacidad de toma de decisiones, 318, 327-328 diagnóstico, 194, 210 fármacos que deben evitarse, 194, 210 por múltiples infartos, 183, 200 pronóstico, 194, 210 vascular, 183, 200 Dependencia, trastorno de la personalidad, 342, 351-352 Depresión. Véase también Bipolar, trastorno detección, 229 puerperal, 150, 170, 181, 198 recurrencia, 193, 210 riesgo de suicidio, 179, 196, 270, 281 síndrome de Cushing, 320, 330 tratamiento, 179, 191, 193, 196, 208, 209 Dermatitis, de contacto, 123-124, 130, 136 herpetiforme, 38 Dermatomiositis, 299, 299f, 309 Dermoide, quiste, 135 Desarrollo, trastorno difuso del, 205, 188, 205 Desintegración infantil, trastorno, 205 Desplazamiento, 352 DEXA. Véase Absorciometría con rayos X de energía dual (DEXA) Diabetes mellitus, complicaciones visuales, 10, 43 concentraciones de colesterol, 215, 230 diagnóstico, 3, 37 embarazo, 152, 172, 220-221, 239 inhibidores de la enzima convertidora de angiotensina, 15, 48 pancreatitis, 8, 40 tipo 2, 319, 329 tratamiento, 294, 305 Diabética, cetoacidosis, 276-277, 288 Diarrea del viajero, 223, 243 Didelfo, útero, 159 Dietilamida del ácido lisérgico (LSD), 205 Difenilhidantoinato, efectos secundarios, 124, 136 Digoxina, insuficiencia cardiaca, 29, 61, 336 mecanismo de acción, 31, 63 Discinesia tardía, 188, 205 Dismorfismo corporal, trastorno, 283 Disociación, 352 Distónica, reacción, 325, 335 Disulfiram, 199, 209 Diuréticos tiazídicos, 28, 60 Diverticulitis, 161-162 DMARD. Véase Antirreumáticos modificadores de la enfermedad, fármacos (DMARD) Doble burbuja, signo, 113, 127 Dolor progresivo, 327 Dopamina, antagonistas, 202 Down, síndrome, 112-113, 127, 234, 276, 287-288 Doxiciclina, enfermedad de Lyme, 19, 52 Drepanocitosis, 296, 306 Duelo, 186, 203 Dunphy, signo, 98 Duodenal, atresia, 112-113, 127 compresión, 301, 312 úlcera, 83, 86, 104, 107 DVT. Véase Trombosis venosa profunda (DVT) E Eagle-Barrett, síndrome, 111, 126 Eclampsia, 151-152, 172 Ecocardiograma, aórtica estenosis, 30, 62 fibrilación auricular, 28, 59 Ectópico, embarazo, después de ligadura tubaria, 139, 157 diagnóstico diferencial, 139, 157 Edad gestacional, 141, 149, 159, 170 cálculo con ecografía, 149, 170 Edwards, síndrome, 287 Eikenella corrodens, 52 Embarazo, anomalías congénitas, frecuencia, 153, 173 cambios, en aparato hematopoyético, 345, 355-356 respiratorios, 152, 172 cardiopatías, 152, 173

http://bookmedico.blogspot.com Índice alfabético categorías de riesgo de los fármacos, 141, 159, 343, 353 convulsiones, 152, 173 diabetes mellitus, 152, 172 diagnóstico diferencial, 139, 157 hemorragia transvaginal, 322, 332 hipertensión, 151-152, 171-172 hipertiroidismo, 150-151, 171 inhibidores de enzima convertidora de angiotensina, 151, 171-172 lupus eritematoso sistémico, 322, 331 molar, 274-275, 285-286, 344, 354 necesidades de hierro, 345, 355-356 primeros meses, 141, 159 vacunas, 148-149, 169, 219, 237-238 valoración prenatal, 224-225, 244-245 Embolia pulmonar, 290 Embriología del colon, 88, 109 EMDR. Véase Oculares, movimientos, desensibilización y reprocesamiento (EMDR) Endocarditis, profilaxis, 218-219, 235-236, 236c Endocrina múltiple, neoplasia (MEN), 85, 106 Endometrial, biopsia, 143, 163 Endometrio, cáncer, 143-145, 163-166 cicatrización quirúrgica, 141, 158-159 Endometriosis, 142, 160, 295, 306 Enterobius vermicularis, 120, 134 Enterococcus faecalis, 19, 52 Enterocolitis necrosante, 127 Enterovirus no relacionado con polio, 115, 129, 302, 313 Envenenamiento por cáusticos, 278, 289-290 Epidérmica tóxica, necrólisis (TEN), 24, 24f, 56 Epidermoide, carcinoma, anal, 80, 101 cutáneo, 224, 244 Epiglotitis, 132 Epstein-Barr, virus, 319, 329 Eritema infeccioso, 132 Eritropoyetina, 303, 313 Escherichia coli, cistitis aguda, 74, 93 hemolítico-urémico, síndrome, 130 infección de vías urinarias, 276, 288 Esclerosante, colangitis, 297, 297f, 308 Esferocitosis hereditaria, 326, 337 Esófago, cáncer, 7, 40, 83, 104 divertículo, 83, 104 espasmo, 86, 107 obstrucción, 320-321, 330-331 perforación, 78, 98, 347, 357 varices, 70, 90, 348, 358 Especificidad, 216, 232-233, 302, 312-313 Espinales, nervios, 74, 93 Esplenectomía, 77, 97 Esprue celiaco, complicaciones, 5, 38 diagnóstico, 5, 38 Esquistosomosis, 242-243 Esquizoafectivo, trastorno, 201 Esquizofrenia, 193, 201, 209 Esquizofreniforme, trastorno, 293-294, 305 Esquizoide, trastorno de la personalidad, 352 Estatinas, 3-4, 37, 215, 230, 283 Esteatorrea, pancreatitis crónica, 7, 40 Estimulantes, 202-202 Estómago, cáncer, 85, 106 irrigación, 86, 107 linfoma, 85, 106, 271, 282 Estreñimiento en enfermedad de Hirschsprung, 77, 97 Estreptococos grupo A (GAS), estado de portador, 115, 129 faringitis, 115, 129 impétigo, 116, 130 Estreptococos grupo B (GBS), colitis, 132 recién nacidos, 114-115, 117-118, 129, 131 Estrés postraumático, trastorno (PTSD), 187, 204-205 Estrógenos, tratamiento de sustitución. Véase también Hormonal, tratamiento de sustitución (HRT) después del tratamiento del cáncer endometrial 144-145, 165-166 riesgo de cáncer endometrial, 32, 65

Estroma del cordón sexual, neoplasias, 310 Evans, síndrome, 346, 356 Evitación, trastorno de la personalidad, 351 Ewing sarcoma, 256, 265 Éxtasis. Véase MDMA F Familiar, poliposis adenomatosa (FAP), 39, 80-81, 102 Faringoesofágico, divertículo. Véase Zenker, divertículo Fascitis necrosante, 79, 99 Fecal, incontinencia, 147, 168 Femoral, nervio, lesión quirúrgica, 156, 160, 176 FeNa. Véase Fracción excretada de sodio (FeNa) Fenciclidina (PCP), 188-189, 199, 205-206, 206 Fenobarbital, interacciones con warfarina, 28, 60 Feocromocitoma, 253, 262 Fetal, alcoholismo, síndrome, 123, 136 vigilancia, 154-155f, 156, 177 FHH. Véase Hipercalcemia, hipocalciúrica familiar (FHH) Fibroadenoma, 253, 262 Fibromialgia, 350 Fibroquística, mastopatía, 253, 262 Fibrosis quística, 151, 171 Fiebre, convulsiones por, 121-122, 134-135 en recién nacidos, 113, 128 Fingidos, trastornos, inducidos por el cuidador, 180-181, 197-198 Fisiológica, anemia, del embarazo, 355 ictericia, 296-297, 307 Fisostigmina, 199 Flumazenil, 199 Fluoruro, complementos, 222, 241 Fluoxetina, 44, 200 Flurazepam, 44 Fobia, 183-184, 201 Folato, deficiencia, 21, 53-54 Fólico, ácido, 151, 171 Fórmulas para lactantes, 118, 132 Fracción excretada de sodio (FeNa), 14, 47 Fractura, brazo, 118, 131 complicaciones, 75, 95 preescolares, 111-112, 111f, 126 tibial, niños, 111-112, 111f, 126 Furosemida, insuficiencia cardiaca, 29, 61 G Gamma-aminobutírico, ácido (GABA), 201 Gastrectomía, 86-87, 108 Gástrica, úlcera, 76, 86, 95-96, 107 Gástrico, cáncer, 85, 106 linfoma, 85, 106, 271, 282 Gastrina, aumento de las concentraciones, 77, 97, 321, 331 fisiopatología, 88, 109 funciones, 83, 104 Gastrinoma, diagnóstico, 88f fisiopatología, 88, 109 neoplasia endocrina múltiple, 85, 106 ubicación, 75, 95 Gastrointestinales, hormonas, 83, 104 Gato, mordedura de, 20, 53 Genitofemoral, nervio, lesión quirúrgica, 160 GERD. Véase Reflujo gastroesofágico, enfermedad (GERD) Germinativas, tumores de células, 300, 310. Véase también Ovárico, cáncer Gestacional, neoplasia trofoblástica, 344, 354 Giardia lamblia, 113, 128 Gingival, hiperplasia, 124, 136 Glasgow, escala de coma, 74, 94, 94c Glaucoma agudo de ángulo cerrado, 10, 43-44 Glucagonoma, 298, 308 Glutamato, 201 Gonocócica, artritis, 41 Gonorrea, 112, 126 Goodpasture, síndrome, 258, 266 Gota, cambios histológicos, 294, 294f, 305 diagnóstico diferencial, 41

http://bookmedico.blogspot.com

363

fisiopatología, 305 tratamiento, 9, 42 Gottron, signo, 309 Gráficos de crecimiento, 227, 227f, 247 Graves, enfermedad, 150-151, 171. Véase también Hipertensión Grey Turner, signo, 98 Grupo testigo, asignación al azar con, estudio, 273, 284 GTN. Véase Gestacional, neoplasia trofoblástica Guillain-Barré, síndrome, 114, 129 H Haloperidol, 185, 202 Halsted, mastectomía radical, 82, 103 Hammans, chasquido, 78, 98 Hashimoto, tiroiditis de, 251, 251f, 260 Hemangioma, 323, 333 Hematuria, 339, 349 Hemodiálisis, 303, 313-314 Hemofilia, 118, 131 Hemolítica, anemia, 356 Hemolítico-urémico, síndrome (HUS), 117, 130 Hemorrágico, infarto, 258, 266 Hemorroides, 79, 100 Henoch-Schonlein, púrpura (HSP), 324, 334 Heparina, 29, 61trombocitopenia inducida por, 323, 333 Hepático, adenoma, 300, 310-311 hemangioma, 323, 333 Hepatitis, detección en el embarazo, 225, 244-245 diagnóstico, 15-16, 48-50 vacuna, 229 Hereditario no poliposo, cáncer colorrectal, síndrome (HNPCC), 6-7, 39, 144, 166 Heridas, cicatrización, 300, 310 infección, 79, 87, 99, 108 Hernia, 74, 94 Heroína, 182, 199, 205 Herpes simple, adquirido en etapa perinatal, 297, 307-308 infección corneal, 294, 304-305 manifestaciones clínicas, 25, 25c 56 Herpes zoster, 25, 26, 56, 57 Hidatiforme, mola. Véase Molar, embarazo Hidrocefalia con presión normal (NPH), 200, 210 Hidronefrosis, 251, 251f, 260, 275, 286 Hidrópico, edema, 262 Hierro, anemia por deficiencia, 20-21, 53-54, 121, 134 Hígado, adenoma, 300, 310-311 anatomía, 86, 107-108, 107f colangitis eslcerosante, 297, 297f, 308 hemangioma, 323, 333 trasplante, 72, 92 Hiperactividad con déficit de atención, trastorno (ADHD), 184, 191, 201-202, 204 Hipercalcemia, 341, 351 hipocalciúrica familiar (FHH), 341, 351 Hipercortisolismo, 320, 330 Hiperlipidemia, recomendaciones para la detección, 213, 228, 271, 282 tratamiento, 3-4, 37, 215, 230 Hiperparatiroidismo, después de tiroidectomía 75, 94 hipercalcemia o, 351 hipocalcemia después de cirugía, 345-346, 356 primaria, 73, 93, 269, 280 tratamiento, 22, 54 Hiperpotasemia, 12-13, 46-47 Hipertensión, anticonceptivos orales, 341, 351 clasificación, 215, 231, 231c, 304c complicaciones renales, 302-303, 313-314 embarazo, 151-152, 171-172 pautas para tratamiento, 216, 231, 294, 304 recomendaciones para la detección, 213, 228 valoración inicial, 1, 35, 293, 304 Hipertiroidismo, embarazo, 150-151, 171 puerperio, 23, 55-56 tiroiditis subaguda, 332 tratamiento, 22, 55

http://bookmedico.blogspot.com 364

Índice alfabético

Hipertrófica, miocardiopatía, 217, 233 Hipocalciemia, 345-346, 356 Hipocondriasis, 208, 272, 283 Hipofosfatemia, 277, 288 Hipopotasiemia, 76, 95 Hipotiroidismo, lactantes, 121, 134 síndrome de Down, 112, 127 tiroiditis, durante el puerperio, 343, 353-354 de Hashimoto, 251, 251f, 260 Hipotonía, 112, 127 Hipovolemia, 76, 95, 277, 288-289, 348, 358 Hirschsprung, enfermedad, 77, 97, 249, 259 Histerectomía, complicaciones, 142, 161 estudio de Papanicolaou después de, 144, 165 Hiponatremia hipotónica, 12, 45-46 Hipoparatiroidismo, 22, 54 Hipoglucemia, 23, 55 Histoplasmosis, 17, 17f, 50-51 HMG-CoA, inhibidores de la reductasa. Véase Estatinas HNPCC. Véase Hereditario no poliposo, cáncer colorrectal, síndrome (HNPCC) Hodgkin, enfermedad, 254, 263 Hormona de crecimiento, deficiencia, 133 Hormonal, tratamiento de sustitución (HRT). Véase también Estrógenos, tratamiento de sustitución riesgo de cáncer endometrial, 32, 65 riesgos y beneficios, 2-3, 36-37 HSP. Véase Henoch-Schonlein, púrpura (HSP) Hueso hambriento, síndrome, 345-346, 356 Huevo, alergia, 320, 330 HUS. Véase Hemolítico-urémico, síndrome (HUS) I Ictericia, fisiológica, 296-297, 307 por leche materna, 307 IgA, nefropatía, 256, 265 Ileostomía, diferencias con colostomía, 86, 108 Imipramina, 186, 203 Impétigo, 116, 116f, 130 Implantación del huevo, 141, 159 Inanición, 274, 285 Incontinencia urinaria, 147, 168 de urgencia, 147, 168 Indometacina, 42 Infarto miocárdico, manifestaciones clínicas, 28, 60-61 tratamiento, 29, 61 farmacológico, 28, 60 Infecciones, pruebas, 123, 135 Influenza, vacuna, 148-149, 169, 219, 237 Infrarroja, coagulación, 79, 100 Injerto contra hospedador, enfermedad, 332 Inmunitaria, púrpura trombocitopénica (ITP), 116-117, 130, 356 Inmunizaciones, adultos, 213, 214, 229, 271, 282 alergia al huevo, 320, 330 embarazo 148-149, 169, 219, 237 niños con inmunodepresión, 111, 126 niños, 229 recién nacidos, 300, 309-310 tétanos, 78, 99, 219, 236-237 viajes al extranjero, 223, 242-243 Inmunodeficiencia humana, virus de la (VIH), adquirida en etapa perinatal, 124, 136, 148, 169 artritis reactiva como síntoma inicial, 42 detección, 148, 169 efectos celulares, 257, 266 infección aguda, 275, 286-287 infecciones oportunistas, 20, 52-53 nefropatía, 15, 48, 274, 285 Inmunodepresión en pacientes trasplantados, 72, 92 Insulina, 23, 55, 294, 305 Intelectualización, 352 Interferón gamma, 257, 266 Interleucina-2, 257, 266 Intervalo de confianza (CI), 218, 221, 226-227, 235, 240, 246-247

Interventricular, comunicación, 341, 351 Intestinal, obstrucción, 81, 81f, 102-103 Intestino, delgado, biopsia, 5, 38 medio, vúlvulo del, 70, 89-90, 127, 128 Intraabdominal, presión, aumento, 74, 93 Intraepitelial epidermoide de alta malignidad, lesión (HGSIL), 144, 164-165 Intususcepción, 69, 69ff 89 Isquemia mesentérica aguda, 84, 105 Isquémica, colitis, 275, 287 ITP. Véase Inmunitaria, púrpura trombocitopénica J Jalea de grosella, evacuaciones, 89 Jones, criterios, 128 Juvenil, artritis reumatoide, 135, 317, 327 K Kallmann, síndrome, 141, 158 Kawasaki, enfermedad, 118-119, 131-132 Klinefelter, síndrome, 288 L Labial, aglutinación, 146, 167 Lactantes, causas de lesión, 112, 127 causas de muerte, 347, 357 Ladd, bandas, 789 Laparoscopia, 74, 93 Laringotraqueobronquitis vírica, 119, 132-133 Leche, materna, ictericia por, 307 de vaca, intolerancia a las proteínas, 99, 132 Leche y alcalinos, síndrome, 54 LEEP. Véase Ablación electroquirúrgica con asa, procedimiento (LEEP) Lesiones, causas en lactantes, 112, 127 Leucemia, crónica linfocítica, 252, 261 niños, 113, 128 Leucocoria, 9, 42 Lewy, cuerpos, enfermedad, 186-187, 204 Leydig, tumores de células, 97 Limítrofe, trastorno de la personalidad, 342, 352 Linfocitos citotóxicos, 257, 266 Linfoide, hiperplasia, 257, 266 Linfoma, niños, 113, 128 gástrico, 85, 106, 271, 282 tipos, 254, 263 Linfoquiste, 142, 161 Lípidos, perfil de, 213, 228, 271-282 Lipoide, nefrosis, 300, 311 Lisch, nódulos, 280 Lisinopril, 319, 329 Litio, 11, 44 Lorazepam, efectos secundarios, 199 síndrome de Tourette, 202 supresión etílica aguda, 301, 311 trastorno de pánico, 186, 203 LSD. Véase Dietilamida del ácido lisérgico (LSD) Lupus eritematoso sistémico (SLE), 321-322, 331 Lyme, enfermedad, 19, 19f, 52 Lynch, síndrome. Véase Hereditario no poliposo, cáncer colorrectal, síndrome (HNPCC) M Macrosomía, 172 Magnesio, sulfato de, 152, 172 Malrotación, 114, 128 Maltrato infantil, fracturas, 126 frecuencia, 112, 127 Mama, lesiones. Véase también Mamario, cáncer mujeres jóvenes, 147, 168 tipos, 253, 262 valoración, 33, 66 Mamario, cáncer. Véase también Mama, lesiones diagnóstico, 71, 82, 82f, 90, 344-345, 344f, 354355 factores de riesgo, 355 ganglio linfático centinela, biopsia, 71, 90 genética, 153, 174 inflamatorio, 253, 262 pronóstico, 345, 355 tratamiento, 82, 103

http://bookmedico.blogspot.com

Mamografía, 33, 66 Manchas de color vino, 323-324, 333 Maniaco, episodio, 277, 289. Véase también Bipolar, trastorno Mano-pie, síndrome, 296, 306 Mano-pie-boca, enfermedad, 132 Mantoux, método. Véase Proteínico purificado, derivado (PPD) Marfan, síndrome, 217, 233-234 Marihuana, 191, 205, 207-208 Mastectomía, 71, 103 Mastitis, 149, 170 McBurney, punto, 98 McCune-Albright, síndrome, 140, 158 MDMA, 190, 206-207 Meckel, divertículo, 70, 90, 345, 356 Mejillas rojas, enfermedad, 132 Melanoma, factores de riesgo, 224, 244 manifestaciones clínicas, 255, 264 pronóstico, 26, 57, 81, 103 Memantina, 11, 44-45 MEN. Véase Endocrina múltiple, neoplasia (MEN) Meningitis, aséptica, 115, 129 bacteriana, 115, 129 lactantes, 113, 128 Menopausia, 2-3, 36-37 Menstrual, ciclo, 142, 160 Mesentérica, isquemia, 84, 105 Metaanálisis, 240 Metabólica, acidosis, 276-277, 288, 342-343, 353 alcalosis, 76, 95-96 Metformina, 336 Metilfenidato, 201, 202 Metotrexato, artritis reumatoide, 319, 328-329 soriasis, 25, 57 Metronidazol, enterocolitis por C. difficile, 79, 99, 270, 280 infecciones por Giardia lamblia, 113, 128 Mieloma múltiple, 15, 47-48, 253-254, 253f, 262263 Molar, embarazo, 274-275, 285-286, 344, 354 Mononucleosis infecciosa, 319, 329 Montañas rocosas, fiebre exantemática de las, 122, 135 Mucocele, 135 Müeller, anomalías de los conductos, 141, 159 Muerte súbita infantil, síndrome (SIDS), 347, 357 Mujeres deportistas, tríada, 234 Murphy, signo, 98, 109 N Naloxona, 199 Naltrexona, 192, 209 Narcisista, trastorno de la personalidad, 196 Nasolagrimal, obstrucción del conducto, 122, 135 Nefroesclerosis hipertensiva, 302-303, 313-314 Nefropatía por medio de contraste, 47 Nefrótico, síndrome, 15, 47-48, 274, 285 Negación, 352 Negativo, valor predictivo (NPV), 217, 232-233, 301-302, 312-313 Neumatosis intestinal, 127 Neumocócica, vacuna, 229, 325 Neumonía, extrahospitalaria, 278-279, 279f, 290, 290c niños, 111, 126 P. carinii, 290 Neumopatía obstructiva crónica, (COPD), 32, 64-65 Neumotórax, 69, 89 Neural, defectos del tubo, 151, 171 Neurofibromatosis-1, 269, 280 Neuroléptico maligno, síndrome, 188, 205 Nictalopía, 7, 40 Nilo occidental, virus, 16, 50 Niñera, codo de, 131 Nitroglicerina, angina, 28, 60 interacción con sildenafil, 45 Nitroprusiato sódico, 30, 63 NNH. Véase Número necesario, para dañar (NNH)

http://bookmedico.blogspot.com Índice alfabético NNT. Véase Número necesario, para tratar (NTT) No hodgkiniano, linfoma, 252, 261 Noradrenalina, 201 Norwalk, virus, 128 NPH. Véase Hidrocefalia con presión normal (NPH) NPV. Véase Negativo, valor predictivo (NPV) Número necesario, para dañar (NNH), 220, 238 para tratar (NTT), 216, 220, 232, 238 Nutrición parenteral, 75, 94 O Obsesivo-compulsivo, trastorno (OCD), 185, 202 Obstétrica, lesión, 147, 168 Obturador, nervio, lesión quirúrgica, 142, 160 OCD. Véase Obsesivo-compulsivo, trastorno (OCD) Oculares, movimientos, desensibilización y reprocesamiento (EMDR), 202 Oftalmia neonatal, profilaxis, 300, 310 Oliguria, 323, 333 Oposición desafiante, trastorno (ODD), 204 Orales, anticonceptivos, anticoncepción de urgencia, 140, 157 contraindicaciones, 341, 342, 351, 352 efectos secundarios, 341, 351 mecanismo de acción, 320, 330 reducción del riesgo de cáncer ovárico, 153, 173-174 Órbita, celulitis, 25, 57 Óseas, neoplasias, 250, 250f, 256, 260, 265 Osteoclastoma, 265 Osteoporosis, concentraciones séricas de calcio, 147, 168 diagnóstico, 8, 41 factores de riesgo, 8, 41, 140, 147, 157, 168 pruebas de detección, 220, 238 recomendaciones para la detección, 214, 229 tratamiento, 9, 42, 146, 168, 220, 238-239 Osteosarcoma, 256, 265, 317, 327 Otitis, externa, 123, 135 media, 115, 129 Ovárico, cáncer, características, 255, 264 genética, 153, 174 manifestaciones clínicas, 274, 285 pruebas diagnósticas, 274, 284, 300, 310 quimioterapia adyuvante, 153, 174-175 reducción de factores de riesgo, 153, 173-174 tratamiento quirúrgico, 155, 176c Ovario, quiste, 161 torsión, 145-146, 167 tumores de baja malignidad, 154, 175 Ovarios poliquísticos, síndrome (PCOS), aspecto del ovario, 256, 264-265 diagnóstico, 24, 56, 124, 136, 140, 158 inducción de hemorragia menstrual, 142, 160 manifestaciones clínicas, 24, 56, 124, 136 tratamiento, 124, 136 Óxido nitroso, 11, 44 P Paclitaxel, 153, 174 Paludismo, prevención, 112, 242 PAN. Véase Poliarteritis nudosa (PAN) Páncreas, cáncer, 8, 40, 41 gastrinoma, 75, 95 Pancreatitis, aguda, 74, 93, 326, 336-337 cálculos biliares, 4-5, 4f, 37-38 crónica, 7-8, 40 Pánico, trastorno, 185-186, 203 Papiledema, 9-10, 43 Papilotomía,, 4-5, 38 Paracetamol, interacción con alcohol, 5, 38 Parainfluenza, virus, 132 Paranoide, trastorno de la personalidad, 179, 196, 352 Paratiroides, glándulas, adenoma, 73, 93 funciones, 121, 134 lesión quirúrgica, 75, 94 Parenteral, nutrición, 75, 94 Parkinson, enfermedad, 183, 200-201

Paroxetina, 186, 202, 203 Parvovirus, 132 Pasteurella multocida, 52 Patau, síndrome, 287 PCOS. Véase Ovarios poliquísticos, síndrome (PCOS) PCP. Véase Fenciclidina (PCP) Pélvica inflamatoria, enfermedad, absceso tuboovárico, 299, 308 diagnóstico, 146, 167 infertilidad, 139, 157 tratamiento, 146, 167 Pélvico, absceso, 161 dolor, 145-146, 167, 298, 308 Pélvicos, órganos, prolapso de, 145, 166 Pénfigo vulgar, 323, 332-333 Penicilina, alergia cruzada, 27, 58 mecanismo de acción, 16, 50 Péptica, estenosis, 7, 40 úlcera, 76, 95-96 Pericárdico, taponamiento, 278, 289 Peridiverticular, absceso, 161 Peroneo, nervio, compresión, 161 Peutz-Jeghers, síndrome, 39 Pica, 188, 205 Pick, enfermedad, 200, 210 “Pico de pájaro”, aspecto, 107 PID. Véase Pélvica inflamatoria, enfermedad Pilórica, estenosis, 70, 89 obstrucción, 76, 95-96 Placenta previa, 322, 332 Plan B, 321, 331 Plaquetarios, trastornos, 345, 355 Plasmaféresis, 129 Plasmáticas, células, discrasias, 253-254, 253f, 262-263 Plummer, enfermedad, 324, 334 PMS. Véase Premenstrual, síndrome (PMS) Pneumocystis carinii, 290 Poder estadístico, 218, 235 Poliarteritis nudosa (PAN), 252, 260-261 Polimialgia reumática, 340-341, 350 Polio, enterovirus no relacionado con, 115, 129 vacuna, 242 Pólipos colónicos, 81, 102 Poliposis adenomatosa familiar (FAP), 39, 80-81, 102 Poliuria, 120, 133 Porcelana, vesícula biliar en, 87, 108, 334 Positivo, valor predictivo (PPV), 216-217, 232-233, 301-302, 312-313 PPD. Véase Proteínico purificado, derivado (PPD) PPI. Véase Protones, inhibidores de la bomba de (PPI) PPV. Véase Positivo, valor predictivo (PPV) Precoz, pubertad, 140, 158 Prednisone, 329 Preeclampsia, 151-152, 172 Preescolares, fracturas, 111-112, lllf, 126 Premenstrual, síndrome (PMS), 273-274, 284-285 Prenatal, consulta, 224-225, 244-245 Presión arterial, medición, 231. Véase también Hipertensión Primidona, 10-11, 44 Probenecid, 9, 42 Procaína, 45 Propiltiouracilo, 22, 55 Propranolol, 23, 56 Prospectivo con asignación al azar y con grupo testigo, estudio, 240, 273, 284 Protamina, sulfato, 21, 54 Proteínico purificado, derivado (PPD), detección de tuberculosis, 222, 240-241 infección por virus de inmunodeficiencia humana, 20, 53 valoración de resultados, 123, 135 Protones, inhibidor de la bomba de (PPI), 2, 36 Protrombina, tiempo (PT), 21, 54 Pruebas de detección, 50 años de edad, varones, 213, 228-229 65 años de edad, mujeres, 214, 229

http://bookmedico.blogspot.com

365

PSA. Véase Antígeno prostático específico (PSA) PSC. Véase Colangitis esclerosante, primaria (PSC) Pseudomonas aeruginosa, 123, 135 Psicosis puerperal, 150, 170, 198 PT. Véase Protrombina, tiempo (PT) PTSD. Véase Estrés postraumático, trastorno (PTSD) Pubertad, edad normal de inicio, 273, 284 precoz, 140, 158 signos, 120, 133 tardía, 120, 133, 139, 157 Pudendo, nervio, lesión quirúrgica, 160-161 Puerperal, depresión, 150, 170 psicosis, 150, 170, 198 tiroiditis, 343, 353-354 Pulgar, signo, 133 Pulmonar, cáncer, adenocarcinoma, 254-255, 254f, 263-264 células pequeñas, 33-34, 66, 300, 309 contraindicaciones para resección pulmonar, 76, 96 edema, 29, 61 embolia, 290 Púrpura anafilactoide, 324, 334 Q Quemaduras, cálculo del área de superficie corporal, 347, 358 complicaciones, 74, 93, 324, 333-334 tratamiento inicial, 72, 91-92 Queratosis actínica, 244 Quetiapina, 183, 200-201 Quimioterapia, mecanismo de acción, 32, 65 regímenes, 32-33, 66 Quinidina, 28, 30, 30f, 60, 62-63 Quinta enfermedad, 132 Quística, fibrosis, 151, 171 R Ranson, criterios, 74, 93, 336 Raquitismo, 134 Razón de probabilidades, 218, 221, 235, 240 Reactiva, artritis, 8-9, 41-42 Rectal, cáncer, 80, 101-102 Rectovaginal, fístula, 147, 168 Recuerdos vívidos, 190, 207 Reed-Sternberg, células, 254, 263 Reflujo gastroesofágico, enfermedad (GERD), complicaciones, 7, 40 manifestaciones clínicas, 2, 35 tratamiento, 2, 36 Regional, enteritis, 76, 96 Reitan-Indiana, prueba neuropsicológica, 208 Reiter, síndrome. Véase Reactiva, artritis Renal, acidosis tubular, 342-343, 353 adenocarcinoma, 256, 265 insuficiencia, 323, 333, 333c diagnóstico diferencial, 323, 333 enfermedad ateroembólica, 14, 47 necrosis tubular aguda, 324, 333-334 Resfriado de verano, 302, 313 Respiratoria, alcalosis, 277, 288 Respiratorio, virus sincitial (RSV), 347, 358 Retina, desprendimiento, 326, 337 Retiniana central, arteria, oclusión, 347, 357 Retinoblastoma, 9, 42-43 Retraso mental, 113, 127 Rett, trastorno, 188, 205 Reumática, fiebre, 114, 128 Reumatoide, artritis, 257, 266, 318-319, 328-329 Reye, síndrome, 125, 136 Rhus, dermatitis, 124, 136 Rickettsia rickettsii, 135 Riesgo absoluto, reducción (ARR), 216, 232 Riesgo relativo, 216, 231-232 reducción, 216, 232 Risperidona, 182, 198, 200, 325, 335 Robo subclavio, síndrome, 83, 105 Rotavirus, 128 Rovsing, signo, 98 Rubéola, vacunación, 229

http://bookmedico.blogspot.com 366

Índice alfabético

S S3, galope, 325, 335 Sacacorchos, patrón, 107 Salbutamol, 31, 64 Secretina, 104 Seminoma, 77, 96-97 Sensibilidad, 216, 232-233, 302, 312-313 Séptica, artritis, 74, 94, 256, 265 Series de casos, estudio, 284 Serotonina, 201 inhibidores selectivos de recaptación de (SSRI), depresión, 191, 208 síndrome premenstrual, 274, 285 trastorno limítrofe de la personalidad, 342, 352 Sertralina, 187, 204-205, 342, 352 Seudodemencia, 186, 203-204 Seudogota, 41 Seudomembranosa colitis, 132 Seudotumor cerebral, 43 Sheehan, síndrome, 22, 55, 141, 150, 159-160, 170 Sildenafil, 11, 45 Simulación, 192, 208-209 Sinusal, histiocitosis, 254, 263 Sistémico, lupus eritematoso (SLE), 321-322, 331 Solar, protección de la luz, 224, 244 Somatización, trastorno, 283, 340, 349-350 Somatomedina C, 133 Soplos, 269, 307 cardiacos, 296, 307 Soriasis, 25, Stevens-Johnson, síndrome, 332 Still, enfermedad, 135, 317, 327 Streptococcus pneumoniae, meningitis bacteriana, 115, 130 neumonía extrahospitalaria, 279, 290 Sturge Weber, síndrome, 323-324, 333 Súbita, muerte cardiaca, 296, 307 Suicidio, 270, 281 Superior, arteria mesentérica, síndrome de pinzamiento, 301, 312 trombosis, 84, 105 T T, etapa, 238 Tabaquismo, interrupción, 32, 65, 189, 206, 349 pasivo, 223-224, 243 Tabicado, útero, 159 Tacrolimo, 18, 51 Talla baja, 119-120, 133 Tamoxifeno, 143, 163-164 Taquicardia, 31, 63 Tasa de riesgo, 226-227, 246-247 Taxol. Véase Paclitaxel Temporal, arteritis, 341, 350 TEN. Véase Epidérmica tóxica, necrólisis (TEN) Teofilina, 125, 137 Teriparatida, 42 Testicular, disgenesia, 288 Testiculares, tumores, 77, 96-97, 256, 265 Tétanos, 78, 99, 219, 236-237 Tetraciclina, 125, 137 TIA. Véase Transitoria, isquemia cerebral (TIA) Tiamina, 199 Tiazídicos, diuréticos, 28, 60 Timolol, 28, 60 Tiña del cuerpo, 130 Tioridazina, 200 TIPS. Véase Transyugular intrahepática portosistémica, derivación (TIPS) Tiroidectomía, 73, 92 Tiroideo, cáncer, clasificación, 352 diagnóstico, 250, 250f, 259-260 pronóstico, 342, 352

tratamiento, 73, 92-93, 300, 311 Tiroideos, nódulos, 22, 23, 54-55, 55, 324, 334 Tiroiditis, de Hashimoto, 251, 251f, 260 De Quervain, 322, 331-332 puerperal, 343, 353-354 subaguda, 322, 331-332 Torácica, sonda, 71, 91 Torácico, conducto, lesión quirúrgica, 82, 103 traumatismo, 71, 91 Tórax, traumatismo, 278, 278f, 289 Tos crónica, 2, 35-36 Tourette, síndrome, 185, 202 Tóxica, necrólisis epidérmica (TEN), 24, 24f, 56 Tóxico, bocio nodular, 324, 334 Tóxicos, ingestión, 278, 289-290 Toxocara canis, 134 Tracoma, 15, 130, 310 Transitoria, isquemia cerebral (TIA), 84, 105 Transvaginal, hemorragia, cáncer cervicouterino, 142-143, 162 embarazo, 322, 332 posmenopáusica, 143, 163 Transversal, estudio, 240, 273, 284 Transyugular intrahepática portosistémica, derivación (TIPS), 348, 358 Traqueal, intubación, 77, 97 Traqueoesofágica, fístula, 77, 97 Trasplante, hepático, 72, 92 inmunodepresión, 72, 92 rechazo, 322, 332 Traumatismo, abdominal, 77, 77f, 97 aórtico, 74, 94-95 cefálico, 74-75, 94, 94c columna cervical, 74, 93 esplénico, 77, 77f, 97 penetrante de tórax, 278, 278f, 289 torácico, 71, 91 valoración, 71, 91 Trazodona, 205 Triamtereno, 23, 55 Triángulo de Calot, 87, 109 Triazolam, 311 Tricíclicos, antidepresivos, 29, 61 Tricomonosis, 126 Trimetoprim/sulfametoxazol, 20, 51-52 Trisomía 13, 287 Trisomía 18, 287 Trisomía 21. Véase Down, síndrome Trombocitopenia, inducida por heparina, 323 333 niños, 116-117, 130 Tromboplastina parcial activada, tiempo (aPTT), 21, 54 Trombosis venosa profunda (DVT), 87, 108, 161 Trompas de Falopio, ligadura, 139, 157 Tuberculosis, detección, 123, 136, 222, 240-241 epidemiología, 252, 260 infección por virus de inmunodeficiencia humana, 20, 53 Tuberosa, esclerosis, 117, 131 Tuboovárico, absceso, 299, 308 Tumoral, lisis, síndrome, 12-13, 32, 46-47, 65 Turcot, síndrome, 39 Turner, síndrome, 139, 157, 234, 288 U Úlcera, gástrica, 86, 107 péptica, 76, 95-96 Ulcerosa, colitis, cáncer de colon, 264 colangiocarcinoma y, 6, 39 colangitis esclerosante primaria, 6, 39 complicaciones, 39

http://bookmedico.blogspot.com

diagnóstico diferencial, 6, 6f, 38, 100-101 manifestaciones clínicas, 6, 6f, 39, 80, 100-101 tratamiento, 39 Unicorne, útero, 159 United States Preventive Services Task Force, 228 Uréteres, cálculos, 75, 95 lesiones quirúrgicas, 72, 81, 92, 103 Uretritis química, 120, 133 Urgencia, anticoncepción de, 140, 157, 321, 331 Uterinos, miomas, 145, 166 Útero, anomalías congénitas, 141, 159 V Vaciamiento gástrico rápido, síndrome, 108 Vacunas. Véase Inmunizaciones Vaginal intraepitelial, neoplasia (VAIN), 143, 162-163 Valor predictivo. Véase Negativo, valor predictivo; Positivo, valor predictivo Varicela, 226, 246 VATER, asociación, 126 Vejiga hiperactiva, síndrome, 147, 168 Vena cava superior, obstrucción, 300, 309 Venlafaxina, 200 Ventilación, ajustes, 303, 314 mecánica, 303, 314 Ventricular, fibrilación, 29, 61 Verapamil, 60 Vesícula biliar, anatomía, 87, 109 carcinoma, 324, 334 porcelana, 87, 108, 334 Viagra. Véase Sildenafil Viajero, diarrea del, 223, 243 medicina preventiva en el, 223, 242-243 Vías urinarias, infección de (UTI), lactantes, 276, 288 niños, 120, 133 prolapso de órganos pélvicos, 145, 166 Vibrio vulnficus, 18-19, 18f, 52 Vida media de los fármacos, 5, 38 Violencia intrafamiliar, 222-223, 241-242 Viruela, 226, 246 vacuna, 242 Vitamina A, 300, 310 Vitamina B12, 21, 53-54, 344, 354 Vitamina C, 134 Vitamina D, 22, 54, 134 Vómito, bilioso, 70, 89-90, 114, 128 en proyectil, 70, 89 von Recklinghausen, enfermedad, 269, 280 Vulva, enfermedades, 146, 153, 154, 167, 173, 175 Vulvovaginal, atrofia, 343, 353 W Warfarina, 21, 54 Wechsler, escala de inteligencia para niños, 208 Wellbutrin. Véase Bupropión Wernicke, encefalopatía, 199, 301, 311 Wilms, tumor, 249-250, 259 X X frágil, cromosoma, síndrome, 190-191, 205, 207 Y Yatrogenia, 218, 234-235 Yodo, deficiencia, 121, 134 Z Zenker, divertículo, 83, 86, 104, 107, 275, 286 Zidovudina, 18, 51 Zinc, deficiencia, 75, 94, 121, 121f, 134 Zollinger-Ellison, síndrome, 77, 95, 97, 109 Zoloft. Véase Sertralina